0% found this document useful (0 votes)
12 views

IOS Master (1)

The document discusses the complexities of statutory interpretation, emphasizing the role of ambiguity and misunderstanding in legal contexts. It outlines various rules and principles of interpretation, the functions of courts, and notable cases that illustrate these concepts. The article concludes by exploring the challenges of inferring legislative intent and the implications for judicial interpretation of statutes.

Uploaded by

Nirbhay
Copyright
© © All Rights Reserved
We take content rights seriously. If you suspect this is your content, claim it here.
Available Formats
Download as PDF, TXT or read online on Scribd
0% found this document useful (0 votes)
12 views

IOS Master (1)

The document discusses the complexities of statutory interpretation, emphasizing the role of ambiguity and misunderstanding in legal contexts. It outlines various rules and principles of interpretation, the functions of courts, and notable cases that illustrate these concepts. The article concludes by exploring the challenges of inferring legislative intent and the implications for judicial interpretation of statutes.

Uploaded by

Nirbhay
Copyright
© © All Rights Reserved
We take content rights seriously. If you suspect this is your content, claim it here.
Available Formats
Download as PDF, TXT or read online on Scribd
You are on page 1/ 137

Ambiguity and Misunderstanding in Titles

Law
Preamble
Can Legislatures Constrain Judicial
Interpretation Deeming Provision

Interpretation and Construction Headings

Classification of Statutes Marginal Notes

Sources of Legislative Intent Explanation

Basic Rules of Interpretation Punctuation Marks

Sentia Legis Illustrations

Ex Visceribus Actus Proviso

Ut Res Magis Schedules

Fundamental Rules of Interpretation Exception/Saving Clause

Literal Rule Trauvax Preparatories

Mischief Rule External Aids of Int.

Golden Rule Historical Facts & Circumstances

Harmonious Construction Political, Social, Economic Developments

Subsidiary Rules of Interpretation Parliamentary History

Expressio Unius Reference to Other Statutes

Ejusdem Generis Textbooks

Causus Omissus Dictionaries

Reddendo Singula Singulis Foreign Decisions

Noscitur A Sociis Contemporanea Expositio

Constitutional Interpretation Presumptions in Interpretation


Principles Legislative Presumptions
Interpretation/Definition Clause Presumptions in Aid
Internal Aids to Interpretation Presumptions – Constitutional Validity
Presumption that Legislation does not Interpretation of Subordinate
contain Negatory Provisions Legislation
Presumption against Retrospectivity Interpretation of Fiscal Statutes
Presumption Against Exceeding Canons/Principles of Interpretation
Constitutional Powers
FS - Strict Construction
Principles of Natural Justice
FS - Beneficial Construction
Presumption - Oust or Restrict
Jurisdiction of Courts FS - Harmonious Interpretation
Presumption - Territorial Operation FS - Mischief Rule
Presumption against Injustice FS – External aids of int.
Presumption against Inconvenience
Exemption from Taxation
Statutes Affecting Jurisdiction of
Doctrine of Fairness
Courts

General Principles Pertaining to Interpretation of Penal Statutes


Jurisdiction of Courts Application rule of construction for
Cases of breach of statutory duties penal statutes

Omission to exercise statutory power Strict Construction of Penal Statutes

Extent of Exclusion Purposive Interpretation Approach

Tribunal Order when void Mens Rea in Statutory Offences

Exclusion of Jurisdiction of Superior Remedial v. Penal Statutes


Courts
Mimansa Rules
General and Special Statutes
Kinds of Mimansa
Kinds of Statutes
Axioms
Types of Repeal
Principles
Doctrine of Implied Repeal
Maxwell and Mimansa Rules
Expiry and Repeal of Statutes
Exceptions to Indian Courts (WN)
Subsidiary Circumstances of IR

Repeal (WN) Temp & Permanent Statute (WN)

Express Provision (WN) Consequences of Repeal (WN)


INTRODUCTION

Why do we need Statutory Interpretation?

 Broad terms used in the statute itself


o Usage of vague terms
o Statute may contain technical definitions -- simple words might not be defined in the
statute and may be open to multiple interpretations.
o Eg: 2005 Sept 9 - HSA - woman as coparcener in Hindu ancestral property -- issue
with ‘on the date of passing of this amendment’ became very ambiguous
 The date was the ambiguous term -- when will the right emanate from?
 If it is retrospective? - no
 Ambiguity
o Individual’s understanding is irrelevant -- for this we go to the ‘functions of the
court’
o Eg: ejusdem generis -- reading with the ‘same kind’ -- drafting error can be rectified
 When there is change in the use of Language
o Modern laws - more technical terms used in drafting
 Drafting or printing error

Functions of the court w.r.t interpretation:

 Find out the facts of the case when the dispute is brought to the court and find out the
issues
 Find out which law is applicable
 Analyse the law applicable
o Finding out the meaning of the law
o After meaning -- honour the meaning of clear and if not (due to ambiguity for
example) then apply to the best possible way
 Apply law to the facts to the particular case (imp.)
o Honouring the meaning of the law
 Judgement delivery - here the court interprets the law wherever necessary
 What ‘law’ means --
o Salmond -- Statute is a legislative will of the legislature
o Statute made by the Legislature
o Legislative will is addressed to the individual or categories of individuals (i.e. to
whom is the law addressed).
o A statute is the legislative will of the legislature. Therefore, the first function of the
persons to whom statutes are addressed – and that includes courts – should be to
find out the meaning of the statute and honour the meaning so found. That is what is
meant by analysis (finding out the meaning) and application (honouring the meaning
so found) of law.
 Checking constitutionality of law
o The court until and unless its challenged before the court will always presume its
constitutional until and unless proved otherwise.
“More often than before, statutory text is the ending point as well as the starting point for
interpretation”
 Interpretation within 4 corners of law
 Court will only look in the statutory text for interpretation
 Statutory text

Reed Dickerson’s View

 “When law is imperfect, incomplete, absent the judge has to extend and cover the case at
hand. If there is a directly applicable law, the judge applies the law otherwise supplies the
law..” -- Cognition and creation -- Reed Dickerson
o Functions of the court:
 cognition (ascertaining the meaning of the statute)
 creation (assigning the meaning)
 Thus judges make the law in the guise of interpreting the existing law
 Legislative words are like empty vessels in which the judges pour judicial meaning
 A judge in this context has two difficult but closely related and equally important
responsibilities (Traditional Approach)
o To read the statute and find out how far it is applicable to the dispute at hand; and
o If the meaning so ascertained does not resolve the dispute, to adjust the law so that it
is applicable to the dispute and if necessary even create law for that purpose.
 Interpretation is essentially an act of recognition or discovery
 This task is very difficult because communication are often imperfectly framed and
inadequately transmitted.
o This results in a loss of corresponding part of the message. The reader has to make a
compensating effort to find the true meaning of the communication.
 He has to apply logical induction and deduction
 This is more difficult in case of complex communication such as statute
 Statute is the medium of communication between lawmaker and the law user.
 According to Keeton the function of judges:
o The judge has to decide upon the exact meaning of what legislature has actually
said, and
 This in real life situation is possible to an extent (my view)
 Exact meaning → literal interpretation
 Very difficult to establish the exact meaning of law
 How does the court do it?
 Analyse the law within the 4 corners of the law
 Use internal and then external aids
o The judge has to consider what the legislature might have intended to say, but did
not say because of some reason

Ambiguity and Misunderstanding in law - Sanford Schane (Asynchronous ||


Essay)

The present article begins with the fact that ‘words’ play a crucial role in the creation and
understanding of any legal context or matter. The interpretation of such legal provisions may at
times vary between individuals and this is when the necessity of the intervention of the courts is
duly recognised to assist in the removal of the ambiguities (be it in contracts or legislative acts).
The article further discusses 3 cases with respect to how courts have addressed the scope and issues
of ‘ambiguity’ in legal parlance.
1. Frigailment Importing Co. v. BNS International Sales Corpn. - better known as the
‘chicken’ case involves a contractual interpretation of “what chicken is based on the
contractual agreement”. The judge reasoned that the interpretation of ‘chicken’ by both the
sides are to be accepted and referred to the terms of the contract for being assisted with any
grounds of interpretation (if provided) and stated that "the word 'chicken' standing alone
is ambiguous
The case has also dwelled into the objective and the subjective theory attributable to
contract law; where these two theories play a crucial role in understanding the burden of
interpretation in contracts and one such key principle is that was explained by Arthur L
Corbin - "Contracting parties must be made to know that it is their written words that
constitute their contract, not their intentions that they try to express in the words. They, not
the court, have chosen the words;...”. This case thus follows the strict adherence of the plain
meaning rule. The essential principle of contract relating to the meeting of minds is also
quintessential to understanding the scope of the words included in the contract.
The judge also referred to the interpretation practice of ‘four corners’ (which basically deals
with the interpretation to be done within the ambit of the contract/text itself).
2. Raffles v. Wichelhaus - the case involves the transportation of bales of cotton between
Bombay to Liverpool on an English ship called “Peerless”. The issue arose due to the fact that the
parties were bonafidely unaware about the existence of two ships bearing the same name and
travelling between the same locations but at different times. The buyer and seller intended on the
transportation being done through the different ships without benign aware of each other’s intent.
The primary contention was that the contract did not explicitly mention which ship is to be used but
merely mentioned the name of the ship, thus leading to latent ambiguity.
The case, unlike the Frigaliment case deals with the subjective theory of contractual
interpretation that held that the contract lacked a meeting of minds (consensus ad idem),
thus not making the contract binding on the parties.
Courts sometimes make a distinction between latent ambiguity and patent ambiguity.
The latter applies when words in an agreement have more than one objective meaning.

3. Interstate Commerce Commission v. Allen E. Kroblin Inc - this case as well included
about the types of chicken and the category that they fall in - manufactured product or agricultural
product, as certain exemptions were granted to agricultural products. The court noted that the
classification of chicken bore ambiguous words.
The court relied on a point of view pertaining to the semantic enigma that arises from
vagueness of categorization, where two inherently different things should not be placed
together. The court also relied on the legislative history for means of extrinsic aid.
Hence with these 3 cases, the author then proceeded to define as to what ‘ambiguity’ exactly
means. Few key points are:
 Lack of clarity
 Doubtful or uncertain words
 Obscure meaning having indefiniteness of expression.
 Lexical ambiguity potentially occurs whenever a word has more than one objective or
dictionary meaning.
 Syntactic ambiguity is the other common type. It has to do with grammatical structure.
The article further goes on to explain the varying dictionary meanings and usage of the words that
lead to having multiple meanings thus expanding the scope of interpretation that was attributed to
the case by the judges.

In conclusion:
a. In Frigaliment, the ambiguity that the court realised was "lexical ambiguity".
b. In Raffles, the court was faced with "referential indeterminacy".
c. In ICC, the court encountered with "vagueness in categorization" when it came to the
ambiguous terms. The Court noted that the legislature had failed to provide definitions for the two
crucial terms mentioned in the Act

Can Legislatures Constrain Judicial Interpretation of Statutes?

One of the key methods of interpretation that has been discussed in the present article is
regarding the interpretation from the analysis of the legislative intent involved in the statute (which
is said to be an external mode of interpretation), which has always fallen in a vexatious grey area.
For example, the author has referred to to the method adopted by Prof. Moore who would have
liked to use "intention" as a possible way to find determinate meaning in a statute, but after
thorough consideration he reluctantly concluded that the meaning of a statute cannot be ascertained
by recourse to legislative intent; and that of Stanley Fish who contended that there can never be a
definitive theory or set of rules of interpretation.
Hence due to the fluidity that has existed since time pertaining to interpretation, jurists and
academicians have always felt that there exists a ‘meta-theory’ of interpretation that is quite often
required. One of the most notable methods of interpretation followed by the ‘deconstructionists’
(those who break-down the statute to gain a better understanding) is as follows:
Reading the statute → observing and choosing the best method of interpretation → explain
as to how the said rule was used and followed to arrive at a ground of simplification.
The Author lays down the hypothesis that he seeks to address though the article - the
anticipated mental context of addressees partially shapes the strategy of legislators in formulating
statutes; surmising primarily based on the contention of Stanley Fish.
Interpretation is said to be a stage in the process of communication between the legislature
that enacts a statute and the members of the judiciary (or practicing bar or the public) who are the
addressees of the statute. The primary concern of the legislature is to not have the statute declared
void by the vice of such interpretation.

1. Communication is constrained by the receivers mindset


 A statute is said to be effective from the legislator's point of view, if it is interpreted
with reasonable accuracy.
 Due to this, the legislators tailor the words of the statute so that the audience will
interpret it the way they intend on it being interpreted.
 There is an example about how communication through coded messages works in
the military.
 Any form of ambiguity is seen to arise or increase if the sender is increasingly
unsure about who the receiver is.
 Ascribing desires, beliefs, purposes of the author are said to be quintessential in the
process of deriving meaning and interpretation

2. Can the message prepare its audience?


o Basic surmise of interpretation is that the reader is a ‘reasonable’ reader which leads
to a subsequent ease in understanding so that the crux of the topic remains.
o The author has also cited the discussion between Einstein and Nabokov regarding
what is “common sense” and the difference in approach taken in their works.
 Nabokov's opening paragraph itself assumes the existence of a decoder in the
minds of his readers.

3. The Problem of Inferring Intent


o The approach taken here is distinct from the Fishian approach
o In reality it is observed that one can ever know with certainty what any author's
intention or strategy was.
o According to Fuller, statutes are enacted by legislatures in order to manage the
affairs of society with some degree of general rationality. When we read a statute,
we have to interpret it in light of this assumed rea-sonable purpose. Fuller's best
example, I think, is in his "Case of the Speluncean Explorers."
o Case: In Fehler v. Neegas there was before this Court for construction a statute in
which the word "not" had plainly been transposed from its intended position in the
final and most crucial section of the act. This Court refused to accept a literal
interpretation of the statute, and in effect rectified its language by reading the word
"not" into the place where it was evidently intended to go.
o It is also observed that usually whatever the Congress intended, the courts were able
to find some intent or purpose in the statutes that, in particular cases, led to results at
variance with what Congress intended.
o One such innovative approach that has been taken in this discussion is the “business
purpose rule” that developed in the Gregory v. Helvering Case -- which was used in
the interpretation of Tax Statutes which have been interpreted in the most strict
manner based on the nature of the legislation
4. Radical Interpretation:
o According to Professor Kelman -- most extreme and excessive cases have "critical"
rhetorical power.
o The author has referred to Dworkin’s position where he claims that one cannot or
should not misinterpret any text unless someone is deliberately perverse.
o Dworkin’s writing has also been criticized as a having scope of radical
interpretations; which when compared to that of Shakespeare is more liberal as the
exists a reasonable claim and understanding that the audiences that were far more
"knowing" about Shakespeare's actual "intent" than theatergoers have been since
then; when compared to Dworking’s works.

5. Jurisprudence as audience preparation:


o This section relies the role jurisprudence plays in the interpretation and
understanding of texts.
o The most difficult problems of audience pre-selection and preparation are not to be
found in literature but rather in jurisprudence. Legislators (and framers of
constitutions) have an unlimited number of words at their disposal, but they can use
them only once, and what they say has significant consequences.
o A statute once enacted lays down the law until it is amended; and even if it is
amended, the time gap between the original enactment and the amendment is
governed by the once-only choice of words that the legislature passed.
o Jurisprudence consists of theories of interpretation of law: meta-interpretive theories
that are supposed to stand apart from any particular body of law. A great deal of the
effort that has gone into jurisprudence has been devoted to attempts to define "law."
o Statutes are said to ideally favour one specific approach of jurisprudence.
o If an audience believes a given jurisprudential theory, that audience has forfeited a
few of its degrees of interpretive freedom, and that if the legislature knows this fact
about its audience, it may be able to use jurisprudence to its advantage.
o Any existing language qualifies as a system of at least as much complexity as
ordinary arithmetic, and hence Godel's proof applies to legal, textual, and linguistic
demonstrations.
o Hart's positivistic core-penumbra distinction to two propositions:
 (a) what we think of as falling within the core of the statute could be subject
to radical reinterpretation, and
 (b) what we think of as falling within the penumbra is the typical "arguable"
case.
Concluding points:
 If, the legislature knows in advance that its interpretive com munity will consist mostly of
positivists, its best strategy would be to write those specific exceptions into the statute.
(vehicle example of Fuller and Hart)
 the inclusion of specific exceptions requires the legislature to have foreseen the panoply of
desired exceptions in advance.
 there is no a priori reason why positivists rather than naturalists will take a wider view of
any given statute.
 that there will be a tendency toward varying breadths of interpretation depending upon the
general jurisprudential theory of the interpretive community.
 A legislature that knows in advance what that general jurisprudential theory is likely
to be will have a slightly more precise way of communicating the message it wants to
communicate than a legislature that is oblivious to this theory

Interpretation and Construction

 Salmond - “by interpretation or construction is meant, the process by which the courts seek
to ascertain the meaning of the legislation through…”
 Lord Denning in Seaford Court Estates Ltd. v. Asher
o “English knowledge is not an instrument of mathematical precision… it would
certainly save the judges from the trouble if the acts of parliament were drafted with
divine precision and perfect clarity, in the absence of it, when a defect appears, a
judge cannot simply fold hands and blame the draftsmen…”
 G Williams -- Words and phrases are symbols that stimulate mental references to the
refernents.
o But in the case of Deputy Chief Controller of Imports and Exports vs. K.T.
Kosalram -- “but words of any language are capable of referring to different context
and times”.
 After the enacting process the legislature becomes functus officio so far as that particular
statute is concerned, so that it cannot itself interpret it.
 The Legislature can no doubt amend or repeal any previous statute or can declare its
meaning but all this can be done only by a fresh statute after going through the normal
process of law making. -- J. P. Bansal v. State of Rajasthan, 2003 5 SCC 134
 Shyam Sundar vs. Ram Humar, 2001 8 SCC 24 -- Rules of interpretation are not our
masters, they are our servants. They are aids to construction, presumptions and pointers.
They are meant to assist the court in advancing the ends of justice

Interpretation Construction
 It is the art of finding true  It is the drawing of conclusion respecting
sense of any form of word, subjects that lie beyond the direct expression of
i.e., the sense which author the text from elements known from and given
intended to convey. in the text.
 It is the process referring to  It relates to the meaning of the words used in
both spirit and reason of law. the statute.
 It is the mental exercise to  Construction is the result of the mental
gather the intention of the exercise.
maker of the law.

 Interpretation find the meaning and construction establishes that meaning

 Meaning of Interpretation & Construction.


o Interpretation of statute is finding out the meaning of the statute in order to ascertain
the will of the legislature.
o Interpretation is the art of finding out the sense of any form of words,
while construction is the drawing of conclusions with respect to subjects which are
beyond the direct expression of the text.
o Therefore, when court goes beyond the language of the statute and seeks help from
external aids it is construing the statute.
o Justice Scalia in United Savings Association vs. Timbers of Inwood Forest
Associates 1988
 Statutory Construction is a holistic endeavour
 A provision that may seem ambiguous in isolation is often clarified by the
remainder of the statutory scheme because the same technology is used
elsewhere in a context that makes its meaning clear or because only one of
the permissible meanings produces a substantive effect that is compatible
with the rest of the law.
o Words used in a particular statute and the courts find that its ambiguous, the same
words may have meaning in the statute and the court can interpret it harmoniously --
from the same statute itself
o Intention of legislature == scheme of the legislation

Meaning and Classification of Statutes:

 Black’s Law -6th Ed:


o Statute - formal written enactment of a legislative authority that governs a
country/state/city etc; or declare policy.
o Typically statutes command ot prohibits something or declare policy -- often used to
distinguish law made by legislative bodies from the judicial decision of the common
law and regulations issued by Govt agencies.
 Will of the legislature
 Bouvier Law Dictionary:
o “A law established by the act of the legislative power, that is to say, an Act of the
legislature
o Among civilians -- the term “statute” is generally applied to laws and regulations of
every sort, every provision of the law which ordains, permits or prohibits anything is
designed a statute, without considering from what source it arises.”
 Bill: Act when introduced in the House of Parliament is known as Bill. It transforms into
Act, when passed by the Houses of Parliament, and the same approved by the Hon’ble
President of India. Bills are of four kinds:
o Ordinary Bill
o Private Members’ Bill
o Money Bill &
o Financial Bill

 Types of Statutes:
o Codifying- codify rewritten law on a subject
o Declaratory- merely declare or explain an existing law
o Remedial- alteration of common law or a judge made law
o Amending – alteration of statute
o Consolidating – when they consolidate several previous statute relating to the same
subject matter with or without alterations.
o Enabling- remove a restriction or disability
o Disabling – restrain the alienation of property
o Penal – when they impose a penalty.

Need for Interpretation:

 Complexity of statutes: The nature of the subject, numerous draftsmen and the blend of
legal and technical language can result in incoherence, vague and ambiguous language.
 The impossible task of anticipating every possible scenario also leads to the use of
indeterminate language.
 Language, words and phrases are an imprecise form of communication. Words can have
multiple definitions and meanings. Each party in court will utilize the definition and
meaning of the language most advantageous to their particular need.

Basic Principles of Interpretation:

Intention of the Legislature:

 When a statute is clear and unambiguous, the inquiry into legislative intent ends at that
point.
 When a statute could be interpreted in more than one fashion that legislative intent must be
inferred from sources other than the actual text of the statute.
 Mens or sententia legis (the duty of judicature is to act upon the true intention of the
legislature) It assimilates two aspects-
o What the word mean
o Conveys the concept of purpose and object (so process of construction is both literal
and purposive)
 Ita Scriptum est – as the words are
 Sources of Legislative Intent:
o The text of the bill as proposed to the legislative body
o Amendments to the bill that were proposed and accepted or rejected,
o The documentation of hearings on the subject
o Legislative records or journals
o Speeches and floor debate made prior to the vote on the bill
o Legislative subcommittee minutes, factual findings, and/or reports
o Other relevant statutes that can be used to understand the definitions in the statute on
question
o Other relevant statutes which indicate the limits of the statute in question
o Legislative files of the executive branch, such as the governor or president
o The constitutional invalidity of the other portions of the statute?)
o Legislative intent, which is the reason for passing the law

 Intention of legislature can be explored through:


o Words
o The context
o The subject matter
o The effects and consequences
o Spirit and reasons of law.
o A bare mechanical interpretation of the words and application of legislative intent
devoid of concept or purpose will reduce most of the remedial and beneficent
legislation to futility.
o If a statutory provision is open to more than one interpretation the court has to
choose that interpretation which represents the true intention of the legislature.
District Mining Officer v Tata Iron and Steel Co. (2001) 7 SCC 358
 One of the well-recognised canons of construction is that the legislature speaks its minds by
use of correct expression and unless there is ambiguity, the court should adopt literal
construction if it does not lead to absurdity. → Usha Devi v. Madhya Pradesh AIR 1990
MP 268. (Please refer to Roshan Lal Goswami v Gobind Raj AIR 1963 Punj 532.)
 The court is not entitled to alter the language of section to fit in with the supposed intention
of the legislature. → Dattatraya Baliram Naik v. Ramanbhai AIR 1962 Bom 236.
 Union of India v. Elphinstone spinning and weaving co. 2001 SC 724c- Supreme Court
approved the usage of literal rule as ‘cardinal principle of construction’
 The purpose of Interpretation of Statutes is to help the Judge to ascertain the intention of the
Legislature – not to control that intention or to confine it within the limits, which the Judge
may deem reasonable or expedient.
 Statute should be Construed so as to make it Effective and Workable – if statutory provision
is ambiguous and capable of various constructions, then that construction must be adopted
which will give meaning and effect to the other provisions of the enactment rather than that
which will give none.
 The process of construction combines both the literal and purposive approaches. The
purposive construction rule highlights that you should shift from literal construction when it
leads to absurdity.

According to Salmond, there are two cases in which the litera legis need not be taken as
conclusive and to find out the sententia legis external aid may be sought.
o When the letter of law is obviously defective and fails to express a single, definite
coherent and complete idea.
o When the text leads to such an unreasonable result that it is self evident that the
legislature could not have meant what it has said.
o This leads us to two types of interpretation.
 1. Grammatical or literal and
 2. Logical interpretation.
o (Important)
 Literal legis → literal meaning
 Sentia legis → intention of the legislature (i guess)
 Rule says that we must stick to the literal rule always and only if not possible
then we have to seek external aid
 Eg: Berriman case -- railway accident where court denied relief based on
interpreting the law based on literal rule
 Few cases:
o State of MP v. Balram Mihani (2010) 2 SCC 602, → “where language of a statute
is very general and not clear, the courts should interpret it with contextual
background.”
o Bharat Petroleum Corp. v. Maddula Ratnavalli (2007) 6 SCC 81 → “Where two
views are possible the view which satisfies the Constitutional rights or requirements
must be preferred.”
o Shanker Raju vs UoI WP C No. 311 of 2010 →
 In a court of law, what the legislature intended to be done or not to be done
can only be legitimately ascertained from what it has chosen to enact either
in express words or by reasonable and necessary implication
 Where the Legislature clearly declares its intent in the scheme of a language
of Statute, it is the duty of the Court to give full effect to the same without
scanning its wisdom or policy and without engrafting, adding or implying
anything which is not congenial to or consistent with such express intent of
legislature

o Kehar Singh v. State (delhi admin) 1988 3 SCC 609 →


 The court first examines the words of the statute. If the words are precise and
cover the situation in hand, there is no need to go further. Those words
should be expounded in the natural and ordinary sense.
 But if the words are ambiguous, uncertain or any doubt arises as to the terms
employed, it is the paramount duty of the court to put upon the language of
the legislature a rational meaning.
 The meaning of every word, section and provision should be examined. The
Act as a whole with reference to the necessity which gave rise to the Act has
to be examined. It should be considered to ensure coherence and consistency
within the law as a whole and to avoid undesirable consequences. For these
purposes Court calls in external and internal aids.
o Dadi Jagannadhan vs. Jammulu Ramulu 2001 7 SCC 71
 The court must start with presumption that legislature did not make a
mistake
 It must interpret so as to carry out the obvious intention of legislature.
 It must not correct or make up a deficiency, neither add or read into a
provision, words which are not there particularly when literal reading leads
to an intelligible result.
o Sultana Begum vs. Prem Chand Jain 1997 SCC 373
 It is the duty of the courts to avoid a head-on clash between two sections of
the Act and to construe the provisions which appear to be in conflict with
each other in such a manner as to harmonize them.
 The provisions of one section of a statute cannot be used to defeat the other
provisions unless the court, in spite of its efforts, finds it impossible to effect
reconciliation between them.
 Courts have to bear in mind that when there are two conflicting provisions in
the act which cannot be reconciled with each other , they should be so
interpreted that if possible effect should be given to both.

BASIC RULES OF INTERPRETATION:

The Essence of the law lies in the spirit, not in its letter, but letter are the only way in which
intentions are expressed. The words are external manifestation of intention that it involves. When
there is possibility of one or more interpretation of statute, courts has to adopt the interpretation
which reflects the ‘true intention of legislature’ which can also be considered as the legal meaning
of the statutory provisions. Intention of legislature is assessed either in express words or by
necessary implication keeping in mind the purpose or object of the statute.

Intention of the legislature always serves as reference to the meaning of words used by legislature
which are objectively determined. It is nowhere seen or expressly provided, it has to be assessed by
the guiding rules of interpretation.

 The intention of legislature shall have two aspects:-


o “Meaning’’: That which tells what the words mean.
o “Purpose and Object”: That which includes purpose and object of enacting the
statute. So, when intention of legislature is not found, it is assessed from the statute
with a combination of ‘meaning of the words’ and in the light of the purpose or
objects.
 Guiding lines to frame the intention of legislature are: –
o 1. The context
o 2. The subject matter.
o 3. The effects and consequences.
o 4. The spirit or reason of the law.

 How to interpret statutes


o Intention of Legislature
 Legislatures have supreme authority in Law making. Courts while
interpreting the law must strive to find out the real intention of Legislature.
o Statutes must be construed to make it effective and workable
 Sometimes provisions in law may be vague and inconsistent. This rule states
that law should be construed in such a way that it will make the law effective
and operative.
o Statutes must be read as a whole in it's context
 While construing the provisions in a statute , all the parts of the statute
should be taken together and read as a whole to find out the intention of the
legislature. They are therefore, not to be construed in isolation.
 This is one of the most important principles of legislation.
 According to this principle a statute is to be interpreted ‘Ex visceribus
actus.’

Sentia legis → intention of the legislature

Sententia Legis means to bring out the true meaning of law.


o The Sententia legis gives liberty to the courts to go beyond the literal meaning of the
words. Whenever there is a problem as to the meaning of a particular word
expressed in the statute then the courts can apply the logical interpretation that
brings out the true meaning of the words expressed in the statute.
 PROCESS OF INTERPRETATION
o The process of interpretation is very important as it conveys how to find out the true
intention of the maker and enables us to come to a suitable conclusion free from
absurdity.
o SALMOND has defined it as “the process by which the Courts seek to ascertain the
meaning of the Legislature through the medium of authoritative forms in which it is
expressed.”
o Karnataka State Financial Co operation v N . Narsimahaiah (2008) 5 SCC ; AIR
2008
 The Supreme Court said while interpretation the following questions shall
always be considered by the interpreter:
 (i) Why the particular law has been made?
 (ii) What is the purpose?
 (iii) Has the purpose been achieved?

 The process of interpretation has two elements namely ;-


o GRAMMATICAL RULE OF INTERPRETATION
 Grammatical interpretation provides for the literal construction. It means that
words of a statute are to be given their ordinary and natural meaning if such
meaning is clear. In such a case, effect should be given to the provisions of
the statute irrespective of the consequences.
 The language of the statute is to be interpreted according to the rules of
grammar.
 This is considered the safest rule of interpretation.
 Union Of India v Flip Tiago De Gama of Vedem Vasco De Gama (1990)
SCC 277 → The very purpose of interpretation is to provide a clear
legislative intent and to prevent unreasonableness, and when the words used
are themselves very clear then there is no need of further interpretation of it.
 If the language of the court is plain the duty of the court is only to give effect
to it.
 The courts should not construe words contrary to their ordinary and literal
meaning.
 There has been circumstances where courts have faced difficulties in the
process of grammatical interpretation like inconsistency & Ambiguity. The
inconsistency may arise when a particular law or statute is inconsistent with
one and other. The ambiguity may arise as a result of a particular word
having two or more meanings.

LOGICAL INTERPRETATION
 Logical Interpretation means such an interpretation aims to logically fulfill
the intention of the legislature. It is known by various names like- Doctrine
of liberal interpretation: sententia legis.
 Under it, courts have to find out the intention of the legislature by a thorough
study of statutes, and it has to see what the reasons for framing this statute
are.
 When the language of a statute is unclear or the object and meaning of
statute are not derived then the rules of logical interpretation are used to
know the intention of the statute. This was proved by the Allahabad High
court in Ramashreya v/s District Panchayat Raj Officer, Gorakhpur
 It is the duty of Judicature to act upon the true intention of the
legislature. This is guided by the Maxim “Sententia Legis” i.e., true
intention of legislature.
 The logical interpretation is based on the belief that the words used may have different
meaning than the maker has thought to have, and there may be a chance of
miscommunication and then the courts are at the liberty to peep in to the true nature of
words so expressed.
 There are certain rules of logical interpretation that can be followed to know the intention of
the legislature.
o a) Read the Statute as a whole.
 State of West Bengal v/s Union of India (1964) S C R 371—it decided that
the Court should not only depend on that section which is under
consideration before it, to know the intention of legislature. The court should
seriously consider every provision of Statute.
o b) Provisions of Statute should not be separated.
o c) Remove the Inconsistency of Words
o d) Consequences are not considered.
o e) Logical Construction should be done
o f) Equitable Construction
EX VISCERIBUS ACTUS
 The maxim ex visceribus actus is a long- recognized rule of construction. The Literal
meaning of the maxim is “within the four corners of the Act”.
 As per this principle a provision in a statute cannot be interpreted in isolation, because it
may lead to undesirable results not intended by the legislature. By reading the entire statute,
the meaning of the words can be interpreted with the aid of provisions and words contained
in the light of other sections in the same statute.
 The principle states that the whole context and intent of enacting a statute that is elucidated
through its preamble must be kept in mind while interpreting a provision of that statute.
This must be especially true when the meaning of the Section is dubious.
 When the provisions in a statute are interpreted in this way, the courts should not deviate
from the intention of the legislature. The scheme of the act as a whole should be the
guiding principle while a statute is interpreted by applying this rule.
 “The authorities on the interpretation of statutes generally agree that a statute is to be read
as a whole and that every clause is to be construed with reference to the other clauses of the
act and its context, to the greatest extent possible”, Maxwell on Interpretation of Statutes.
 Cases:
o In Poppatlal Shah v State of Madras, the supreme court observed that it is a settled
rule of construction that in order to ascertain the legislative intent, all the constituent
parts of a statute are to be taken together and each word, phrase or sentence is to be
considered in the light of the general purpose and object of the statute.
o ‘’If the choice is between two interpretations’’, said Viscount Simon, L.C. In Nokes
vs. Doncaster Amalgamated Collieries, Ltd. (AC at p. 1022) → “the narrower of
which would fail to achieve the manifest purpose of the legislation, we should read
the whole statute and avoid a construction which would reduce the legislation to
futility and should rather accept the bolder construction based on the view that
Parliament would legislate only for the purpose of bringing about an effective
result.”
o In M/s Philips India Ltd v. Labour Court Madras (AIR 1985 SC 1034) , the
Supreme court held that no canon of statutory construction is more firmly
established than that the principle that statute must be read as a whole. This is a
general rule of construction applicable to all statutes alike. This rule is so firmly
established that it is variously styled as elementary rule and as a settled rule.
 Every clause of statute must be construed with reference of the context and other clauses of
the act, to make consistent enactment of the whole statute.
The Contextual Approach
o Contextual reading is a well-known proposition of interpretation of statute. The
clauses of a statute should be construed with reference to the context vis-à-vis the
other provisions so as to make a consistent enactment of the whole statute relating to
the subject-matter. The rule of 'ex visceribus actus' should be resorted to in a
situation of this nature.
o This rule, that context must be used or the ex visceribus actus rule, that is ‘from the
inside of the Act’ was clearly implied in Broken Hill South Limited (Public
Officer) v The Commissioner of Taxation (New South Wales) (1937) 56 CLR 337:
 The first approach must be to ascertain whether the words have a clear meaning in the
general context in which they appear. That general context is the statute taken as a whole.
Which might appear to be unclear when read in isolation might be quite sensible when the
whole of the statute is considered. There might be other parts of the statute which show the
court how particular words and phrases were intended to be understood.
 A statute is best interpreted when we know why it was enacted.
 It is well-settled principle that while interpreting a statute, the interpretative function of the
court is to discover the true legislative intent. It must be read, first as a whole, and then
section by section, clause by clause, phrase by phrase and word by word and therefore,
taking into consideration the contextual connotation and the scheme of the Act, its
provisions in their entirety.

Case: Indian Handicrafts Emporium Vs. Union of India., (Judgement 27/08/2003) → “We are
concerned with the reason and object for which the amendments (to be read as law) have to be
made. We must take into consideration the text and context of the amending Acts and the
circumstances in which they had to be brought about.”

 Nasiruddin vs. Sita Ram Agarwal (2003) 2 SCC 577


o It is well settled that the real intention of the legislation must be gathered from
the language used.
o

 Superintendent and Remebrance of Legal Affairs to Govt of WB vs. Abani Maity (1979) 4
SCC 85
o “Exposition ex visceribus actus is a long-recognised rule of construction. Words in a
statue often take their meaning from the context of the statute as a whole. They are
therefore, not to be construed in isolation.
o For instance, the use of the word 'may' would normally indicate that the provision
was not mandatory. But in the context of a particular statute, this word may connote
a legislative imperative, particularly when its construction in a permissive sense
would relegate it to the unenviable position, as it were, 'of an in effectual angel
beating its wings in a luminous void in vain'.

 Balram Kumawat vs. UoI 2003 (7) SCC 628


o Brief Facts:
 The appellants M/s Unigems had imported mammoth fossil said to be of an
extinct species in the year 1937. The stock of mammoth fossil held by the
appellants is said to be periodically checked by the statutory authorities.
 According to the appellants the distinction between mammoth and elephant
ivory is that whereas mammoth belongs to an extinct species, the Ivory of
elephant is of an extant living animal. The appellants state that mammoth
ivory is distinguishable by visual and non-destructive means vis-a-vis
elephant ivory and even in convention on International Trade in Endangered
Species (CITES) their distinguishing features have been pointed out.
o Issue:
 Whether 'mammoth ivory' imported in India answers the description of
the words 'ivory imported in India' contained in Wild Life
(Protection) Act, 1972 (hereinafter referred to as 'the said Act') as amended
by Act No. 44 of 1991 is the question Involved in these appeals which arise
out of a common judgment and order dated 20.3.1997 passed by a Division
Bench of the Delhi High Court.
o Contention of Appellant:
 That the purport and object of the Act cannot be sub-served by placing a ban
on trade in mammoth Ivory, as it is distinguishable from Elephant ivory.
Taking us to the provisions of the said Act, mammoth ivory does not answer
the description of 'wild animal‘; therefore, the provisions contained in
Chapter VA of the said Act would not be attracted.
o Decision:
 What has been banned is ivory. There is complete prohibition of trade in
ivory. Such a complete prohibition is a reasonable restriction within the
meaning of Clause (6) of Article 19 of the Constitution of India.
 The law in no uncertain terms says that no person shall trade in ivory. It does
not say that what is prohibited is trade in elephant ivory or either types of
ivory.
 The Parliament, therefore, advisedly used the word 'ivory' instead of
elephant ivory. The intention of the Parliament in this behalf, in our opinion,
is absolutely clear and unambiguous. we cannot assume that the Parliament
was not aware of existence of different types of ivory. If the intention of the
Parliament was to confine the subject matter of ban under Act 44 of 1993 to
elephant ivory, it would have said so explicitly.
o As noticed hereinbefore, the object of the Parliament was not only to ban trade, in
imported elephant ivory but ivory of every description so that poaching of elephant
can be effectively restricted.

Ut Res Magis Valeat Quam Pereat


 The principle of UT RES MAGIS VALEAT QUAM PEREAT follows from the general
principle of interpretation pursuant to which interpretation must always seek to
effectuate the intention of the legislature. The literal meaning of the maxim is, ‘the
thing may have effect rather than be destroyed. This principle of the maxim was
enunciated on Murray v Inland Rev. Commissioner. It was stated that, the alternative that
will introduce uncertainty in the smooth working of the legislation should be rejected and
the alternative that promotes the object of the statute must be chosen.

 The maxim means that it is better for a thing to have an effect than to be made void, which
means that it is better to validate a thing instead of invalidating it. A statute is considered to
be an authentic repository of legislative will and therefore it is the function of the court to
interpret it according to the intent of them that made it. That function of the court has to be
abided by the maxim Ut Res Magis Valeat Quam Pereat.
 It is the duty of the court to try and avoid that construction which attributes irrationally to
the legislature. And hence must prefer such a construction which renders the statutory
provision constitutionally valid instead of making it void.
 In case of a situation where there are two constructions possible from a single
provision, of which one renders the provision inoperative while the other give effect to the
provision, the latter which gives effect to the provision is adopted and the former is
discarded. The court must hesitate if the interpretation does not give effect to the provision.
This is also know as rule of effectivess.
 Maxwell observes that if the choice is between two interpretations, the narrower one
would fail to achieve the manifest purpose of the legislation. Under such a circumstance,
we should avoid the construction which would reduce the legislation to futility and accept
the bolder construction paving way for an effective outcome.
 When it comes to interpreting any provision or law it is very vital to note that there are
almost always two interpretations that may arise. One which is ultra vires while the other
which is intra vires. According to this maxim, the latter interpretation shall always prevail
over the former.
 It usually begins with the presumption in favour of constitutionality and prefers the
construction which solely embarks the statute within the domain of competency of the
legislature.
o However, it is also noted that if the presumption of a Constitution fails, then in such
a case the statute cannot be rendered operative or valid accordingly. In the landmark
case of Indira Sawhney Vs. Union of India and Others, the Supreme Court had
struck down the state legislation as it was a violation of the constitution and was
ultra-vires of the legislative competency.
 While resuming the constitutionality of any problem the words of the provision should
not be given any form of unnecessary extension.
 It is a rule of Reasonable construction.
o Example, in the case of Dr. A.L. Mudaliar vs. LIC of India (1963) 33 Comp Cas.
420 (SC), it was held that the Memorandum of Association of a company must be
read fairly and its import derived from a reasonable interpretation of the language
which it employs. Further, in order to determine whether a transaction is intra vires
the objects of a company, the objects clause should be reasonably construed: neither
with rigidity nor with laxity.
o Thus, if the Court finds that giving a plain meaning to the words will not be a fair or
reasonable construction, it becomes the duty of the court to depart from the
dictionary meaning and adopt the construction which will advance the remedy and
suppress the mischief provided the Court does not have to resort to conjecture or
surmise. A reasonable construction will be adopted in accordance with the policy
and object of the statute.
 Case laws:
o The supreme court in Commissioner of Sales Tax v. Mangal Sen(AIR 1975 SC
1106), the court has to abide by the maxim, ut res magis valeat quam pereat, lesy
the intention of the legislature may go in vain or be left to evaporate in thin air.
o In Tinsukia Electronic Supply Co Ltd. V State of Assam( AIR 1990 SC 123), the
Supreme Court strongly lean against any construction which tends to reduce a
statute to futility. It is of no doubt that if a statute is absolutely vague and its
language wholly intractable and meaningless, the statute could be declared void for
vagueness. It is therefore, the duty of the court to make what it can of the statute
knowing that, the statutes are meant to be operative and that nothing short of
impossibility should allow a court to declare a statute unworkable.
o The principle of , ut res magis valeat quam pereat is variously referred to as
effect utile, the principle of effectiveness.
 Conclusion:
o The provisions of any statute must be construed in an operative as well as effective
manner based on the principle of “Ut Res Magis Valeat Quam Pereat”. There is
obviously no doubt that if a statute is absolutely vague and the language is mostly
untraceable and somewhat meaningless the statute can be declared void for the
vagueness that it carries.
o The maxim is pressed into service so as to contend that the duty of every court is to
constitute the enactment of a provision in such a way so as to implement it for
enforcing a taxing law or regulating law.
o According to this maxim, the courts strongly lean against a construction which
reduces the statute to futility and there is a presumption in favour of the
constitutionality of the statute.

Dhoom Singh Vs. Prakash Chandra Sethi

In Dhoom Singh v. Prakash Chandra Sethi[4], an election petition was filed by Mr. C against Mr.
A, who won the elections of Legislative Assembly from Ujjain North Assembly Constituency. Mr.
A raised an objection that the election petition save the annexures weren't signed by the petitioner
i.e. Mr. C, therefore the petition fails to comply with the mandate set out in S. 81(3) of the
Representation of People's Act and hence is liable to be dismissed under S. 86(1) of the same Act.
One Mr. B who later made an intervening application while the hearing of the petition claiming
that Mr. C has colluded with Mr. A and therefore to he should be allowed to be impleaded in the
proceedings. High Court dismissed his claims stating that the provision only speaks of 'withdrawal
or abatement but doesn't provide for 'intervention' by a third party. Sensing defect in the scheme of
the Statute, apex court held:

The argument that in such a situation 'the intention of the legislature that a petition should not fail
by reason of any bargain or collusion between the election petitioner and the successful candidate
would be frustrated' was repelled on the ground 'there is undoubtedly a lacuna in the Act, because it
makes provision when an election petitioner allowed to withdraw, but makes no such provision if
he just refuses to prosecute.

Hence, based on the arguments put forth and the interpretation principles followed the court held
that “the intention of the legislature that a petition should not fail by reason of any bargain or
collusion between the election petitioner and the successful candidate would be frustrated'' was
repelled on the ground. Hence dismissing the appeal.

Scope and Meaning of ut res magis valeat quam pareat:


The maxim deals with the situations as to where at times legal provisions may give rise to
complex situations involving the scope of their application. Here the general principle of
interpretation should be in pursuance to the interpretation that addresses the true intent of the
legislature. The issue in this case was primarily surrounding the scope of ‘abatement’ and
‘collusion’. Under this maxim, it is to be observed that a statute is always considered to be an
authentic repository of legislative will and it is thus the function of the court to interpret it
according to the will of the drafters. Furthermore it is also the duty of the court to avoid to the best
possible extent any construction that may attribute irrationality to the legislature. Hence the same is
observed in the present case as to how the SC did not overstep its limits and interpreted the scope
of collusion to the nest possible way under the RPA.

Fundamental Rules of Interpretation:

Literal Rule Home

 Often, judges look to see if there can be a literal meaning to the words used in the disputed
statutory provision.
 No Legal Rules exist which state which rule of Interpretation can be used and the rules of
interpretation that have been identified, are not themselves legal rules.
 Rule:
o Words must be given their natural, popular and grammatical meanings.
 words of a statute must first be given their natural, ordinary and popular
meaning, and
 sentences and phrases must be given their grammatical meaning.
 The courts should not be over zealous in searching for ambiguities or
obscurities in words which are plain. Tata Consultancy Services V. State of
A.P. (2005) 1 SCC 308
 Ramavtar Budhaiprasad v. Assistant Sales Tax Officer, AIR 1961 SC
1325: The Supreme court was faced with a question with the meaning of
"vegetable", as it had occurred in the C.P and Berar Sales Tax Act, 1947 as
amended by Act of 1948,whether the word vegetables included betel leaves
or not. The Supreme Court held that "being a word of everyday use it must
be construed in its popular sense". It was therefore held that betel leaves
were excluded from its purview.

o Exact meaning should be preferred to a loose meaning.


 Pritipal Singh V. Union of India (AIR 1982 SC 1413, P. 1419(1982)), it
was held that there is a presumption that the words are used in an Act of
Parliament correctly and exactly and not loosely and inexactly.
o Technical words must be construed in their technical sense.
 According to Lord Esher waist or skin are well known terms applied to a
ship and nobody would think of their meaning the waist or skin of a person
when they are used in a statue dealing with ships.
 Language of the Statute must be preserved:
o Courts should not add words to the statute or substitute different words for the
words used by the legislature.
o Matter which should have been included by the legislature, but has not been
included (casus omissus) should not be supplied by the Courts.
o No word should be rejected as surplus or redundant.
 Conditions of Literal Rule:
o Statute may itself provide a special meaning for a term, which is usually to be found
in the interpretation section.
o Technical words are given ordinary technical meaning if the statute has not specified
any other.
o Words will not be inserted by implication.
o Words undergo shifts in meaning in course of time.
o It should always be remembered that words acquire significance from their context.
Cases :
Fisher v. Bell (1960) → very important!!!!!!!
o The Law: Restriction of Offensive Weapons Act 1951 – convict people who offer
knives for sale . The Act intended to reduce the number of dangerous weapons
available.
o Facts of the Case: A shopkeeper displayed in his shop window flick knives with a
price ticket behind it.
o Defendant was initially charged, however on appeal he was acquitted because:
o He had not technically ‘offered’ the knives for sale, because under contract law, his
display was an invitation to treat and it was the customers who were making the
offers.
o Court assumed that parliament knows the legal technical meaning of the word
‘offer’ so the Act was rendered ineffective here.
 London and North Eastern Railway v Berriman (1946): VERY IMPORTANT
o In this case a rail worker was killed whilst oiling a track; no ‘stopping man’ had
been provided. Under statute, compensation is provided on death of workers
‘replacing or relaying’ track. The statute did not cover oiling and so compensation
wasn’t given.
 R V Harris case (1836)
o where the defendant bit the nose off the victim. The statute stated the offence was
‘to stab or wound’. Under The Literal Rule, biting is not stabbing, cutting or
wounding (implying the use of an instrument). The defendant was proven not guilty.
 Procter & Gamble v HMRC [2008]:
o Pringles are not potato crisps because they are not made wholly or exclusively from
potato, the potato content is less than 50%, they are also made from dough. Also
distinguishing them from crisps is their packaging, and "unnatural shape”. What
Pringles are made from was a question of law; which is found by combining two
issues of fact; were they made of mostly of potato, in a way other crisps are made.
o Regular Pringles are not potato crisps applying these tests.
o Following the judgment, Pringles, in all flavours are free from Value Added Tax
(VAT). Because they are manufactured from dough, “Pringles” are more like a cake
or a biscuit

 Problems of Interpreting Statutes ( Exceptions)


o Language is not a precise tool
 Twinning vs. Myers 1982:
 “It shall be a criminal offence for any vehicle to enter the park”.
 What is a vehicle? Could we expect a vehicle for a disabled person
from entering parks, or a child’s bicycle, or did roller skates amount
to a vehicle? These are all open to debate depending on our
interpretation of the word ‘vehicle’ therefore words often take their
meaning from their context, so there are shades of meaning.
 Courts have to read and understand the words used in light of the specific
statute (eg: so for vehicles, it would have to read the MV Act).
o The meaning of words changes over time.
 When the Telegraph Act 1869 was passed the telephone had not been
invented, so in subsequent cases it was necessary to extend certain
provisions of the Act to cover telephone messages.
o The drafting of the legislation might have been hurried.
 When the courts need new laws, they need to pass a new law quickly
through parliament. Nobody challenges the wording because everyone is
happy with the purpose of the legislation
 The intention of Parliament is best achieved by giving the words their ordinary natural
meaning wherever such words are capable of a literal meaning.
 R v Bloxham:
o Bloxham paid £500 for a car, and promised a further £800 once the log book was
delivered. The log book was not delivered and Bloxham realised the car must have
been stolen. Eventually he sold the car for £200.
o Is Bloxham liable for theft?
o S22 Theft Act 1968(1) - “A person handles stolen goods if he dishonestly
undertakes or assists in their disposal by or for the benefit of another person”.
o Answer → X is another person who benefited from buying a cheap car, therefore
Bloxham is guilty.
 The words of a statute are first understood in their natural, ordinary or popular sense and
phrases and sentences are construed according to their grammatical meaning unless that
leads to some absurdity. →
o Reiterating the same objects Das Gupta J. in Mahadeolal Kanodia v Administrator
General of West Bengal AIR 1960 SC 936 observed that “the intention of the
Legislature has always to be gathered by words used by it, giving to the words their
plain, normal, grammatical meaning.”
 In M/s. Hiralal Ratanlal v. STO AIR 1973 SC 1034, the Court observed that
o In interpreting a statutory provision the first and foremost rule of interpretation is
the literal construction. All that the Court has to see at the very outset is what does
the provision say. If the provision is unambiguous and if from the provision the
legislative intent is clear, the Court need not call into aid the other rules of
construction of statutes. The other rules of construction are called into aid only when
the legislative intent is not clear.
 Advantages of Literal Rule:
o No scope for the judges own opinions or prejudices to interfere.
o Respects parliamentary supremacy and upholds separation of power.
o Encourages drafting precision, promotes certainty and reduces litigation.
 Disadvantages of Literal Rule:
o The Literal Rule can create loopholes in law, as shown in the Fisher v Bell (1960)
case and the R v Harris (1960).
o The Literal Rule can also lead to injustice.
o In the London and North Eastern Railway v Berriman (1946) This can undermine
public confidence in the law.
o Clearly, the literal approach has another disadvantage in that one judge’s literal
interpretation might be very different from another’s. Casey says: “What may seem
plain to one judge may seem perverse and unreal to another.”
o Sometimes the use of the literal rule may defeat the intention of Parliament.
o For instance, in the case of Whiteley vs Chappel (1868; LR 4 QB 147), the court
came to the reluctant conclusion that Whiteley could not be convicted of
impersonating "any person entitled to vote" at an election, because the person he
impersonated was dead. Using a literal construction of the relevant statutory
provision, the deceased was not "a person entitled to vote." This, surely, could not
have been the intention of Parliament. However, the literal rule does not take into
account the consequences of a literal interpretation, only whether words have a clear
meaning that makes sense within that context. If Parliament does not like the literal
interpretation, then it must amend the legislation
 Other rules of statutory interpretation include, but are not limited to:
o Statutes should be internally consistent. A particular section of the statute should not
be inconsistent with the rest of the statute.
o When the legislature enumerates an exception to a rule, one can infer that there are
no other exceptions.
o When the legislature includes limiting language in an earlier version of a statute, but
deletes it prior to enactment of the statute, it can be presumed that the limitation was
not intended by the legislature.
o The legislature is presumed to act intentionally and purposely when it includes
language in one section but omits it in another.
o Where legislation and case law conflict, courts generally presume that legislation
takes precedence over case law.
o The Rule of Lenity: in construing an ambiguous criminal statute, a court should
resolve the ambiguity in favor of the defendant.
o A court may also look at: the common usage of a word, case law, dictionaries,
parallel reasoning, punctuation
o Statutes are sometimes ambiguous enough to support more than one interpretation.
In these cases, courts are free to interpret statutes themselves. Once a court
interprets the statute, other courts usually will not go through the exercise again, but
rather will enforce the statute as interpreted by the other court.

Mischief Rule Home


The Mischief rule emanates from the judgement of Lord Coke in the landmark Heydon’s case. This
rule of interpretation emphasises on a purpose construction which can be attributed to the fact that
the purpose of a statute is the most important consideration while being applied to a case. In
essence, this rule of interpretation focuses on ‘curing’ the mischief done by the law.

The rule factors in the following questions that are to be addressed by the judge:
a. What was the common law before the making of an act.
b. What was the mischief for which the present statute was enacted.
c. What remedy did the Parliament sought or had resolved and appointed to cure the disease of
the commonwealth.
d. The true reason of the remedy.
The application of this rule has certain drawbacks, the major being that the judges get more
discretion to ascertain the intent of the legislature while making the law, thus undermining the
supremacy of the legislature in the law-making power.The rule has nevertheless been used to
resolve ambiguities in cases where the literal or golden rule cannot be duly and effectively be
applied. The mischief rule has been seen as a modern rule of statutory interpretation rather than a
stand-alone tool. The following are a few notable cases:

1. RMDC v. UOI - In RMDC v Union of India the definition of ‘prize competition’ under s
2(d) of the Prize competition act 1955, was held to be inclusive of only those instances in
which no substantive skill is involved. Thus, those prize competitions in which some skill
was required were exempt from the definition of ‘prize competition’ under s 2(d) of the
Act. Hence, in the aforementioned case, the Supreme Court has applied the Heydon’s Rule
in order to suppress the mischief was intended to be remedied, as against the literal rule
which could have covered prize competitions where no substantial degree of skill was
required for success.
2. DPP v Bull - A man was charged with an offense under s.1(1) of the Street Offences Act
1959 which makes it an offense for a ‘common prostitute to loiter or solicit in a public street or
public place for the purposes of prostitution’. The magistrates found him not guilty on the grounds
that ‘common prostitute’ only related to females and not males. The prosecution appealed by way
of case stated. The court held that the Act did only apply to females. The word prostitute was
ambiguous and they applied the mischief rule. The Street Offences Act was introduced as a result
of the work of the Wolfenden Report into homosexuality and prostitution. The Report only referred
to female prostitution and did not mention male prostitutes. The QBD, therefore, held the mischief
the Act was aimed at was controlling the behavior of only female prostitutes.
3. Smith v. Hughes - The brief facts were that the defendant was a common prostitute who
lived at London and used the premises for the purposes of prostitution. On November 4, 1959,
between 8.50 p.m. and 9.05 p.m. the defendant solicited men passing in the street, for the purposes
of prostitution, from a first-floor balcony. The prostitutes were convicted. Onappat Held dumining
the appeals, But on the true construction of section 1 (1), taking into consideration the mischief at
which the Act of 1999 was aimed at mattered not where a prostitute stood (bether on a balcony, or
a room behind a chmod, or spen, or half-open window), if her solicitation was projected to and
addressed to somebody walking in the street, she was guilty of an lence against section 1 (1).

R.M.D. Chamarbaugwala v. Union of India [AIR 1957 SC 628] Elaborate.

The Parliament enacted the Prize Competitions Act, 1955. The petitioner contended that the
provisions laid down in the Act, and Rules made thereunder, encroached upon the fundamental
rights to carry on business guaranteed by Article 19(1)(g) and that it amounted to prohibition and
not merely a regulation. Moreover, S.2(d) that defined Prize Competition not only included
Competitions in which success rate was depended upon chance but also those in which success rate
was depended upon substantial degree of skill and, therefore, the impugned law constituted a
single, inseverable enactment, it must fail in its entirety.

The rule of interpretation used in this case is the mischief rule. An illustration of the application of
the rule is also furnished in the construction of section 2(d) of the Prize Competitions Act, 1955.
This section defines "Prize Competition" as meaning "any competition in which prizes are offered
for the solution of any puzzle based upon the building up arrangement, combination or permutation
of letters, words or figures". The question was whether in view of this definition, the Act applies to
competitions which involve substantial skill and are not in the nature of gambling. The Supreme
Court, after referring to the previous state of the law, to the mischief that continued under that law
and to the resolutions of various States under Article 252(1) authorising Parliament to pass the Act
stated, "Having regard to the history of the legislation, the declared object thereof and the wording
of the statute, we are of opinion that the competitions which are sought to be controlled and
regulated by the Act are only those competitions in which success does not depend on any
substantial degree of skill".

The Supreme Court held that S. 2(d) should be construed keeping in mind the historical
background of the Act and the mischief it intends to suppress. Keeping these factors in mind, it is
clear that the legislation was enacted to regulate and control prize competitions of gambling nature.
The use of word control would not have been appropriate if it was intended towards competition of
skill. Therefore, the competitions which were intended to be controlled and regulated under the Act
were only those in which success does not depend upon any substantial degree of skill.

Golden Rule Home


 This rule is a departure from the literal rule of interpretation and asserts that the
literal rule may be modified
 “Golden Rule” is the modified form of interpretation. According to the rule if the
grammatical or literal interpretation leads to some absurdity or some repugnancy or
inconsistency with the rest of the provisions, that sense may be modified so as to avoid that
absurdity or inconsistency; but not further.
 The golden rule requires that the literal rule should be applied to the statute in the first
instance, but that if the literal rule results in an ambiguity or absurdity the court should try
to interpret it in another manner so as to avoid the ambiguity or absurdity.
 A rule of statutory or legal document interpretation which allows a shift from the ordinary
sense of as word(s) if the overall content of the document demands it.
 Thus, Golden rule
o avoids obvious foolishness
o only applied when literal rule leads to absurdity
o where an absurdity arises from the literal interpretation,
o give a reasonable meaning the words being construed
 Grammar is a good guide to meaning but a bad master to dictate.
o The rule was evolved by Parke B (who later became Lord Wensleydale) in Becke v
Smith (1836) and Grey v Pearson, (1857), who stated, "The grammatical and
ordinary sense of the words is to be adhered to unless that would lead to some
absurdity or some repugnance or inconsistency with the rest of the instrument in
which case the grammatical and ordinary sense of the words may be modified so as
to avoid the absurdity and inconsistency, but no farther. This was also followed in
the cases of State of Kerala vs. West Coast Planters (and) Sirsilk Ltd. vs. Govt of
AP.
 Example → CLRA Act: Judicial Interpretation --
o Main Issue regarding the act is that → Whether the Act itself provides the workers
of an industry, the right to get absorbed by the industry after contract labour is
abolished there?
o Following are the observations of the court:
 No specific legislation which dealt in detail with the problem of the contract
labour
 No law to regulate contract labour.
 requirement of an Act which completely dealt with the regulations of the
contract labour.
 After the enactment of the Act, which took place on the 5th September, 1970
but came in force on the 10th February, 1971, the courts did not have to face
much difficulties as regarding
 the facilities which should be provided to these contract labour (S.
16,17,18 and 19).
 The definition of employer,
 contractor (S.2(c))
 and workmen (S.2(b)) were also provided by this Act which helped
the court to interpret the meaning of these words.
 The courts also construed as to when the laborers would be considered as
contract laborers or not.
 Adler v George [1964] QBD
o Adler gained access to a RAF station (a prohibited place within the meaning of the
Official Secrets Act 1920) and was actually within its boundaries. He obstructed a
member of Her Majesty's forces engaged in security duty in relation to the station
‘in the vicinity of a prohibited place’ He argued that, as he was actually in the
prohibited place, he could not be said to be "in the vicinity" of the prohibited place.
o Held: The defendant was guilty of the offence because "in the vicinity of" should be
interpreted to mean on or near the prohibited place
 R v Allen (1872) LR 1 CCR 367
o The defendant was charged with the offence of bigamy under s.57 of the Offences
Against the Person Act 1861. The statute states 'whosoever being married shall
marry any other person during the lifetime of the former husband or wife is guilty of
an offence'. Under a literal interpretation of this section the offence would be
impossible to commit since civil law will not recognize a second marriage any
attempt to marry in such circumstances would not be recognized as a valid marriage
o Held: The court applied the golden rule and held that the word 'marry' should
be interpreted as 'to go through a marriage ceremony'. The defendant's
conviction was upheld.

 Narrow and Wide Approach:


o Under the Narrow approach the court “can only choose between the possible
meanings of a word...” i.e. if one meaning is apparent that one must be adopted
o Under the wider approach the courts are granted the right to “modify the words in
order to avoid a problem”, this situation arises where there is an obvious and clear
meaning but this meaning would lead to an absurd result.
o Re Sigsworth (1935).
 The judge would not allow for a murderer to benefit from his crime thus
applied the golden rule and held the next of kin would not inherit the estate
“where they had killed the deceased”.
 Problem with Golden Rule:
o Judges are able to add or change the meaning of statutes and thereby become law
makers infringing the separation of powers.
o Judges have no power to intervene for pure injustice where there is no absurdity
 Advantages of Golden Rule
o Errors in drafting can be corrected immediately eg: R v Allen (1872).
o Decisions are generally more in line with Parliament's intention
o Closes loopholes
o Often gives a more just result
o Brings common sense to the law

Harmonious Construction

When there is a conflict between two or more statues or two or more parts of a statute then the rule
of harmonious construction needs to be adopted. The rule follows a very simple premise that every
statute has a purpose and intent as per law and should be read as a whole. The interpretation
consistent of all the provisions of the statute should be adopted. In the case in which it shall be
impossible to harmonize both the provisions, the court’s decision regarding the provision shall
prevail.

The rule of harmonious construction is the thumb rule to interpretation of any statute. An
interpretation which makes the enactment a consistent whole, should be the aim of the Courts and a
construction which avoids inconsistency or repugnancy between the various sections or parts of the
statute should be adopted. The Courts should avoid “a head on clash”, in the words of the Apex
Court, between the different parts of an enactment and conflict between the various provisions
should be sought to be harmonized.

Harmonious Construction should be applied to statutory rules and courts should avoid absurd or
unintended results. It should be resorted to making the provision meaningful in the context. It
should be in consonance with the intention of Rule makers. Rule of Harmonious construction is
applicable to subordinate legislature also.

The objective of harmonious construction is to avoid any confrontation between two enacting
provisions of a statute and to construe the provisions in such a way so that the harmonize. The basis
of this rule is that the Legislature never envisages to provide two conflicting provisions in a statute,
for the reason that it amounts to self-contradiction.

The intention of legislature is that every provision should remain operative. But where two
provisions are contradictory, it may not possible to effectuate both of them and in result, one shall
be reduced to futility as against the settled basic principle of ut res magis valeat qauam pereat.
Therefore, such a construction should be allowed to prevail by which existing inconsistency is
removed and both the provisions remain in force, in harmony with each other.

Cases:
1. K M Nanavati v. State of Bombay (1960)
The matter was related to the power conferred to the Governor under Art 161 and the court has to
interpret the scope of Art 161 and Art 142 (1) of the constitution. In the present case, Bombay HC
passed sentence against the accused. The petitioner then approached the Governor who passed a
suspension order against the sentence of Bombay HC.
When the matter reached before Hon’ble Supreme Court, the court applied the rule of Harmonious
Construction and held that the absolute power of granting suspension to the Governor under Art
161 becomes absolved when the matter becomes sub judice. In such situations, there is a complete
scope of interference with the judicial power of the court under Art.142.

2. Sri Venkataram Devaru & Ors v. State of Mysore & Ors (1957)
In this case the Supreme Court applied the rule of harmonious construction in resolving a conflict
between Articles 25(2)(b) and 26(b) of the Constitution and it was held that the right of every
religious denomination or any section thereof to manage its own affairs in matters of religion
[Article 26(b)] is subject to a law made by a State providing for social welfare and reform or
throwing open of Hindu religious institutions of a public character to all classes and sections of
Hindus [Article 25(2)(b)].

3. Pandit MSM Sharma v. Shri Sri Krishna Sinha and Ors. (1959)
The Supreme Court applied the rule of harmonious construction and held that though Art. 194 (3)
is subordinate to Art. 21 but Indian Constitution is the supreme law in the country and therefore, a
person can be expunged from publishing the official records of the Assembly. This is not a
complete prohibition on FR of that person.

4. The Sirsilk Ltd & Ors. v Govt. of AP & Ors (1963)


An interesting question relating to a conflict between two equally mandatory provisions, viz., ss
17(1) and 18(1) of the Industrial Disputes Act, 1947, is a good illustration of the importance of the
principle that every effort should be made to give effect to all the provisions of an act by
harmonizing any apparent conflict between two or more of its provisions. Section 17(1) of the Act
requires the government to publish every award of a Labour Tribunal within thirty days of its
receipt and by sub – section (2) of section 17 the award on its publication becomes final. Section
18(1) of the Act provides that a settlement between employer and workmen shall be binding on the
parties to the agreement. In a case where a settlement was arrived at after the receipt of the award
of a Labour Tribunal by the Government but before its publication, the question was whether the
Government was still required u/s 17(1) to publish the award. In construing these two equally
mandatory provisions, the Supreme Court held that the only way to resolve the conflict was to hold
that by the settlement, which becomes effective from the date of signing, the industrial dispute
comes to an end and the award becomes infructuous and the Government cannot publish it.
Subsidiary Rules of Interpretation:

Expressio unius est exclusio alterius

 Express mention of one thing implies the exclusion of another.


 Express words in the statute shut the door to further implication.
 “Notwithstanding anything…” - bars the possibility of inclusion of any other condition
 If one or more things of a particular class are expressly mentioned in an enactment, it means
that the other things belonging to the same class are excluded from the domain of the
enactment.
 Maxwell - Where two expressions have been used in a statute one of which generally
includes the other, the more general expression excludes the less general.
 Lopes L.J. observed that it is often a valuable servant but a dangerous master to follow in
the construction of statutes
 Applicability: This maxim will be applied where specific words are being mentioned in
any statute or enactment. This maxim will not be applied where general words are
mentioned along with specific words.
 Applicable: where specific words are mentioned in any statute or enactment.
 Where it does not apply: where general words are mentioned along with specific words
 Words are added with consciousness. If words are excluded then the legislators did not
want to have added the same
 R vs. Inhabitants of Sedgely:
o A statute raised taxes on 'lands, houses and coalmines'.
o The defendant owned a limestone mine.
o The court held that it did not apply to limestone mines as these were not specifically
mentioned nor did the statute suggest that it would apply to other type of mines.
 R vs. Secretary of State for the Home Department:
o Held by the court to exclude the father of an illegitimate child from rights under
immigration law at the time, because the definition section specifically mentioned
the mother alone
 Hakam Singh vs. Gammon India Ltd.
o Held: It is not open to the parties by agreement to confer jurisdiction on a court,
which it does not possess under the Civil Procedure Code. But where two court or
more have jurisdiction to try a suit, an agreement between the parties that the
dispute between them shall be tried in one of such courts is not contrary to public
policy. Such an agreement does not contravene S. 28 of the Contract Act.
 M/s Swastik Gases Pvt. Ltd. V. Indian Oil Corp. Ltd
o The SC provided clarity by stating that "the absence of words like 'alone', or
'exclusive jurisdiction' is neither decisive nor does it make any material difference in
deciding the jurisdiction of a court.
o The principle of expressio unius est exclusio alterius would be applicable to such
cases.
o Where the contract specifies the jurisdiction of courts at a particular place and such
courts have jurisdiction to deal with the matter, we think that an inference may be
drawn that parties intended to exclude all other courts.

Ejusdem Generis Home

 Latin term for "of the same kind".


 In an enumeration of different subjects in an Act, general words following specific words
may be construed with reference to the antecedent matters, and the construction may be
narrowed down by treating them as applying to things of the same kind as those previously
mentioned, unless of course, there is something to show that a wider sense was intended. If
the particular words exhaust the whole genus, then the general words are construed as
embracing a larger genus.
 General words follow specific words in a statutory enumeration, the general words are
construed to embrace only objects similar in nature to those objects enumerated by the
preceding specific words.
 When a general word follows specific words of a distinct category, the general word may be
given a restricted meaning of the same category.
 The general expression takes its meaning from the preceding particular expression because
the legislature by using the particular words of a distinct genus has shown its intention to
that effect.
 When it is used:
o The rule is applied to resolve the problem in giving meaning to groups of words
where one of the words is ambiguous or inherently unclear or vague.
The conditions to apply them are given in the UP electricity board case.

 When does it not apply


o If the preceding term is general, the rule of ejusdem generis will not apply.
o If the specific words do not belong to a distinct genus, this rule is inapplicable.
o If a general word follows only one particular word, that single particular word does
not constitute a distinct genus and therefore ejusdem generis rule cannot be applied.
 Uttar Pradesh State Electricity Board v. Harishankar
In this the court held that conditions under which EG will be applicable. The conditions are
:-
o The statute contains an enumeration of specific words.
o The subjects of enumeration constitute a class or category.
o That class or category is not exhausted by the enumeration.
o The general terms follow the enumeration.
o There is no indication of a different legislative intent.
 Nature Lovers Movement v. State of Kerala
o K. S. Radhakrishan J. held "it is essential for the application of ejusdem generis rule
that the enumerated things before the general word must constitute a category or a
genus or a family which admits a number of species or members. If the specified
things preceding general words belong to different categories, this principle of
construction will not apply".
 Powell v. Kemptom Park Racecourse
o The defendant had been operating an outside betting place.
o The Act stated "house, office room or other place of betting."
o The court had to define if 'other place' would cover the defendant's situation.
o As the terms in the list all referred to indoor places, it was decided that 'other place'
also referred to an indoor place.
 In Ujjain Bai v. State of U.P. the Supreme Court observed that Article 12 winds up the list
of authorities falling within the definition by referring to “other authorities” within the
territory of India which cannot, obviously, be read as ejusdem generis with either the
Government or the Legislature or Local authorities.
 Allen v. Emmerson
o Examples of particular words followed by general words were given;
o In the expression, "books, pamphlets, newspapers and other documents" private
letters may not be held included if 'other document' be interpreted ejusdem generis
with what goes before.
o But in a provision which reads "newspaper or other documents likely to convey
secrets to the enemy", the words 'other documents' would include documents of any
kind and would not take their meaning from newspaper.
Casus Omissus
 Term casus omissus means “cases of omission”.
 Omission in a statute cannot be supplied by construction.
 A matter which should have been provided but actually has not been provided in a statute
cannot be supplied by the courts, as to do so will be legislation and not construction. →
Hansraj Gupta v. DMET Co. Ltd. AIR 1933 P.C. 63
 Court can interpret the law but cannot legislate. → Padma Sundara Rao v. State of T.N.
 A casus omissus cannot be supplied by the court by judicial interpretive process except in
the case of clear necessity and when reason for it is found in the four corners of the statute
itself.
 The language employed in the statute is the determinative factor of the legislative intent.
The first and primary rule of the construction is that the intention of the legislature must be
found in the words used by the legislature itself. The question is not what may be supposed
and has been intended but what has been said. The court only interprets the law and cannot
legislate it.
 It is a rule of statutory construction that Casus Omissus which means that case omitted from
the language of a statute but within the general scope of the statute and which appears to
have been omitted due to inadvertence or by overlook cannot be supplied by the court.
 S.P. Gupta v. President of India AIR 1982 SC 149
o the Supreme Court held that when the language of a statute is clear and
unambiguous there is no room for application of the doctrine of Casus Omissus or of
pressing into service external aid in such a case, the words used by the statute speak
for themselves and it is not the function of the court to add words or expression
merely to suit what court thinks is the supposed intention of the legislature
 Raghunath Rai Bareja v. Punjab National Bank (2007) 2 SCC 230
o The Apex Court observed that even if there is defect or omission in the words used
by the Legislature, the Court cannot correct or make up the deficiencies especially
when literal reading thereof produces an intelligible result
 Ramesh Mehta v. Sanwal Chand Siinghvi AIR 2004 SC 2258.
o The Supreme Court observed that although court cannot supply to casus omissus, it
is equally clear that it should not interpret a statute so as to create a casus omissus
when there is really none.
 When can omitted words be supplied?
o When the lacuna in the language is of such a nature that unless the omitted word is
supplied the statute cannot operate or the true intention of the legislature could not
be established, the courts have inserted the missing word in the language of a statute
to ensure that law is not turned to nullity.
o Jacob Mathew v State of Punjab (2005) 6 SCC 1,
 Section 304-A of the IPC was construed by the Supreme Court and Casus
Omissus was supplied. It was held that the word “gross” has not been used in
Section 304-A, yet it is settled that in criminal law negligence and
recklessness to be so held must be of such a high degree as to be “gross”.
The Expression “rash or negligent act” as occurring in Section 304-A has to
be read qualified by the word “gross”.

Reddendo Singula Singulis

 Meaning:
o Reddendo singula singulis is a Latin term that means by referring each to each;
referring each phrase or expression to its corresponding object.
o It is one of the best settled rules of construction that words in different parts of the
statute must be referred to their appropriate connection, giving to each in its place
its proper force, and if possible rendering none of them useless or superfluous.
 Introduction:
o “I devise and bequeath all my real and personal property to A”, which will be
construed reddendo singular singulis by applying “devise” to real property and
“bequeath” to personal property.
o The best example of the rule reddendo singular singulis was quoted from
Wharton’s Law Lexicon thus:
 “ If any one shall draw or load any sword or gun, the word draw is applied to
‘sword’ only and the word load to ‘gun’ only, the former verb to former
noun, and the latter because it is impossible to load a sword or to draw a gun.
 Where a complex sentence has more than one subject, and more than one object, it may be
the right construction to render each to each, by reading the provision distributively and
applying each object to its appropriate subject.
 In K.V. Kamnath v. K.Rangappa Baliga and Company, AIR 1969 SC 504,
o the Supreme Court stated that where a sentence contains several antecedents and
several consequents they are to be read distributively. That is, the words are to be
applied to the subjects to which they appear by context most properly to relate and
to which they are most applicable.
 Raja Katra vs Corporation of Calcutta, 1973 CriLJ 1505on 16 February, 1973
o It has been observed in this case that:
o The object or purpose of all construction or interpretation is to ascertain the
intention of the law-makers and to make it effective ... if the courts were permitted
to ignore the expressed intent of the legislature, they would invade the province of
the legislature and violate the tripartite theory of Government.
o A reference was also made to the principle known as "Reddendo Singula Singulis“
o It is also well established as a principle of Statute construction that words in
different parts of a statute must be referred to their appropriate connection, giving to
each in its place, its proper force and effect, and if possible rendering none of them
useless or superfluous even if strict grammatical construction demands otherwise.

NOSCITUR A SOCIIS Home


 According to Lord Mc Millan, "Noscitur a sociis" means 'a word may be known by the
company it keeps.'
 Know from the association A word will be interpreted in the context of surrounding
words. Meaning of a word is to be judged by the company it keeps.
 A word or phrase can be read on its own as it stands. However, the maxim proposes another
possible meaning. Words and phrases next to and near the word or phrase in question might
indicate a meaning that is not apparent when the word or phrase is viewed on its own.
 In the obvious case the neighbouring words create an alternative meaning by suggesting
that words in question should receive a restricted scope.
 Maxwell: When two or more words susceptible of analogous meaning are coupled together,
they are understood to be used in their cognate sense.
 The words take their color from and are quantified by each other.
 Meaning of the general words restricted to a sense analogous to that of the less general.
 V. R. Krishna Iyer J. expressed the necessity of the maxim in the following words:
 "If birds of a feather flock together, noscitur a sociis is a common sense guide to
construction."
 When can it be used:
o When the intention of the legislator is not clear.
o When the associated words have analogous meaning.
 Rainbow Steels Ltd. And Anr. V. Commissioner of Sales Tax, Uttar Pradesh
o The interpretation of the word 'old' as used in "old, discarded, unserviceable or
obsolete machinery, stores or vehicle etc. was considered.
o It was held that the four adjectives – old, discarded, unserviceable, obsolete, which
are susceptible to analogous meaning are clubbed together while qualifying
machinery. The first adjective is more general than the other three and the meaning
of the more general adjective 'old' is being restricted to a sense analogous to that of
less general namely 'discarded, unserviceable or obsolete.'
o The expression 'old' was therefore construed to refer to machinery that had become
non-functional or non-usable.

 Pradeep Agarbatti, Ludhiana v. State of Punjab


o Schedule 'A' Entry 16 of Punjab Sales Tax Act reads: "cosmetics, perfumery and
toilet goods excluding toothpaste, tooth powder, kumkum and soap."
o Whether dhoop and agarbatti will fall under 'perfumery' as mentioned above?
o With reference to the Punjab Sales Tax Act, it was held that the word 'perfumery'
means such articles as used in cosmetics and toilet goods viz, sprays etc. But does
not include dhoop and agarbatti.
 Foster v. Diphwys Casson
o A statute stated that explosives taken into a mine must be in a "case or canister."
o The defendant used a cloth bag.
o Court held that the bag could not have been within the statutory definition because
the Parliament's intention in using 'case or cannister' was referring to something of
the same strength as a canister.

EJUSDEM GENERIS NOSCITUR A SOCIIS

 'of the same kind'  'know from the association'


 Arises if a catchall phrase ends a list.  Arises if questionable meaning of a
 Used for interpreting loosely written doubtful word.
statutes.  Used for interpreting questionable
 If the statute lists specific classes of the words in statutes.
things and then refers to them in  The meaning of a doubtful word can
general. be known from the accompanying.
 A general term after a list of particular  The surrounding usage of words can
terms is only about those things similar reveal the meaning of the
to the particular terms. questionable words or phrases.
CONSTITUTIONAL INTERPRETATION PRINCIPLES

1. Doctrine of Severability: RMDC vs. Union of India

Law:Mysore Lotteries and Prize Competitions Control and Tax Act, 1951.

Issue: Whether the said legislation was ultra vires the Constitution under Article 19(6)?

Arguments: Firstly, the business carried out did not amount to trade and hence was not hit by
Art.19(6). With a certain part of the Act being held invalid, the contention by the petitioners was
that since the parts of the Act were connected, the entire Act was supposed to be held ultra vires.
The Court, discussed the doctrine of severability as follows: It is not the whole Act which would be
held invalid by being inconsistent with Part III of the Constitution but only such provisions of it
which are violative of the fundamental rights, provided that the part which violates the fundamental
rights is separable from that which does not isolate them. But if the valid portion is so closely
mixed up with invalid portion that it cannot be separated without leaving an incomplete or more or
less mingled remainder the court will declare the entire Act void. This process is known as doctrine
of severability. Only the part of the act that was ultra vires was seen to be ‘severed’ from the Act.

2. Doctrine of Eclipse: Bikaji Narain v. Stateof Madhya Pradesh

The law concerned is C.P. and Berar Motor Vehicles Amendment Act, 1947 - authorized the State
Government to make up the entire motor transport business in the province to the exclusion of
motor transport operators. This provision, though valid when enacted, became void on the
commencement of the Constitution in 1950 as they violated Article 19 (1) (g) of the Constitution.
However, 1951, clause (6) of Article 19 was amended by the first Constitution Amendment Act, as
so to authorize the Government to monopolies any business. The Supreme Court held that "the
effect of the amendment was to remove the shadow and to make the impugned Act free from all
blemish or infirmity.It became enforceable against citizens as well as non-citizens after the
constitutional impediment was removed. This law was merely eclipsed for the time being by the
fundamental rights. As soon as the eclipse is removed the law begins to operate from the date of
such removal.

3. Doctrine of Basic Structure : Keshavananda Bharativs. Stateof Kerala

The Supreme Court for the first time recognised the doctrine of basic structure. The doctrine
primarily holds constituent powers over ordinary legislative powers. The powers and functions of
the Indian Parliament and State legislatures are subject to limitations laid down in the Constitution.
The Constitution does not contain all the laws that govern the country. Parliament and the State
Legislatures make laws from time to time on various subjects, within their respective jurisdictions.
The general framework for making these laws is provided by the Constitution. Parliament alone is
given the power to make changes to this framework under Article 368. Unlike ordinary laws,
amendments to constitutional provisions require a special majority vote in Parliament. The court
held Parliament could not use its amending powers under Article 368 to 'damage', 'emasculate',
'destroy', 'abrogate', 'change' or 'alter' the 'basic structure' or framework of the Constitution.

The same was upheld in Indira Gandhi v. Raj Narain, where a constitutional amendment that
removed the powers of review of the judiciary in matters of elections were held to be violative of
the basic structure doctrine.

4. Doctrine of Harmonious Construction : K MNanavati vs. State of Maharashtra

The case concerns the life imprisonment granted to the accused. Relevant provisions of law are:
Article 142 which deals with the special powers of the Supreme Court with regards any matter
pending before it and the other Article 161, the general power of any governor at any time to
suspend the sentence against the accused. The present case involved the governor commuting the
sentence against K M Nanavati. The Court, using the principle of harmonious construction held
that Article 161 does not deal with the suspension of sentence during the period when the matter is
sub-judice in this Court.

5. Doctrine of Pith and Substance : State of Bombay vs. FN Balsara

The Bombay Prohibition Act was challenged on the ground that it accidentally encroaches upon
import and export of liquor across custom frontier, which was a central subject. The Court while
upholding the impugned legislation declared that the Act was in pith and substance a State subject
even though it incidentally encroached upon a central subject. The doctrine is applied when there is
a conflict with the central and state legislation, the ‘pith and substance ‘of that legislation is to be
applied to go into what the true object of the legislation meant.

6. Doctrine of Prospective Overruling : Lily Thomas vs. UOI

The SC ruled that any MLA, MLC or MP who was found guilty of a crime and given a minimum
of 2 years imprisonment would cease to be a member of the House with immediate effect. The
Doctrine of Prospective Overruling dictates that a decision made in a particular case would have
operation only in the future and will not carry any retrospective effect on any past decisions. The
present case saw the doctrine being applied to all prospective disqualifications
7. Doctrine of Repugnancy : M Karunanidhi vs. UOI

A repugnancy is defined as a repugnancy as an inconsistency or contradiction between two or more


parts of a legal instrument (such as a statute or a contract). Such an inconstancy may appear
between two pieces of legislation where it would be impossible to bring effect to one without
disobeying the other. In the present case, the Supreme Court held that, where the provisions of a
Central Act and a State Act in the Concurrent list are fully inconsistent and absolutely
irreconcilable, the Central Act will prevail and the State Act will become void in view of the
repugnancy.

8. Doctrine of Colourable Legislation : D C Wadhwa vs. Stateof Bihar

According to the Doctrine of Colourable legislation, ‘a constitutional authority cannot do indirectly


what it is not permitted to do directly.’ A similar situation was observed in the instant case.
Ordinances were promulgated beyond the constitutional mandate. The executive had the
unreasonable power to promulgate an ordinance and then re-promulgate the same without
constitutional considerations. The same was seen as Colourable exercise of power.

9. Doctrine of Constitutional Silence- Bhanumati vs. State of UP

Constitutional silences are seen as certain aspects of the constitution that are not expressly written
down, but are upheld as ‘silent’ principles. The non- existence of a right to secession can be one
such hypothetical example. The issue in this case concerned the validity of the U.P (Panchayat
Laws) Amendment Act, 2007. It was contended before the Supreme Court that the provision of a
"no confidence motion" in the legislation was inconsistent with Part IX of the Constitution as the
latter did not provide for any such ground for removal of a Chairperson of a Panchayat. Rejecting
this contention, the Court observed that many issues in our constitutional jurisprudence evolved out
of this doctrine of silence and that in the present situation, it cannot be interpreted to exclude the
provision of no-confidence motion in respect of the office of the Chairperson of the panchayat just
because of its silence on that aspect.

10. Doctrine of Territorial Nexus- Shrikant Bhalchandra Karulkar v. State of Gujarat

In order to give effect to the laws made by a state for extraterritorial purpose, a nexus between the
object and state must be shown. The State legislature has the jurisdiction to make laws within its
territorial jurisdiction. Territorial nexus is one such exception which allows the state to make laws
for extraterritorial operations if it shows that there exists a nexus between the object and the state.
The Court held that the state legislature is conferred with the power to enact legislation for extra-
territorial operations complying with the provisions enshrined under Article 245 and 246.The laws
made by the state legislature is applicable to a person and his acts within the territorial limits of a
state is not considered as extra territorial.

UNIT 2: INTERNAL AIDS TO INTERPRETATION

INTRODUCTION

 Parts of an Enactment:
(i) Operative- Sections Schedules, Provisos, Saving Clause, etc
(ii) Amendable Descriptive - Long title, Preamble, Illustrations
(iii) Unamendable Descriptive - Date of commencement, chapter numbers, heading of a
section, format, punctuations. (Punctuations are presumed to be intentionally drafted that
way. It can be construed otherwise only for the purposes of the judgement and cannot be
read otherwise within the entire scheme of that legislation).

 When can aids of construction be ‘legitimately’ invoked?


o Words having more than one meaning
o Uncertainty
o No clarity as to which meaning to be assigned with reference to assembly of words.
o Language of the provision
o More than one distinct constructions of the language are possible.
o When result of literal interpretation is absurd
o Results after literal interpretation is against the policy or object of the Act.
o Natural and ordinary meaning fails to effectuate the legislative intent
o Obstructs advancement of the object of the statute.
 Intention of Legislature - State of Maharashtra v Marwanhee F Desai (2002) SC
o The Supreme Court held that true intent of the legislature shall have to be gathered
and deciphered in its proper spirit having due regard to the language used therein.
 Intrinsic aid to interpretation
o Various parts of the same statute/act pressed into service by the courts for
construing anyone of its provisions.
o whenever difficulty arises as to meaning of a statutory provision due to ambiguity of
words and the true intention of Legislature cannot be inferred from language in
such an event it is necessary to read the statute as a whole in its context following
the principle ex visceribus actus (within the four corners of the Act) and every
part of the statute may be called in aid.
 Ram Narain v State of UP 1957 SC -- It is no sound principle of construction to interpret
expressions used in one Act with reference to their use in another Act. The meaning of the
words and expressions use in an Act must take their colour from the context in which
they appear. (ex visceribus actus)
 Jagir v State of Bihar 1976 SC -- ‘The general rule of construction is not to look at the
words but to look at the context’, said the Supreme Court while interpreting the word
‘owner’ in a transport legislation in Bihar.

TITLES Home

 Ashwini Kumar Ghose v Arabinda Bose 1952 SC -- well settled law that title of a statute is
an important part of the Act and may be referred to for the purposes of ascertaining its
general scope and of throwing light on its construction.
 The title of a statute gives a fairly good idea as to what subject matter the statute deals
with or what is contained in the enactment.
 Amarendra Kumar Mohapatra & Ors. vs. State of Orissa & Ors.
o “The title of a statute is no doubt an important part of an enactment and can be
referred to for determining the general scope of the legislation. But the true nature of
any such enactment has always to be determined not on the basis of the given to it
but on the basis of its substance.”

Long Titles

 Long title describes the enactment and does not merely identify it.
 Long title is a part of the Act and can be referred to ascertain the object, scope and purpose
of the Act.
 Long title cannot override the plain and clear meaning from the enactment.
 R v Secretary of State for Foreign and Commonwealth Affairs 1994 -- Long Title of an
Act is the part of the Act and is admissible as an aid to its construction.
 gives a general description of the object of the Act and as such, the policy and purpose
of the Act may be derived from its long title.
 Donovan J -- long title may be looked into to resolve that ambiguity or doubt but in
absence of doubt or ambiguity, the meaning of statute cannot be narrowed down or
restricted by reference to long title.
 Lord Moulton in Vacher & Sons v London Society of Compositors 1911-13 -- Title is a
statutory nickname to obviate the necessity of always referring to the Act under its full and
descriptive title.

Short Title

 Short title merely identifies the enactment and is chosen for convenience.
 The short title is always given on the top of the statute book.
 An Act or Regulation may be cited by reference to its short title.
 The object of short title is identification and not description

Use of Titles for resolving ambiguity

 Kedarnath v State of WB 1953


o Section 4 of the West Bengal Criminal Law Amendment (Special Courts) Act 1949
was challenged before the Supreme Court on the ground that it violated Article 14 of
the Constitution.
o Long Title of the Act read, “An Act to provide for more speedy trial and more
effective punishment of certain offences.”
o In the view of the Long Title, the Supreme Court held that the Act was meant to
give discretion to the State Govt. as to which offence deserved to be tried by special
courts under special procedure and therefore contention that Act was violative of
Article 14 was rejected.

Limitations of Titles as Internal Aid

 Title has no role to play when the words employed are plain and precise and bear only one
meaning.
 Title can be called in aid only when there is an ambiguity in the language.
 Title cannot be used to narrow down or restrict the plain meaning of the language.
 Title cannot prevail over the clear meaning of an enactment

PREAMBLE

 Preamble reflects the spirit of the legislation.


 Expresses the scope, object and purpose of the Act more elaborate than the long title.
 Preamble may recite the ground and the cause of making a statute and the evil which is
sought to be remedied by it.
 Can be used when:
o When main enactment gives rise to doubts or where the phrase has more than one
meaning.
o When main enactment is clear but has certain limitations, that is, there is a confusion
in reading the limitation.
 Cannot be used when:
o When Preamble is indicative and not clear.
o to prove vires of legislation
o where Preamble gives general and wide meaning but the unclear enactment however
gives a narrow meaning
o through an amendment new perspective given to certain terms in the Main
enactment - Preamble can't be used to retain the old meaning
o if Preamble is itself repealed through an amendment.
 Anit Das v State of Bihar 2000 SC -- The Preamble is a key to unlock the legislative intent.
 However, it can ONLY be used when the language of the statute is NOT CLEAR.
 statement given in the beginning of the statute.
 Sets out the scope, object and purpose of the Act.
 summary and reflects the gist of law.
 Expression of intention of the legislature in bringing out the enactment.
 It may recite the ground and cause of making the statute, the evils sought to be remedied.
 Supreme Court has held that the preamble can be used as an aid in construing a provision,
when the provision is ambiguous.- Re Kerala Education Bill, 1957
 Lucknow Development Authority v. M.K. Gupta, AIR 1994 SC 787,
o The Supreme Court observed that although the preamble can be a useful assistance
to ascertain legislative intent, but, it cannot be used to control or qualify the precise
and unambiguous language of enactment.
 Burakar Coal Co. Ltd v. Union of India AIR 1961 SC 954
o It is one of the cardinal principles of construction that where the language of an Act
is clear, the preamble must be disregarded though, where the object or meaning of
an enactment is not clear, the preamble may be resorted to explain it.
 Gullipilli Sowria Raj v. Bandaru Pavani (2009) 1 SCC 714
o Word ‘may’ used in Section 5 of the Hindu Marriage Act, which provides : ‘ A
marriage may be solemnized between any two Hindus….’.
o A marriage between a Christian and a Hindu solemnized under the Hindu marriage
Act was held to be void based on reference to the preamble which read as : “ law
relating to marriage among Hindus.”
 State of WB v Anwar Ali Sarkar
o It was contended that Sec.5 of the West Bengal Special Courts Act, 1950, was
unconstitutional and void as it contravened Art. 14 of the constitution.
o That section provided that a special court shall try such offences or class of
offences, or cases or classes of cases, as the state Government may direct.
o It was contended on behalf of the state that the preamble should be read as part of
the section, (The Preamble read, 'whereas it is expedient to provide for the speedier
trial of certain offences') and that the proper interpretation would be ‘only those
cases and offences which in the opinion of the State Government required
speedier trial could be assigned to the special court’.

Constitution Preamble

 Constituent Assembly Debates, Vol. X.


o A resolution was adopted by the Constituent Assembly that “preamble stands part of
the Constitution.”
 In numerous cases it was held that the Constitution, including the Preamble, must be
read as a whole and in case of doubt interpreted consistent with its basic structure to
promote the great objectives stated in the Preamble.
o Keshavananda Bharati v. State of Kerala,
o Minerva Mills v. Union of India,
o Re. Kerala Education Bill.
o The majority judgement in Kesavananda and Minerva Mills strongly relied upon the
Preamble in reaching the conclusion that the power of Amendment conferred by
Article 368 was limited and did not enable Parliament to alter the basic structure or
framework of the Constitution.

Limitations of Preamble as Internal Aid to Construction

 Language of the provision is reasonably capable of alternative construction


 Preamble cannot either restrict or extend the meaning and scope of the words
 In case of conflict between preamble and a section, the preamble would succumb and
Section shall prevail.
 Preamble cannot be used to control or qualify the clear and precise language of enactment
 Preamble cannot limit the operation of a provision
o In Motipur Zamindari Co. v. State of Bihar
 the assessee was asked to pay taxes for 1951-52 and 1952-53 under Bihar
Sales Tax as it fell under category of dealer under section 2 of the Act.
 The Act was amended in 1950 and the Preamble to amending Act stated it
amends the existing act and lays down rules of sales tax under the Act for the
financial year beginning on April 1st 1950. Assessee relied on the preamble
to not pay taxes among other things. Court rejected the appeal.

DEEMING PROVISION

 ‘Deemed to be considered’ vs ‘as may deem fit’


 ‘Deemed to be considered’ is a Deeming provision wherein it is presumed. the latter is not
a deeming provision, instead it is a discretionary provision providing a choice.
 A deeming provision creates legal fiction through presumption. a provision that creates or
gives effect to something that is not there but would take the effect of having been there
because of the legislative authority or sanction.
 Every deeming provision has an assumption and supposition
 Purpose of deeming provision:
o enhance the purpose of the legislation.
o removal of reasonable doubt or point of uncertainty by introduction of an artificial
construction.
 drawbacks of deeming provision:
o not a source of law
o Mere Ad Hoc or additional or alternative remedy to meet the needs of Harsh or
unforeseen circumstances
o there cannot be fiction over fiction
o cannot be used to defeat purpose of law / nullify illegal act, if it gives unjust results
o could be used in cross legislations

INTERPRETATION/DEFINITION CLAUSE Home

 provides the meaning of the words used in the statute so as to avoid any uncertainty of the
meaning of that word.
 The object of having a definition clause is to avoid the necessity of frequent repetitions
in describing all the subject matter to which the word or expression so defined is
intended to apply.
 Serves two purposes
o It provides meaning of the word used in the body of statute
o It helps in avoiding a situation in which the detailed meaning of the word is required
to be given at every place where such a word is used.
 Prima facie the definition governs whenever that word is used in the body of the statute.
 Indian City Properties Ltd. v. Municipal Commissioner of Greater Bombay (2005)
o Section 299(i) of the Bombay Municipal Corporation Act, 1888 was in question
o ‘Building’ appearing in the section was construed by the Supreme Court as
appearing in Section 3(s) of the Act as an inclusive one and therefore can be widely
construed.
 Bharat Coking Coal Ltd. v. Annapurna Construction (2008) 6 SCC 732
o The Apex Court explained the effect of the expression “Unless there is anything
repugnant in the subject or context” in the definition clause. It was held that the said
expression permits departure from defined meaning of a word where the context so
warrants.
 In N.D.P. Namboodiripad (Dead) by LRs v. Union of India (2007) 4 SCC 502.
o The Supreme Court held that where a word is defined there can be no reference
reliance on any general meaning.
 In the absence of any reference or a definition, it is not a sound principle of interpretation to
find the meaning of a word from another statute, particularly when such statute is not
dealing with the same subject matter.
o Agricultural Produce Market Committee v. C.I.T (2008)
 It was held that to seek meaning of words used in an Act, the definition
clause of another statute should not be referred to.
 Indira Nehru Gandhi v Raj Narain
o A definition clause in a statute is a legislative device with a view to avoid making
different provisions of the statute cumbersome.
o Where the effect is that wherever the word defined is used in a provision to which
that definition is applicable, the definition of the word gets substituted.
o Where, however, the definition is preceded by the words "unless the context
otherwise requires", the connotation is that normally it is the definition given in the
section which should be applied and given effect to.
o This normal rule may, however, be departed from, if there be something in the
context to show that the definition should not be applied.

Construction of Definitions

 Restrictive and Extensive Definitions


o Different words or phrases are used to furnish definitions.
o These do not take away the ordinary and natural meaning of the words. They either
extend the meaning or give meaning to ambiguous words.
o ‘Means’- exhaustive.
o ‘Includes’- extends the ordinary meaning.
o ‘Means and includes’- exhaustive.
o ‘denotes’- same significance as ‘includes’.
o ‘Deemed to be’- creates a legal fiction.
o ‘that is to say’- illustrative of definition.
o Many definitions start with “unless the context requires otherwise”
o P. Kasilingam v PSG College of Technology
 “The use of the word 'means' indicates that definition is a hard- and-fast
definition, and no other meaning can be assigned to the expression than what
is put down in the definition.
 The word ‘includes’ when used, enlarges the meaning of the expression
defined so as to comprehend not only such things as they signify according
to their natural import, but also those things which the clause declares that
they shall include.
 The words ‘means and includes’, on the other hand, indicate ‘an exhaustive
explanation of the meaning which, for the purposes of the Act, must
invariably be attached to these words or expressions.
 Borrowed definitions
o Sometimes definition section may borrow definitions from an earlier Act
o The definitions so borrowed may not necessarily be in the definition section but in
some other provision of the earlier Act.
o Public nuisance.
o In absence of such pari materia definitions, the General Clauses Act can be looked
into.
 Ambiguous definitions
o Normally definition is intended by legislature to be precise.
o However, when a definition is ambiguous it requires interpretation for the want of
clarity.
o In order to understand in complete the meaning of the word, it should not be read in
isolation and should require taking help of other provisions or the definition for the
same word provided in different statutes.
o The context and general purpose may also be looked upon.
o Pradyat Kumar v CJ Calcutta
 The apellant was dismissed from his services by the respondent and one of
the contentions raised was that he had no power under law to do so. It was
held that Article 229 (1) also contains power of dismissal.
 The court took help of Section 16(1) of GCA which clearly provides that
‘power of appointment’ includes the ‘power to suspend or dismiss’.

Tools to interpret scope of interpretation/definition clause

 MEANS
o Such definition where the word ‘means’ is used in the definition is prima facie
restrictive or exhaustive in nature and does not permit addition of anything else to
the enumerations already mentioned in the definition itself.
 INCLUDES
o Where the word ‘includes’ is used, it is prima facie extensive in nature. The things
of same kind or genus can be added in the enumerations already provided in the
statute.
 MEANS & INCLUDES
o Such definitions where both the words are used are supposed to be exhaustive
 DEEMED TO INCLUDE
o Where the words ‘deemed to include’ are used, it brings in something within the
scope of the defined word which otherwise would not have fallen under it. Such
definition is considered to be inclusive or extensive
 INCLUDES & SHALL NOT INCLUDE
o Such definitions are both inclusive and exclusive.
 Bharat Coop. Bank (Mumbai) Ltd. v. Coop. Bank Employees Union (2007)
o The Supreme Court held that the use of words “means” followed by the word
“includes” in the definition is clearly indicative of legislative intent to make
definition exhaustive.

Use of Definition for resolving ambiguity

 DELHI JUDICIAL SERVICE ASSOCIATION V. STATE OF GUJARAT AIR 1991


o The words “including the power to punish for contempt of itself” occurring in
Article 129 of the Constitution of India were construed by the Supreme Court that
these words do not limit the inherent power of the Supreme Court to punish for
contempt of itself as also of subordinate courts.
 LUCKNOW DEVELOPMENT AUTHORITY V. M.K.GUPTA AIR 1994
o Whether “Housing Construction” is included in “service” as per the definition
provided in Section 2(o) of the Consumer Protection Act, 1986.?
o Held that “housing construction” was included in “service”. It is worth mentioning
that subsequently, by way of amendment, the words “housing construction” were
inserted between the expressions : “board or lodging or both”

HEADINGS

 Generally headings are attached to almost each section, just preceding the provisions.
 Headings are not passed by legislature but they are subsequently inserted after the Bill has
become law.
 Headings are of two kinds:
o One those which are prefixed to a section and the other;
o Which are prefixed to a group or set of sections.
 Headings can be called in aid while construing a section but the importance attached to
headings as internal aid to construction has been differently described by two groups of
thought.
o One group says that heading is to be regarded as preamble to the provisions
following them and providing key to the interpretation of clauses arranged under it.
 Lord Upjohn -- While construing the Act, the Court must read the headings
as well as the body of the Act and that will always be useful pointer as to the
intention of parliament.
o But the other group says that headings can only be taken when enacting words are
ambiguous.
 Lord Hudson -- The construction of relevant section ought not to be
governed ultimately by consideration of headings even though some
attention may be paid to them.
 N.C. DHOUNDIAL V. UNION OF INDIA (2004) 2 SCC 579
 IQBAL SINGH MARWAH V. MEENAKSHI MARWAH (2005) 4 SCC 370
 Bhinka v Charan Singh
o S. 180 of UP Tenancy Act- ejectment of a person who possessed a land ‘otherwise
than in accordance with the provisions of law’.
o The appellants were in possession of the disputed lands and magistrate under s 145
CrPC, declared that they were entitled to be in possession thereof until evicted
therefrom in due course of law.
o The question before Supreme Court arose whether appellants can be ejected without
having title but a declared possession.
o Supreme Court answered in affirmative.
o The court construed ‘possession in accordance with the provisions of law’ as
‘possession with title’
o They came to this conclusion by looking at the heading of the section which read
‘ejectment of a person occupying land without title’

MARGINAL NOTES Home

 Marginal notes are those notes which are printed on the side of a section, generally in a fine
or small print.
 These notes summarize the effect of the section
 However, marginal notes as an aid to construction is not used nowadays.
 C.I.T. Bombay v. Ahmedbhai Umarbhai and Co.
o Patanjali Shastri J. observed that marginal notes cannot be referred to for the
purpose of construing the statute.
 Tara Prasad Singh v. Union of India AIR 1980 SC 1682
o it was held that marginal notes to a section of the statute cannot take away the effect
of the provisions.
 In Bengal Immunity co. Ltd. v. State of Bihar AIR 1955 SC 661
o it was observed that the marginal notes which are appended to Articles of the
Constitution have been held to constitute part of the Constitution as passed by the
Constituent Assembly.
o For this reason these marginal notes or headings have been made use of in
construing the Articles.
 Balraj Kunwar v Jagatpal Singh
o marginal notes to the sections of an enactment cannot be referred to for the purpose
of construing the act.
o There is no justification for restricting the contents of a section by its marginal
notes.
o They are not part of the act.
o A marginal note is merely an abstract of the clause intended to catch the eye.
 State of Bombay v Bombay Education Society
o It was contended that Art-29(2) did not confer any fundamental right on all citizens
generally but guaranteed the rights of citizens of minority groups.
o Reference was made to the marginal note to Art.29 which states : ‘Protection of
interests of minorities. This contention was rejected by the Supreme Court and it
was held that Article 29(2) applies to all citizens

Limitations of Marginal Notes as Internal Aid to Construction

 Rarely used
 Only if ambiguity persists.
 More than one construction possible.
 Cannot control the plain meaning of the words of the enactment
 Can be used for interpretation of only the Section to which they are appended.
 They cannot frustrate the effect of a clear provision.

EXPLANATION

 Explanation removes the doubt which might arise if the explanation was not given
 Explanation may be appended to include something or to exclude something from the
embracement or ambit of the main enactment.
 It also serves to connote the sense of a word or a phrase occurring in the provision.
 Mithilesh Kumari v. Prem Behari Khare AIR 1989
o An explanation is not a substantive provision, it is merely meant to explain or clarify
certain ambiguities crept in a statutory provision

Objective of explanations

 to explain the meaning and intendment of the Act


 Where there is any ambiguity or vagueness in the main enactment, to clarify the same so as
to make it consistent with the dominant object which it seems to sub-serve.
 To provide additional support to the dominant object of the Act in order to make it
meaningful and purposeful.
 Where some gap is left explanation is necessary to suppress the mischief and advance the
object of the Act.

Limitations of Explanation
 It cannot :
o Curtail or enlarge the scope of section
o Control the plain meaning of the words
 Is meant to further explain the meaning of the provision and to remove the doubts.
 An Explanation can be used only for clarification of that section only to which it is
appended.

PUNCTUATION MARKS

 According to traditional view punctuations cannot be used to construe the Act.


 Like the previous aids they can also be used only when meaning is ambiguous.
 They are always subordinate to the requirement of the context.
 Mohd. Shabir v. State of Maharashtra AIR 1979
o In this case Section 27 of the Drugs and Cosmetics Act, 1940 came before the Court
for construction.
o According to this Section, whoever “manufactures for sale, sells, stocks or exhibits
for sale or distributes” a drug without license is liable for punishment.
o The lower court held that mere stocking is not an offence, however, the Supreme
Court pointed out the presence of comma after “manufactures for sale” and
“sells” and the absence of any comma after “stocks”.
o Consequently, it was held that only stocking for sale could amount to an offence and
not mere stocking.
 Ashwini kumar v Arbinda Bose
o When a statute is carefully punctuated and there is doubt about its meaning, weight
should undoubtedly be given to the punctuation.
o Punctuation may have its uses in some cases, but it cannot certainly be regarded as a
controlling element and cannot be allowed to control the plain meaning of the
statute
 M.K. Salpekar v. S.S. Chandari AIR 1998 SC 1841.
 Whirlpool Corporation v. Registrar of Trade Marks, Mumbai, (1998) 8 SCC 1.

ILLUSTRATIONS

 They are examples or instances appended to a section in order to explain the provision of
law contained in a statute.
 They make the meaning of the section abundantly clear by giving examples.
 Mahesh Chandra Sharma v. Raj Kumari Sharma AIR 1996 SC 869.
o They help to elucidate the principle of the Section.
 Mohamed Sydeol Ariffin v. Yeah Oai Gark 43 IA 256
o Case relates to Section 32 (5) of the Indian Evidence Act, 1872 which provides
statement of relevant fact by person who is dead or cannot be found is relevant.
o An illustration is appended to Sub-Section 5 .
o Lord Shaw, in the given case observed that it is the duty of the court to accept
illustrations given, as being both of relevance and value in construction of text.

PROVISO

 When a statute is framed in general terms, the limitation are recorded by use of proviso.
 A proviso ordinarily carves out an exception from the general rule enacted in the main
provision.
 a proviso is a clause which is added to the section to except something from enacting
clause or to limit its applicability.
 Thus, the intention with which a proviso is added, is to carve out an exception to the main
provision thereby removing something from its scope, which otherwise would have been
included in the section.
 Shah Bhojraj Kuverji Oil Mills and Ginning Factory v. Subhash Chandra Yograj Sinha
AIR 1961 SC 1596
o Hidayatullah J. has observed that as a general rule, a proviso is added to an
enactment to qualify or create an exception to what is in the enactment and
ordinarily a proviso is not interpreted as stating a general rule.
 State of Punjab and Anr v Ashwani Kumar
o “If the language of the enacting part of the statute does not contain the provisions
which are said to occur in it you cannot derive these provisions by implication from
a proviso”.

Limitations of Proviso

 Proviso attached to one section or sub-section cannot be used to qualify another section or
sub-section, nor it can be used to except any thing from any other section.
 The ambit and scope of enacting section cannot be widened or curtailed by the proviso.
 Proviso cannot be used to frustrate the real object of the main enactment, unless the words
of the proviso are such that it is its necessary effect.

SCHEDULES

 Schedules are added in the end of the enactment.


 Schedules are considered to be a part of the statute. They generally provide as to how
claims under the Act can be enforced or as to how the powers vested by virtue of the statute
is to be exercised.
 They mainly contain details as prescribed forms for working out the policy underlying the
sections of the statute.
 M/s. Aphali Pharmaceuticals Limited vs. State of Maharashtra
o The Supreme Court held that, in case of a clash between the schedule and the main
body of an Act, the main body prevails and the schedule has to be rejected.

EXCEPTION AND SAVING CLAUSES

Exceptions

 Exception exempts something which would otherwise fall within the purview of the general
words of a statute.
 For instance, there are ten exceptions attached to section 499, IPC which defines
‘Defamation’. These ten exceptions are the cases which do not amount to defamation.
 Similarly there are five exceptions attached to section 300 of the Indian Penal Code which
defines ‘murder’. These five exceptions are the cases which are not murders but culpable
homicide not amounting to murder.
 An exception affirms that the things not exempted are covered under the main provision.
 In case a repugnancy between an operative part and an exception, the operative part must be
relied on.
 Exceptions must be construed strictly and strongly against the party trying to take the
benefit.
 The mention of certain exceptions to the general rule implies that no other exceptions were
contemplated.

Saving Clause

 Saving clauses are generally appended in cases of repeal and re-enactment of a statute.
 By this the rights already created under repealed enactment are not disturbed nor new rights
are created by it. A saving clause is normally inserted in the repealing statute.
 In case of a clash between the main part of statute and a saving clause, the saving clause has
to be rejected.
Non Obstante Clause
 A section sometimes begins with the phrase ‘notwithstanding anything contained etc.’
 It gives the provision to which it is attached an overriding effect in the event of conflict.
 For reference, in the case of
 Aswini Kumar vs. Arabinda Bose
o “the non obstante clause can reasonably be read as overriding ‘anything contained’
in any relevant existing law which is inconsistent with the new enactment, although
the draftsman had primarily in his mind a particular type of law as conflicting with
the new Act.”

TRAVAUX PREPARATOIRES

 preparatory works.
 It constitutes the materials used in preparing the ultimate form of an agreement or statute,
especially of an international treaty.
 The materials constitute a legislative history.
 Travaux preparatoires contain the various documents including reports of discussions,
hearings and floor debates that were produced during the drafting of a Convention, treaty or
an agreement.
 Travaux preparatoires of a statute or treaty are usually recorded so that it can be used later
in order to interpret that particular statute or treaty.
 This is a secondary form of interpretation and is used to clarify the intent of the makers of
the statute or treaty.
 Travaux Preparatories comes into play in certain strenuous circumstances, when internal
aids breaks down
 Basically its used as an external aid of interpretation

Home UNIT -3

External Aids

- Interpretation is determination of a writing, i.e the art of finding out the true sense of any
form of words. It is the process of ascertaining the meaning of any given text
- When no clue is found from the language of the text about the intention of the legislature,
the aids of interpretation are employed.
1. Historical Facts & Surrounding Circumstances
- If language of statute is unambiguous and clear there is no need to use any external aid
- However, in construction of statutes the state of things at the time is a relevant factor. This
includes the time when the statute was passed, the evils it sought to remedy etc
- Historical facts and circumstances are quite useful in order to understand the subject matter
of the statute and arrive at the legislative intent
- Shrimant Suryavanshi v Bhairoba Suryavanshi (2002 3 SCC 676)
- Historical facts and documents preceding the legislation can be take into consideration
while construing its provisions
- Section 53-A of TP Act inserted based on recommendations of special committee set up
by GoI
- Held that reference to the committee report/recommendations can be taken into
consideration while construing the provisions.
- In Auckland Jute Co. Ltd. v. Tulsi Chandra Goswami it was held that the interpreter
should place himself, as far as possible, in the position of those whose words he is
interpreting and the meaning of certain words and terms used in an ancient document or a
statute can be properly explained only by reference to the circumstances existing at the time
when the statute was enacted or the document was written.
- To sustain the presumption of common of constitutionality, the consideration maybe
had to even matters of common knowledge; the history of the times
and every conceivable state of facts existing at the time of legislation which be the
assumed.
- It is permissible to look into the historical facts and surrounding circumstances for
ascertaining the evil sought to be remedied. Value of "historical evolution" of a provision or
"reference to what preceded the enactment" as an external aid to understand and appreciate
the meaning of a provision, its ambit or expanse has been judicially recognised and
textually recommended.
- But this aid to construe any provision which is "extremely hazardous" should be resorted to,
only, if any doubt arises about the scope of the section or it is found to be "sufficiently
difficult and ambiguous to justify the construction of its evaluation in the statute book as a
proper and logical course and secondly the object of the instant enquiry" should be "to
ascertain the true meaning of that part of the section which remains as it was and which
there is no ground for thinking the substitution of a new proviso was intended to alter".
- But "considerations stemming from legislative history must not, however, override the plain
words of a statute". This rule of interpretation, if applied to a limited extent, may give good
results but if it is unduly extended, it may lead to absurdity.
- Summing up, it is of no doubt that historical facts and surrounding circumstances are so
important that they have been described as two footsteps required for walking on the road.
But on each side, there are some dangers to be avoided.

1. Surrounding Circumstances, State of the Law

1. In order to arrive at the intention of the legislature, the state of the law and judicial
decisions antecedent to and being handed down at the time the statute was being passed are
material matters to be considered.
2. Evidence of matters relating to such surrounding circumstances and historical investigation
of which judicial notice can be taken by the court, including reports of Select Committees
and statements of objects and reasons, can be resorted to for ascertaining such antecedent
law and for determining the intention of the legislature.
3. But the Bill and reports of Select Committees are not legitimate material to be called in aid
for arriving at a construction of a statute, i.e. for finding the meaning of words.
4. Parliamentary debates on the floor of the legislature are also inadmissible because the court
is concerned only with what the legislature actually said in the statute.
5. Moreover, plain words in the statute cannot be limited by any considerations of policy.
6. An erroneous assumption by the legislature as to the state of the law has no effect and does
not become a substantive enactment.
7. In the construction of a statute, the worst person to construe it Is the person who was
responsible for its drafting.
8. Courts sometimes make a distinction between legislative debates and reports of
committees’ and treat the latter as a more reliable or satisfactory source of assistance

2. Posterior Political, Social & Economic Developments and Scientific Inventions


- When a change in social condition produces a novel situation which was not in
contemplation at the time when the statute was first enacted, there can be no a priori
assumption that the enactment does not apply to the new circumstances.
- If the language of the enactment is wide enough to extend to those circumstances, there is
no reason why it should not apply.
- Social expectations and attitudes are also important and have to be considered to upkeep the
law. In Anuj Garg v. Hotel Assn. of India an interesting constitutional situation arose.
Under Section 30, Punjab Excise Act, 1914, the Act prohibited the employment of any man
or woman under 25 years of age to be employed in any part of premises wherein
intoxicating drugs were consumed by public. This was a good law in 20th century but
having regard to the present social conditions and equality of man and woman under the
Constitution, the same was declared invalid.
- However, the judges should not substitute their own views in order to fill the gaps.

General words
- It must be remembered that words are not static vehicles of ideas or concept, their content
will always expand. Global changes and outlook in trade and commerce could be a relevant
factor to consider while interpreting a word, what was not considered a necessity a century
back, may be held to be so now.
- Utilizing the above principle, general words are interpreted to include in their widened and
extended meaning to include new inventions and technological advances.
- Following the same principle, the Supreme Court has held that a "telephone line" in
Telegraph Act, 1885 is wide enough to include electric lines used for the purposes of
wireless telegraph. (Senior Electric Inspector v Laxminarayan Chopra)
- Similarly, the Supreme Court, while dealing with the word textiles, held that it includes
cotton, woollen dry felts, because, it said that the "concept" of textiles is not a static
concept. Having regard to newly developed methods, materials, processes and techniques,
this word gets a continually expanding meaning and consequently new kinds of fabrics that
may be invented can be, without doing any violence to the language, legitimately regarded
as textiles. (Porritts & Spencer v State of Harayana)
- On the same principles, general words are construed to include new inventions and
technological advances not known at the time when the Act was enacted.
- As the content of the ideas or concepts conveyed by respective words expand, so does the
content of the words keep pace with the same expanding content and naturally tend to
widen the field of public interest which the section wants to protect.

Scientific Inventions
- Where in some cases when a statute is to be historically interpreted, it looks as if one were
interpreting it the day after it was passed. However, the courts are free to apply the current
meaning of the statute to the present-day conditions.
- It has been observed in State v. S.J. Choudhary, that the legislature is presumed to
anticipate the temporal development and to intend the Act to be applied in such a way as to
give effect to the tone intention. The statute should be so construed as to continuously
update the wordings in accordance with the changes in social conditions, science and
technology.
- The question in this case was expert opinion about the typewriter on which the questioned
document was typed. The question at hand was whether the term handwriting within the
scope of S. 45 of the IEA would include a typewriting expert.
- This was so because what was understood by handwriting in 1872, must now in the present
times after more than a century of the enactment of that provision, be necessarily
understood to include typewriting as well, since typing has become more common than
handwriting and this change is on account of the availability of typewriters and their
common use much after the statute was enacted in 1872. This was an additional reason for
holding that the opinion of the typewriter expert in this context is admissible under Section
45, Evidence Act, 1872.
- In the same case, it was observed that reading of the word "telegraph" to include
"telephone" within the meaning of the word in Act, 1863 and 1869 was a long accepted
practice of judicial construction when telephone was not invented. The same was, thus,
applied to read "typewriting" within the meaning of the word "handwriting" in the Act of
1872.

3. Parliamentary History Home


Legislative History in India
- It has proper been held that legislative history may be consulted by the courts with proper
understanding & circumspection.
- Similarly, historical evolution of a provision can also be relied on as an external aid to
construction.
- In Doypack Systems (P) Ltd. v. Union of India, earlier Acts, dictionaries, history of
legislation, parliamentary history, parliamentary proceedings, state of law, as it existed
when the Act was passed, the mischief sought to be suppressed and the remedy sought to be
advanced by the Act were recognised as external aids.
a) White Papers- Likewise, the white paper issued by the government, giving details of the
facts leading to enactment of a statute, is admissible for understanding the background,
when the court is called upon to interpret and decide the validity of the statute. It must be
noted that extensive reference was made to the white paper by the Supreme Court for the
purpose of understanding the background of the Rama Janmabhumi Babri Masjid Dispute
and the constitutionality of the acquisition of certain areas of Ayodhya Act, 1993.
b) Constituent Assembly Debates - It was held in Golak Nath v. State of Punjab that the
historical background and, perhaps, what was accepted or what was rejected by the
Constituent Assembly while the Constitution was being framed may be taken into account
in finding out the scope and extent of Article 368.
c) Reports - Reports of the Law Commission can be looked into to understand the history of
the legislation, the object with which certain legal provisions are enacted and what
advantages may be derived by adopting a particular policy. As observed in S.P. Gupta v.
Union of India (1981), the report of the committees of the Law Commission are entitled to
great respect as they are prepared by experienced persons after taking into consideration, all
relevant aspects and sometimes the evidence collected by them from several sources. If they
are to be excluded, many opinions expressed in many of the books have to be excluded.
However, in Maharani Kusumkumari v. Smt Kusumkumari Jadeja where the question
was about protection of legitimacy of children born of void marriage, the court did not rely
upon the Law Commission's report as the relevant sections of Hindu Marriage Act, 1955
were amended and it could safely be deduced that Parliament did not held identical view, as
expressed by the Law Commission's report.
d) Parliamentary History - In Consumer Education and Research Society v. Union of
India, the court, while dealing with the constitutionality of Parliament (Prevention of
Disqualification) Amendment Act, 2006, observed that the expression "office of profit" is
not defined in the Constitution. The view that certain offices or positions held by a Member
of Parliament may be either incompatible with his or her duty as an elected representative
of the people, or affect his or her independence, and thus weaken the loyalty to his or her
constituency and, therefore, should disqualify the holder thereof, had its origin in the
parliamentary history of the UK.
e) Speeches & Debates - Speeches of Members of Parliament are external aids and are in a
similar position as Constituent Assembly debates. The reports of commissions and enquiry
committees, which precede the introduction of a Bill at times, have been used or referred
for interpreting the Act.
In Indira Sawhney v. Union of India, it was observed that Constituent Assembly debates
are external aids. They are not conclusive for interpretation of any expression. The court
cannot ignore the present by going into the past.
f) Amendments to a Bill - It is not correct to contend that acceptance or rejection of
amendments to a Bill, in the course of parliamentary proceedings, forms part of the pre-
enactment history of a statute and, as such, might throw valuable light on the intention of
the legislature when the language used in the statute admitted of more than one
construction. The reason why a particular amendment was proposed, or accepted, or
rejected is often a matter of controversy, and without the speeches bearing upon the motion,
it cannot be ascertained with any reasonable degree of certainty. And where the legislature
happens to be bicameral, the Second Chamber may or may not have known of such reason
when it dealt with the measure. (Ashwini Kumar Ghose v Arabinda Bose 1952)

This principle was again referred to by the Supreme Court and substantially modified in
Express Newspapers Pvt Ltd v UOI (1958). Bhagwati J, speaking for the court observed
that there is a consensus of opinion that the circumstances under which a particular word
came to be deleted from the original Bill as introduced in the Parliament and the fact of
such deletion when the Act came to be passed in the final shape are not aids to the
construction of the terms of the statute.18. He, however, added a rider: "It is only when the
terms of the statute are ambiguous or vague that resort may be had to them for the purpose
of arriving at the true intention of the Legislature", and although asserting that it was not
necessary in that case to refer to the circumstance of deletion of the word "minimum" from
the bill, he proceeded in the next paragraph to consider that very circumstance and to hold
that it had the effect of widening the enquiry before the wage-Board

For instance in CIT v. Mahindra and Mahindra Ltd. the court relied on the Minister's
speech, the notes on clauses of the Bill and the memorandum for explaining the expression
"financial non-viability" which was not defined in the Act.

g) Statement of Objects - Bills and the statement of objects and reasons of a Bill are not
material for taking into account to construe provisions where they are absolutely clear.
Thus, the courts have to be careful before making use of this device of interpretation. .
(Ashwini Kumar Ghose v Arabinda Bose 1952)

In State of W.B v UOI Justice Sinha reiterated the same in the following words –
“It is however well- settled that the Statement of Objects and Reasons accompanying a bill,
when introduced in Parliament, cannot be used to determine the true meaning and effect of
the substantive provisions of the statute. They cannot be used except for the limited purpose
of understanding the background and the antecedent state of affairs leading up to the
legislation. But we cannot use this statement as an aid to the construction of the enactment
or to show that the legislature did not intend to acquire the proprietary rights vested in the
State or in any way to affect the State Governments' rights as owners of minerals”

In the said case a clear declaration in the Statement of Objects and Reasons that "the
Central Government does not intend to acquire the proprietary rights vested in the States"
was held to be ineffective to cut down the generality of the words used in the statute which
was construed as conferring power upon the Union to acquire the rights of States in coal
bearing lands.

4. Reference to Other Statutes

1. Statutes in Pari Materia


- The statute has to be read as a whole and the words therein are to be understood in their
context. This is the main rule and as an extension of this rule of context, it is permitted to
refer to other statutes in pari materia. Statutes in pari materia mean statutes dealing with the
same subject-matter or forming part of the same system.
- The rule of reading the statute as a whole, when extended in its application, permits
reference to other statutes in pari materia.
- According to Sutherland Statutes are considered to be in pari materia-to pertain to the same
subject-matter-when they relate to the same persons or thing or have the same purpose or
object.
- A statute must be read as a whole because its words are to be understood in their context.
This rule of context can be extended to refer to other statutes in pari materia, i.e. statutes
dealing with the same subject matter. The word context has a wide meaning which means
including other statutes in pari materia.
- Meaning of the phrase in pari materia is explained in case of United Society v. Eagle Bank
of New Haven, when the two pieces of legislation are of different scopes, they cannot be
said at in pari materia. However, it is not necessary that the entire subject matter in both the
statutes should be identical.
- It is a rule of construction that courts may follow decisions about similar provisions
contained in enactments in pari materia.
- The object behind this rule is to avoid contradiction between two statutes dealing with the
same subject. But where two statutes are not in pari materia, the rule does not apply.
- In Babu Khan v. Nazim Khan it has been held that the court, while construing a provision of
an enactment, often follows the decisions by the courts construing similar provision of an
enactment in pari materia.
- But in the Babu Khan case, it is clear that the Madhya Bharat Land Revenue and Tenancy
Act, 1950contains one integrated scheme providing for remedy to a pucca tenant claiming
restoration of possession under Sections 91 and 93 of the Act. The Madhya Bharat Land
Revenue and Tenancy Act, 1950 was repealed by M.P. Land Revenue Code. In repealing
Act, i.e. M.P. Land Revenue Code, there is no provision like Section 93 of the Act.
Therefore, it must be concluded that Sections 91, 92 and 93 of the Act are not in pari
materia with the provisions of Section 250, M.P. Land Revenue Code.
- It is not a sound principle of construction to interpret a provision of an enactment following
the decisions rendered on a similar provision of an enactment when two statutes are not in
pari materia.
- This in pari materia rule has in its favour, the following points:
1. Use of later statute - The rule facilitates the use of a later statute as exposition of
the meaning of an ambiguous expression.
2. Use of earlier statute- By this rule, it is allowed to make use of an earlier statute to
throw light on the meaning of a phrase used in a later statute in the same context.
3. Contradiction avoided- The rule avoids contradiction between a series of statutes
dealing with the same subject.
4. Presumption- It permits to raise a presumption, in the absence of any context
showing a contrary intention, that the same meaning attaches to the same words in a
later statute, as in an earlier statute, if the words are used in similar connection in
two statutes. However, this presumption is rebuttable because it does not necessarily
follow that the same words used in the two provisions carry the same meaning.

2. Assistance of Later Statutes


- A later statute is normally not, therefore, made use of as an aid to construction of an earlier
one. This principle was referred to and was approved in Peddinti Venkata Murali
Ranganatha Desika Iyengar v. Govt. of A.P.
- Trusts and Societies - validity of provision - Section 76 of A.P. Charitable and Hindu
Religious Institutions and Endowments Act, 1987 and A.P. (Andhra Area) Inams (Abolition
and Conversion into Ryotwari) Act, 1956 - whether Section 76 valid piece of legislation -
Section 76 attempts to destroy effect of law in Inams Abolition Act without amending law
under latter - Section 76 founded on erroneous assumption that Inams Abolition Act did not
bind religious or charitable institutions or endowments or holder of patta granted to him and
land was still with institution and treated occupant as encroacher - legislation founded on
erroneous assumption does not have effect of depriving holder of land or their vested right
acquired under Inams Abolition Act - held, Section 76 is invalid to extent it effects right
acquired under Inams Abolition Act. (Peddinti Venkata Murali Ranganatha Desika
Iyengar v. Govt. of A.P.)
- A legislation declaring certain facts, which are nonexistent as existing and thereafter
proceeding on that assumption, may also be held to be ineffective.
- It is clearly established now that the subsequent legislation on the same subject may be
looked into, in order to see what the proper construction to be put upon an earlier Act where
the earlier Act is ambiguous.
- If the subsequent legislation proceeds upon an erroneous construction of previous
legislation, it cannot alter that previous legislation, but if there is any ambiguity in the
earlier legislation, then the subsequent legislation may fix the proper interpretation which is
to be put upon the earlier. This passage was approved by Lord Buckmaster in Ormond
Investment Co. Ltd. v. Betts.
- Thus, a subsequent Act of Parliament affords no useful guide to the meaning of another Act
which came into existence before the later one was ever framed. Under special
circumstances, the law does, however, admit of a subsequent Act to be resorted to for this
purpose but the conditions under which the later Act may be resorted for the interpretation
of the earlier Act are strict; both must be laws on the same subject, and the part of the earlier
Act which is sought to be construed must be ambiguous and capable of different meanings.

3. Reference of Earlier Statute into later statute


- Incorporation of an earlier Act into a later Act is a legislative device adopted for the sake of
convenience in order to avoid verbatim reproduction of the provisions of the earlier Act into
the later.83. When an earlier Act or certain of its provisions are incorporated by reference
into a later Act, the provisions so incorporated become part and parcel of the later Act as if
they had been "bodily transposed into it.
- A distinction has also been drawn between a mere reference or citation of one statute into
another and incorporation. In the former case a modification, repeal or reenactment of the
statute that is referred will also have effect for the statute in which it is referred; but in the
latter case any change in the incorporated statute by way of amendment or repeal has no
repercussion on the incorporating statute.. It is a question of construction whether a
particular former statute is merely referred to or cited in a later statute or is wholly or
partially incorporated therein.
- The distinction between incorporation by reference and adoption of provisions by mere
reference or citation is not too easy to highlight. The distinction is one of difference in
degree and is often blurred. The fact that no clear-cut guidelines or distinguishing features
have been spelt out to ascertain whether it belongs to one or the other category makes the
task of identification difficult. The semantics associated with interpretation play their role to
a limited extent. Ultimately, it is a matter of probe into legislative intention and/or taking an
insight into the working of the enactment if one or the other view is adopted
- The Punjab Pre-emption Act (Punjab Act 1 of 1913) defined "Agricultural land" by
reference to the definition of this expression contained in the Punjab Alienation of Land
Act, 1900, which Act was repealed by the Adaptation of Laws (Third Amendment) Order,
1951, but the Supreme Court held that the repeal of the Punjab Alienation of Land Act,
1900 had no effect on the continued operation of the Punjab Pre-emption Act and the
definition of "Agricultural land" incorporated in it. (Ramsarup v Munshi)
- Section 23 of the Mysore Improvement Act, 1903 provided that the acquisition under the
Act "shall be regulated by the provisions, so far as they are applicable, of the Land
Acquisition Act, 1894".
- Section 23 of the Land Acquisition Act, 1894 sets out the matters to be considered in
determining compensation. One of the matters so set out is the market value of the land. Till
1923, the market value required to be taken into account was the value at the date of
publication of the declaration under section 6. By Act 38 of 1923, section 23 of the Land
Acquisition Act was amended and the market value became relatable to the date of the
notification under section 4.
- In the case of Special Land Acquisition Officer, City Improvement Trust, Mysore v P
Govindan, the question was whether section 23 of the Land Acquisition Act as amended in
1923 will apply to the acquisitions under the Mysore Act or whether such acquisition even
after 1923 will be governed by section 23 of the Land Acquisition Act as it originally stood.
- In holding that the Land Acquisition Act as amended would apply, the Supreme Court held
that a fair interpretation of section 23 of the Mysore Act, 1903 was that it applied whatever
procedure may be for the time being in force regarding matters regulating compensation
under the Land Acquisition Act.
- In reaching this conclusion the court said that section 23 of the Land Acquisition Act, 1923
lays down the procedure for award of compensation and it has to be followed as it exists on
the date of acquisition for no one has a vested right in a particular procedure

5. Textbooks
- Privy Council observed in Collector of Madura v. Mootoo Ramalinga
Sethupathi,
“according to the established rules in our courts, it is not open to the court to go back to the
old text in order to see whether the interpretations placed on them by the recognised
commentators are correct or not.”
- However, for arriving at a true meaning of an enactment, the courts may refer to text books.
It lies within the discretion of the courts to accept or reject the meaning given in the
textbook.
- The courts have often quoted from Manu, Jimutvahana, Kautilya, Vigyaneshwar and
Yajnavalkya with approval. In the well-known case of Kesavananda Bharati, a large number
of textbooks were looked into.

Home
6. Dictionaries
- When a word is not defined in the Act itself, as was the case in Nagulapati Lakshmamma
v. Mupparaju Subbaia, it is permissible to look into a dictionary to find out the general
sense of the word. i.e. the sense in which the word is understood in common usage.
- A word may bear various shades of meaning. Out of these meanings, a suitable meaning in
regard to the context in which the word is used has to be selected or found out. This is
because it is a fundamental rule of construction that the meanings of words and expressions
used in an Act must take their colour from the context in which they appear.
- It has been rightly observed, therefore, by the Supreme Court that contextual construction
has its own importance and significance. Colour and content emanating from context may
permit sense being preferred to mere meaning. Words loose their thrust when read in vacuo.
- Quite often in interpreting a statutory provision, it becomes necessary to have regard to the
subject-matter of the statute and the object which it is intended to achieve.
- It has been observed in Bolani Ores Ltd. v. State of Orissa that the use of a dictionary ad
lib without an analysis of the entire Act, its purpose and its intent, for ascertaining the
meaning in which the legislature could have used the word or expression, may not lead us to
the right conclusion.
- "Dictionaries", as observed by Krishna Aiyar in SBI v. N. Sundara Money. "are not
dictators of statutory construction where the benignant mood of a law, more emphatically
the definition clause, furnishes a different denotation".
- When a plain meaning of the provision brings out what was intended, dictionary meaning is
not to be considered
- Where an expression has been defined in an Act, it will carry the same meaning. In such a
case, it is an unnecessary exercise to find out what is the general meaning of the expression
because the definition given in the statute is the determinative factor. In the Nagulapati case,
it has been held that judicial dictionary meaning cannot be relied upon where there is an
express statutory provision in regard to that matter.
- As observed in State of Orissa v. Titaghur Paper Mills Co. Ltd, the dictionary meaning of a
word cannot be looked at where that word has been statutorily defined or judicially
interpreted but where there is no such definition or interpretation, the court may take the aid
of dictionaries to ascertain the meaning of a word in common parlance, bearing in mind that
a word is used in different sense according to its content and a dictionary gives all the
meanings of a word, and the court has, therefore, to select the particular meaning, which is
relevant to the context in which it has to interpret that word.
- Dictionary meaning is ruled out when word has a definite prevalent meaning. Also when
legislature uses certain words which have acquired a definite meaning over a period of time,
it must be assumed that those words have been used by the legislature in the same sense.
- Furniture- When the word is not defined in the Act, dictionaries may be helpful. In one
case, where meaning of the word "furniture" was not given, dictionary was permitted to be
referred and the meaning given therein was relied upon. (New Chelur Manufacturers v
CCE)
- Timber- Similarly, in the Titaghur Paper Mills Co. Ltd. case, it was held that the dictionary
meaning of a word cannot be looked at where the word has been statutorily defined or
judicially interpreted. But in absence of such interpretation or definition, the court may
obtain the aid of dictionaries to ascertain the meaning of a word in common par. lance.
Here, the court has got to select the proper and suitable meaning out of equivalent meanings
given in dictionary. In this case, it was held that "timber" and sized timber or dressed logs
are one and the same commercial commodity. Moreover, beams, rafters, planks, etc. are
also included in the word "timber".
- Apprentice- The similar was the case in ESI Corpn. v. TELCO where, in case of
interpretation of the word "apprentice", dictionary was permitted to be referred and its
meaning accepted. However, one thing is to be remembered here and, i.e. judicial
interpretation given to the word defined in one statute does not afford a guide to
construction of the same words in another statute, unless the statutes are in pari materia
legislation.
- In Sarin Chemical Laboratory v. CST, according to the dictionary meaning, tooth powder
is regarded both as an item of cosmetic and toilet requisite but the names of articles, the
sales and purchases of which are liable to be taxed given in a statute unless defined in the
statute, must be construed not in a technical sense but as understood in common parlance
meaning "that sense which people conversant, with the subject-matter with which statute is
dealing would attribute to it". In common parlance, a tooth powder is considered as a toilet
requisite. That meaning accords with the dictionary meaning as well.
Technical meaning- If an expression acquires a special connotation in law, it must be
assumed that the legislature had used it in its legal sense and not with reference to common
parlance or dictionary meaning.
7. Foreign Decisions
- The Supreme Court is not bound by decisions of foreign courts. Foreign decisions and
textbooks have only persuasive value. However, there are countries which follow the same
system of jurisprudence as the Indian jurisprudence and these countries have statutes in pari
materia to our statutes.
- The Indian courts have, therefore, permitted the use of such foreign decisions. However, the
use must be with restraint and sobriety.
- Our statutes are couched in English language and they (statutes in English language) are
considered as authoritative.
- The country is in touch with English jurisprudence and English common law for the last
150 years. Similarity, in political thought. But because of these reasons the Indian courts
never accepted the assistance of foreign decisions with closed eyes.
- As held in STO v. Kanhaiya Lal Mukund Lal Saraf, while seeking assistance of such
decisions, prime importance must be given to the language of the Indian statute, the
circumstances, the setting in which the statute is enacted and the Indian conditions wherein
it is to be applied. Moreover, it should not be forgotten that there is always an element of
risk in grabbing ready-made and hasty assistance from foreign decisions.
- Before independence, i.e. pre-Constitution, Indian courts profusely referred to English
decisions, and that was a common practice. A caution was, therefore, given by the Privy
Council to discontinue this practice, but it was not given up. But it must be confessed that
where the language of our Indian statute is not clear, knowledge of English Law and
precedents have proved to be of valuable assistance.
- As observed in CCE v. Sitaram Agarwala, where an Indian Act is modeled on a prior
English Act, decisions construing the provisions of the English Act are referred to as helpful
guide for construing corresponding provisions of Indian Act. Similarly, the courts get
considerable assistance from foreign decisions while interpreting certain provisions of the
Constitution. However, this assistance is again to be obtained and used with care and
caution.
- The Supreme Court, unlike the English court, interpreted and construed the words "damage
caused by a ship" in Section 443, Merchant Shipping Act, 1958 widely, so as to include
therein, not only physical damage to ship but also damage to cargo carried in the ship. For
interpreting these words widely, there were justifiable reasons, and they were, that there was
no other Act in India covering claims for damage to cargo carried into a ship. In England,
this point is covered expressly by a separate Act. (M.V Elizabeth v Harwan Investment &
Trading Pvt Ltd)
- In certain areas, the Supreme Court may strive to be more progressive in interpretation. A
good example is afforded by Gian Devi Anand v. Jeevan Kumar. In this case, a statutory
tenant (i.e. a tenant whose tenancy is determined and who continues in possession because
of statutory protection in Rent Acts) was recognised to hold a heritable interest unless there
be a contrary provision in the statute. Under English Law, a statutory tenant is not
recognised to hold a heritable interest unless it is conferred by the statute. As observed by
Bhagwati J, this contrary view under the English Law proceeds because of reluctance to the
idea that the law is moving forward from contract to status.

CONTEMPORANEA EXPOSITIO
 The maxim means contemporaneous exposition.
 The word “contemporaneous” denotes “of the same time or period” and “exposition”
denotes “explanation”.
 This means that the words should be understood in the sense which they bore at the time
when the statute was passed.
 The word “contemporanea” or “contemporaneous” means of the same time or period and
exposition or exposition means explanation. The meaning is that interpreting a statute or
any other document by referring to the exposition it has received from contemporary
authority.
 Maxwell observed, “It is said that the best exposition of a statute or any other document is
that which is received from contemporary authority.”
 Lord Esher observed, “The first point to be borne in mind is that the Act must be construed
as if one were interpreting it the day after it was passed.”
 Tata Engineering and Locomotive Co. Ltd. v. Gram Panchayat, Pimpri Waghere, the
word ‘houses’ used in Section 89 of the Bombay Village Panchayats Act 1933 was
construed as not limited to dwelling houses but included all buildings whether used for
residence or for commercial purposes.
 The rule of contemporanea exposition was first laid down by Lord Coke, - it is that the
words of a statute will generally be understood in the sense which they bore when it was
passed or in other words they are to be understood as used with reference to the subject
matter in the mind of the legislature and limited to it
 Limitations:
o Contemporaneous interpretation can be called in aid only where the statute is
obscure or ambiguous and its true meaning cannot be ascertained by resort to
intrinsic aids to construction
o The rule does not apply to the construction of modern statutes
o If an ancient error is clearly proved, it acquires no prescription to pass as right in the
construction of Statutes.

UNIT 4 – PRESUMPTIONS IN INTERPRETATION

- Presumptions are principles of common law and applicable to all laws –


assumptions that court take into account in interpreting statutory provisions.
- Purely auxillary interpretative function – involved only if language is not clear.
- If language of a statute is ambiguous, or if it is fairly open to either of two
constructions, the court may and should consider the effects and consequences
which will follow from construing it in the one way or in the other, and adopt that
construction which will best tend to make the statute effectual and produce the most
beneficial results.
- But if the statute plainly expresses the legislative purpose and meaning on its face, it
must be enforced exactly as it stands and without any regard what-ever to the results
which will flow from it.
- When a court is confronted with two or more possible interpretations of a statute, or
of a clause in it, and each appears to be quite as consistent with the language of the
statute as any other, it is necessary to determine which was really intended by the
legislature.

PRESUMPTIONS AS TO LEGISLATURE.

LEGISLATIVE PRESUMPTIONS: Home

· Legislature does not commit mistake.


· Legislature does not waste its words & they know the law, judicial decisions
& general principles of law.
· They are aware that statute will gain an enlarged meaning during the course
of time.
· Earlier interpretation of a word will hold good in future also.
· Intend to respect International Law and obligations.
· Do not make law that authorises indirect torturous conduct.
· Make law which Society consider fair, honest and reasonable.
· Will be careful in the usage of words.
· All mistakes would be rectified through amendments.
· Causus Omissus - gap filled by legislature.
· Legislature not says anything in vain.
· No superfluity in statute-avoid redundancy or surplusage.
· Not a very strong Presumption and be used with careful caution.
· No word is considered superflows, unless there is an absurdity that arises
with the same.
· Technical word – Technical sense.
· Popular meaning not similar to plain and natural meaning.
 Know the course of legislation.
· If words used in repeated statute new statute same - meaning of judicial
decisions be attributed to the same.

Sakal Deep Sahai vs U.O.I (1973) Appellant Assistant office superintendent- promoted as office
superintendent in North Easterrn Railway but reverted with charge sheet of using premium passes,
demoted without proper hearing. Later changes withdrawn - no stigne attached.

Appellant applied to reinctant in the Post of superintendent & payment of arrears of salary. On
reversion order upheld & not informed. Retirement in 1959. Filed a suit in 1962 - claiming arrears
of salary & allowances & for declaration that from date of reversion to retirement.

Railway employee on prescript salary + other benefits. Limitation Act, 1963 - Provisions Act
1908- with the arrears of salaries, what is limitation period?

No mention of salary / arrears, but falls in wages in 1988. Act also mansions residuary list if any
item not expressly mentioned elsewhere. Then, Court held it was wages & not residuary list. 1908
Act repeated by 1963 Act. Disputes with respect to salaries were again no mention in 1963 act.
Presumption that legislature knows laws and also the existing judicial decisions of the court.

When a revamped Act comes in, no change in wages part to include salary. They could have
altered but didn't. Thurs, legislature was aware of its own law & existing law on the subject & had
accepted it. When legislature passes a law, it might remove the basis of judgment. They do not
nullify or attack the Judgment.
PRESUMPTIONS IN AID OF INTERPRETATION

- In construing a doubtful or ambiguous statute, the courts will presume that it was the
intention of the legislature to enact a valid, sensible, and just law, and one which
should change the prior law no further than may be necessary to effectuate the
specific purpose of the act in question.
- It is the bounden duty of the judicial tribunals to assume that the law- making power
has kept within the proper sphere of its authority, and has acted with integrity, good
faith, and wisdom.
- Consequently, if the words of the law are doubtful or ambiguous, or if the statute is
susceptible of more than one construction, the courts will lean in favour of that
interpretation which will reconcile the enactment with the limitations of legislative
power and with the’dictates of justice and expediency.
- Nor will a court inquire into the motives of the legislature or listen to allegations of
fraud or corruption against its members, nor presume that the legislature acted un-
advisedly or mistakenly, or that it failed to investigate the subject-matter of the
proposed statute and to inform itself and exercise its judgment and discretion, or that
it was induced to enact the statute by deception, fraud, or trickery practiced upon it.
- The object of all construction and interpretation is to ascertain the meaning and
intention of the legislature. If the meaning is obscure, or the intention doubtful, the
courts should seek it out. And in this search, they will be aided by the presumptions.
But if the meaning and intention are clear upon the face of the enactment, there is no
room for construction. In that event, the literal sense of the statute is to be taken as
its intended sense, and the judiciary have nothing to do with considerations of
justice, reason, or convenience. However, the court may consider the past and
present effects of interpreting the statute in a particular way, as well as those which
may be anticipated in the future.

PRESUMPTIONS IN FAVOUR OF STATUTES- CONSTITUTIONAL VALIDITY

- Every Act of the legislature is presumed to be valid and constitutional until the
contrary is shown. All the doubts are resolved in favour of the validity of the Act. If
it is fairly and reasonably open to more than one construction; that construction will
be adopted which will reconcile the statutes with the constitution and the
consequence of unconstitutionality.
- Legislators and judges are bound to obey and support the constitution and it is to be
understood that they have weighed the constitutional validity of every Act they pass.
Hence, the presumption is always in favour of the constitutionality of a statute;
every reasonable doubt must be resolved in favour of the statute not against it and
the courts will not adjudge it invalid unless its violation of the constitution in their
judgements clear, complete, and unmistakable. (Kellogg vs State Treasurer)
- In order to adjudge that an act of the legislature is in violation of the constitution, it
is necessary to be able to show, clearly, how and in what particular it is inconsistent
with the organic law; it is not enough to show that it is impolitic, unwise, or even
absurd.
- Zimbabwe Township Developers (Pvt) Ltd v Louis Shoes (Pvt) Ltd where Georges
CJ said
 “constitutionality is a phrase which appears to me to be pregnant with the possibilities of
misunderstanding. Clearly, a litigant who assert that an Act of Parliament or a Regulation
is unconstitutional must show that it is. In such a case, the judicial body charged with
deciding that issue must interpret the constitution to determine its meaning and thereafter
interpret the challenged piece of legislation to arrive at the conclusion as to whether it falls
within that meaning or it does not. The challenged piece of legislation may, however, be
capable of more than one meaning. If that is the position then if one possible interpretation
falls within the meaning of the constitution and others do not, then the judicial body will
presume that the law-makers intended to act constitutionally and uphold the piece of
legislation so interpreted. ------ because the person alleging unconstitutionality must
establish it, a burden may rest on that person to establish factually that an act does not fall
within the ambit of constitutionality.”

PRESUMPTION AGAINST ALTERATION OF EXISTING LAW MORE THAN


NECESSARY

- Statutes to be in conformity with common law rather than against it


- Change to be brought only if clear and unequivocal language alters it
- Changes must be in consonance with common law and existing statutes à to
discourage undue stabilization/ unsettlement of law
- Illustration: When a revamped Act comes in, no change in wages part to include
salary. They could have altered but didn't. Thurs, legislature was aware of its own
law & existing law on the subject & had accepted it. When legislature passes a law,
it might remove the basis of judgment. They do not nullify or attack the Judgment.
PRESUMPTION THAT LEGISLATION DOES NOT CONTAIN FUTILE
OR NEGATORY PROVISIONS/ STATUTE LAW IS NOT INVALID OR PURPOSELESS

- Presumed that legislature does not intend legislation which is useless.


- Court must determine purpose of legislation and give effect to it.
- Foundation of this presumption is the acknowledgement that legislation has a
functional purpose and object – legislation should be
- R v Forlee- wherein Forlee was found guilty of contravening Act 4 of 1909 by
selling opium. On appeal his Attorney argued that Forlee had not committed an
offence since the Act in question prescribed no punishment. The court relied on the
presumption against futility, finding that a specific offence had been created by the
legislature. The absence of a prescribed penal clause did not render the Act
ineffective, since the court has discretion to impose a suitable form of punishment as
it deems fit.

PRESUMPTION AGAINST RETROSPECTIVITY OR RETROACTIVITY

There is a presumption against enactments having retrospective effect. Even where


retrospectivity is expressly provided for the provision will be strictly construed to keep
retrospectively to plainly applicable circumstances This presumption appears to be stronger in
penal statutes than in civil ones. In civil statutes, although the presumption exists, it is certainly
very weak in character.

The Courts may even introduce a retrospective effect in civil matters if there seems to be enough
justification. The general presumption is that a law should affect future actions only and not those
of the past.

Midland Railway

Co.v. Pye : (1861)

 "It manifestly shocks one's sense of justice that an act legal at the time of doing it should be
made unlawful by some new enactment."
- Article 20 (1) of the Constitution of India provides against ex post facto law in
respect of conviction for offences.
 In the case of Mahomed N O v Union Government it was said “the principle that (in the
absence of express provision to the contrary) no statute is presumed to operate
retrospectively is one recognised by civil law as well as by the law of England. The law-
giver is presumed to legislate for the future…”
- Rule: statutes regulate future conduct; statutes applies prospectively
- A person cannot be convicted of an act that he committed if at the time it was not an
offence. Further, where a statute increases penalty then the increased penalty will
not be retrospective.

PRESUMPTION AGAINST EXCEEDING CONSTITUTIONAL POWERS

· 3 Lists in the constitution that can be seen to delegate the powers of the Legislature and
Parliament

 Parliament has exclusive power to make laws with respect to any of the matters in List I
(Union List)
 State Legislatures have exclusive power to make laws with respect to matters in List II
(State List)
 Regarding List III (Concurrent List), both, Parliament and the State Legislatures, have
power to make law

There is a presumption of constitutionality of the rule or the legislation, unless ex facie it violates
the fundamental rights.

Thus, there is a presumption that the legislature does not exceed its jurisdiction and the burden of
establishing that the Act is not within the competence of the legislature, or that it has transgressed
some constitutional mandates, such as those regarding fundamental rights, is always on the person
who challenges the vires

There is a general presumption that a legislature does not intend to exceed its jurisdiction and
the general words in a statute are to be construed with reference to the powers of the legislature
which enacts it.

No lacune left
 The presumption is that legislature does not leave any lacuna.
 Either by negligence, or by lack of foresight, or because it did not know its job, it has left
some lacuna-no such presumption be made.

· Utkal Contractors & Joinery (P) Ltd. v. State of Orissa(1987)


 Parliament is neither expected to use unnecessary expressions nor is expected to express
itself unnecessarily.
 As it does not use word without meaning something, it does not legislate where no
legislation is called for.

PRINCIPLES OF NATURAL JUSTICE

- It has been clearly established in our law that when a statute authorizes judicial or
quasi judicial powers which may influence individual or property rights, there is a
presumption that, in the absence of an express provision or a clear intention to the
contrary, the powers so given are to be exercised in accordance with the principles
of natural justice.
- Principles : Audi alteram partem + Nemo Judex (person must not be judge in his
own cause)
- Zimbabwe Teachers Association & Ors v Minister of Education in this case
the Government summarily dismissed all striking teachers who had not heeded its
call for their return to work by a set date. The teachers challenged their dismissal on
the basis that the dismissal was unlawful for failure to comply with the audi et
alteram partem rule. The court held that the dismissal was unlawful.
- Sakal Deep Sahai vs U.O.I (1973) Appellant Assistant office superintendent-
promoted as office superintendent in North Easterrn Railway but reverted with
charge sheet of using premium passes, demoted without proper hearing. Later
changes withdrawn - no stigma attached.

PRESUMPTION AGAINST INTERPRETING STATUTE SO AS TO OUST OR


RESTRICT JURISDICTION OF SUPERIOR COURTS

- Presumption against legislature interfering with jurisdiction of courts which is


related to presumption against changing existing law.
- presumption rests on the theory that the law-giver no more intends to affect the state
in its judicial than in its executive organ.
- In favour of horizontal division of powers and independence of judiciary.
- Archipelago (Pvt) Ltd and Sarah Investments (Pvt) Ltd v Liquor Licencing Board
1986 said “the jurisdiction of the High Court to review the decision of any inferior
court is enshrined in the High Court Act and the presumption against this
jurisdiction being ousted in any case or against its exercise being delayed pending
the pursuit of internal remedies is particularly strong”
Part 2
Home

PRESUMPTIONS IN FAVOUR OF STATUTES- TERRITORIAL OPERATION

- It is presumed that the legislature does not design any attempt to transcend the
rightful limits of its authority, to violate the principles of international law, or to
give exterritorial effect to its statutes. In case of doubt or ambiguity, the construction
will be such as to avoid these consequences.
- It must be assumed that the legislature has intended to keep within the prescribed
limits of its authority, and to enact a valid law. Hence, if a statute is, fairly
susceptible of two interpretations, one of which would make it transcend the
boundaries of legislative competence, and the other would make it valid, the latter
interpretation is to be adopted. And a construction involving the exercise of a
doubtful power will not readily be adopted in the absence of direct words, when the
words used admit of an- other construction which steers clear of all, questions in
regard to power. Prima facie, every statute is confined in its operation to the persons,
property, rights and contracts which are within the territorial jurisdiction of the
legislature when enacted it. The presumption is always against any intention to
attempt giving to the act an extra territorial operation and effect.
- The principle of the separation of the powers of government into three co-ordinate
departments requires that each of these should be independent of the others, and that
neither should usurp the functions nor encroach upon the lawful powers of the
others. Hence any act of legislation which should amount to an unlawful assumption
of either executive or judicial powers, or which should arrogate to the legislative
department duties or prerogatives which the fundamental law confides to the other
branches of the government, would be, for that reason, invalid and of no effect. But
an intention thus to exceed the limits of its rightful power is never to be imputed to
the legislature; On the contrary, the presumption is that it has kept within those
limits. And in case of a doubtful or ambiguous law, the construction should be such
as will reconcile the expressed will of the legislature with the limits fixed for the
sphere of its action and with the proper jurisdiction of the other departments.
Another consequence of the presumption against any abuse of power by the
legislature is that any facts, the existence of which is necessary to the validity of an
act of the legislature, are to be taken for true, as an inference from the statute itself.
- The presumption that statutes do not obtain extra-territorially or do not have extra-
territorial application or do not strike acts committed beyond the limits of
jurisdiction of the legislature roots in respect for the territorial integrity of other
states.

Presumption against exceeding Territorial nexus

 Non-sovereign legislatures are competent to legislate with extra territorial effect.


 Law made by such a legislative body bear a real territorial connection with the subject-
matter with which it is dealing.
 Article 245 - No law made by Parliament shall be deemed to be invalid on the ground that it
would have extraterritorial nexus.
 The presumption that statutes do not obtain extra-territorially or do not have extra-territorial
application or do not strike acts committed beyond the limits of jurisdiction of the
legislature roots in respect for the territorial integrity of other states.

PRESUMPTION AGAINST INJUSTICE

- It is presumed that the legislature never intends to do injustice. If a statute is


doubtful or ambiguous, or fairly open to more than one construction that
construction should be adopted which will avoid this result.
- In construing statutes, it is not reasonable to presume that the legislature intended to
violate a settled principle of natural justice or to destroy a vested right to property.
Courts, therefore, in construing statutes, will always endeavour to give such an
interpretation to the language used as to make it consistent with reason and justice.
(Varick vs Briggs)
- Moreover, it is only when the construction is doubtful that the argument from
injustice or failure of justice is of force. The presumption that the legislature intends
to deal justly is, in a sense, rebuttable; and it is of no value whatever when the
language .of the act is clear and explicit.
- No person shall be allowed to gain an advantage from his own wrong- R vs Chief
National Insurance Commissioner- S.24(1) of Social Security Act, 1975 –“a
woman who has been widowed shall be entitled to a woman’s allowance” –
Notwithdtanding the clear language, court applied principle and held a woman who
has killed her husband and subsequently convicted cannot get the widow’s
allowance u/s.24(1).
PRESUMPTION AGAINST INCONVENIENCE

- It is presumed that the legislature never intends its enactments to work public
inconvenience or private hardship; and if a statute is doubtful or ambiguous, or
fairly open to more than one construction that construction should be adopted which
will avoid such results.
- It is always to be presumed that the legislature intends the most reasonable and
beneficial construction of its enactments, when their design is obscure or not
explicitly expressed, and such as will avoid inconvenience, hardship, or public
injuries. Hence if a law is couched in doubtful or ambiguous phrases, or if its terms
are such as to be fairly susceptible of two or more constructions, the courts, haying
this presumption in mind, will attach weight to arguments drawn from the
inconvenient results which would follow from putting one of such constructions
upon the statute, and will therefore adopt the other.
- Where the language of a statute is plain and admits of but one construction, the
courts have no power to supply any real or supposed defects in such statute, in order
to avoid inconvenience or injustice. It may be proper, in giving a construction to a
statute, to look to the effects and consequences when its provisions are ambiguous,
or the legislative intent is doubtful. But when the law is clear and explicit, and its
provisions are susceptible of but one interpretation, its consequences, if evil, can be
avoided only by a change of the law itself, to be effected by legislative and not
judicial action.
- If the words are ambiguous and one construction leads to enormous inconvenience
and the other construction does not, the one which leads to the least inconvenience is
to be preferred. (Reid v. Reid)

Presumptions Relating to Statutes in Pari Materia

 The presumption is that a consolidating statute, which embodies the provisions of earlier
legislation, does not alter the law.
 There is also the presumption that the judicial interpretation of the preceding legislation
covers the consolidating statute as well.
 This principle however does not debar the Courts from invalidating earlier decisions.
 In the case of a 'codifying' statute, i.e., a statute which replaces earlier enactments in
different terms, the presumption is that the case law under the earlier statute has expired
with them.
 When a later enactment expands the scope of the earlier statutes, the earlier statutes may
themselves be accorded a wider interpretation than they would command had the later
enactment not been passed.
 Where a subordinate legislation coming under an Act is left unchanged by a subsequent Act
repealing the main Act, there is the presumption that the subordinate legislation continues to
be in operation.
 Where a subsequent Act incorporates provisions of the previous Act, then the borrowed
provision becomes an integral and independent part of the subsequent Act and are
unaffected by any repeal or amendment in the previous Act, except certain contingencies.
 There is a rule of construction that where a statute is incorporated by reference into a
second statute, the repeal of the first statute by a third does not affect the second, as the
incorporated provisions have become part of the second statute.

The presumption against interpreting a statute so as to oust or restrict the jurisdiction of


superior courts

 There is a presumption against the legislature interfering with the jurisdiction of courts
which is related to the presumption against changing existing law.
 This presumption rests on the theory that the law-giver no more intends to affect the state in
its judicial than in its executive organ.
 The object of this presumption is to vouch for the horizontal division of powers (or trias
politica) and in particular, for the independence of the judiciary, and to ensure access for
individuals to the courts and to adjudicative procedures.
 Under the doctrine of separation of powers the judiciary stands on an equal footing with the
executive and the legislative pillars of the state.
o Note that it applies only to superior courts.

UNIT 5: STATUTES AFFECTING JURISDICTION


OF COURTS
 Presumption against ouster of established jurisdictions,
 creation of new jurisdictions and enlargement of existing jurisdictions,
 Presumption against violation of International Law,
 Presumption against extraterritorial operation of statutes,
 Presumption that State not affected by statute
PRESUMPTION AGAINST EXCLUSION OR OUSTING THE
ESTABLISHED JURISDICTION Home

 A statutory enactment is not competent to take away the jurisdiction conferred by the
Constitution; this jurisdiction can be taken away only by means of amending the
Constitution.
 Kihoto Hollohan v. Zachillhu (1993)
o observed that even a provision in the Constitution conferring finality to the decision
of an authority is not construed as completely excluding judicial review under
Articles 136, 226 and 227 of the Constitution, but it limits it to jurisdictional errors,
for example mala fides, non compliance with rules of natural justice, infirmities
based on violation of constitutional mandates and perversity.
o If the legislature states that the decision or order of a tribunal, or a court shall be
final and conclusive, the remedies available under the Constitution remain
unrestrained or uninhabited."

THE PRESUMPTION AGAINST INTERPRETING A STATUTE SO AS TO


OUST OR RESTRICT THE JURISDICTION OF SUPERIOR COURTS

 There is a presumption against the legislature interfering with the jurisdiction of courts
which is related to the presumption against changing existing law.
 This presumption rests on the theory that the law-giver no more intends to affect the state in
its judicial than in its executive organ.
 The object of this presumption is to vouch for the horizontal division of powers (or trias
politica) and in particular, for the independence of the judiciary, and to ensure access for
individuals to the courts and to adjudicative procedures.
 Under the doctrine of separation of powers the judiciary stands on an equal footing with the
executive and the legislative pillars of the state.
o Note that it applies only to superior courts.

GENERAL PRINCIPLES PERTAINING TO JURISDICTION OF COURTS

Exclusion must be Explicitly Expressed or Clearly Implied

 The provisions excluding jurisdiction of civil courts and provisions conferring jurisdiction
on authorities other than civil courts are strictly construed.
 There is a strong presumption that civil courts have jurisdiction to decide all questions of
civil nature. Therefore, the exclusion of jurisdiction of civil courts is not to be readily
inferred and such exclusion must either be ‘explicitly expressed or clearly implied’.
 Ramayya v. Laxminarayan, [1934].
o The existence of jurisdiction in civil courts to decide questions of civil nature is the
general rule and exclusion is an exception of this rule. Therefore, the burden of
proof to show that jurisdiction is excluded in any particular case is on the party who
raises such a contention
 Bhimsen v. State of U.P., AIR 1955
o Principle applies to all courts of general jurisdiction including criminal courts.
o Exclusion of jurisdiction of ordinary criminal courts can be brought about by setting
up courts of limited jurisdiction in respect of the limited field, only if the vesting and
the exercise of that limited jurisdiction is clear and operative and there is adequate
machinery for the exercise of the limited jurisdiction.
o Rule against exclusion of jurisdiction of courts will not arise, if the intention of
the legislation is plain, clear and manifest to oust the jurisdiction
 Bhatia international v. Bulk Trading
o It was held that, the court in India would have Jurisdiction even in respect of an
international Commercial Arbitration. An ouster of jurisdiction cannot be implied, it
has to be expressed.

Determining the nature of case (class of case) to understand Jurisdictional


extent of courts

 Tests laid down by Justice Willes


o where there was a liability existing at common law, and that liability is affirmed by
a statute which gives a special and peculiar form of remedy different from the
remedy which existed at common law.
 PVX Granite Co’s case (HoL): legislation in question did not create any
new rights but restricted the already existing rights of a land-owner and the
remedy provided under the statute was alternative and did not take away the
ordinary remedy available under the general law
o Where the statute gives the right to sue merely, but provides no particular form of
remedy, there, the party can only proceed by action at common law.
o Where a liability not existing at common law is created by a statute which at the
same time gives a special and particular remedy for enforcing it.
 Secretary of State v. Mask & Co;
 The liability to pay any particular tax does not exist apart from the
statute imposing the tax. The subject is bound to resort to the
remedies provided in the statute for challenging any assessment of
the tax. It was therefore, held that an order of customs authorities
passed under section 182 of the Sea Customs Act, 1878, and
confirmed in appeal under section 188, was not open to challenge in
civil court.
 With respect to first and third of the three classes of cases mentioned by Willes J it has to be
ascertained
o whether the statute in question deals with and regulates an already existing right or
liability, or,
o whether it creates a new right or liability which has no existence apart from the
statute.
 the special remedy provided therein, subject to any provision for the exclusion of ordinary
remedy, will only be construed as an alternative one.
 the remedy will be construed as exclusive even though the statute makes no express
provision for exclusion of ordinary remedy

Cases of breach of statutory duties

 Doe d. Bishop of Rochester v Bridges: Lord Tenterden CJ said:


o When an Act creates an obligation and enforces the performance in a specified
manner, we take it to be a general rule that performance cannot be enforced in any
other manner.
o If an obligation is created but no mode of enforcing its performance is ordained, the
common law may, in general find a mode suited to the particular nature of the case.
 When a statute creating the duty provides for the penalty of fine or imprisonment for
breach of the duty it is regarded as the only manner of enforcing the duty.
 Premier Automobiles Ltd. v. Kamlakar Shantaram, AIR 1975
o When a statute creating the duty provides for the penalty of fine or imprisonment for
breach of the duty it is regarded as the only manner of enforcing the duty.

Omission to exercise statutory power

 Subject to exceptional case, the normal rule is that an omission by a public authority to
exercise a statutory power conferred for the benefit of the public does not give rise to
breach of duty sounding in damages.
 Stovin v. Wise [1996]
o a motor accident took place at a road junction partly because the view was
obstructed by an earth bank adjacent to the road. Although under sections 41 and 79
of the Highways Act, 1980 the local authority has statutory power to remove the
earth bank but it took no steps in that direction.
o The House of Lords held that there was no common law duty on the authority to
exercise the power and omission to exercise it did not give rise to a claim for
damages in negligence
 Union of India v. United India Insurance Company ltd
o Facts: Express Train- collided –bus
o UOI, which owning railways was held guilty of negligence of breach of common
law duty for failing to convert unmanned level into manned level.
o UOI was also held liable for omission to exercise the power under section 13 of the
Railways Act, which provides that the Central Government may inquire a railway
administration to erect fences, screen gates etc.

EXTENT OF EXCLUSION

Construction of Exclusionary Clauses

 If there is no provision to enable an authority or tribunal to hold an enquiry, the same


indicates that civil courts jurisdiction is not excluded.
 If a provision setting up of hierarchy of judicial tribunal exists, it is sufficient to indicate or
to infer that court’s jurisdiction is barred.
 Tribunal may use the principles of procedure contained in the CPC unless inconsistent with
the provisions of the Act
 Even in cases where jurisdiction is excluded by use of prima facie comprehensive language,
it is open to civil courts which are courts of general jurisdiction to decide whether a court,
or tribunal or authority having limited jurisdiction, has acted in excess of its statutory
powers.
 However, civil courts can interfere when the order of the tribunal or authority is really not
an order under the Act conferring special jurisdiction but is a nullity.
 Gaekwar Sarkar of Baroda v Gandhi Kachrabhai
o “It has been determined over and over again that if a person or body of persons
having statutory authority for the construction of works exceed or abuses the powers
conferred by the Legislature, the remedy of a person injured in consequence is by
action or suit, and not by a proceeding for compensation under the statute which has
been so transgressed. Powers of this sort are to be exercised with ordinary care and
skill, and with some regard to property and rights of others. They are granted on the
condition sometimes expressed and sometimes understood, but if not expressed
always understood that the undertakers shall do as little damage as possible in the
exercise of their statutory powers.”
 Firm Radhakrishnan v Ludhiana Municipality
o A suit in civil court will always lie to question the order of a tribunal created by a
statute, even if its order is, expressly or by necessary implication, made final, if the
said tribunal abuses its power or does not act under the Act but in violation of its
provisions

Tribunal’s order when void.

Home
Cases of Nullity

 An order passed without jurisdiction is a nullity


 Jurisdiction is determinable at the commencement of a proceeding-properly assumed-order
passed thereafter will be within jurisdiction.
 Ujjan Bai v State of UP
o shows that an adjudication by a tribunal of limited jurisdiction is void, when:
o (1) action is taken under an ultra vires statute
o (2) the subject matter of adjudication is beyond its competence or the order passed is
such which it has no authority to pass
o (3) the adjudication is procedurally ultra vires being in violation of fundamental
principles of judicial procedure and
o (4) jurisdiction is assumed by wrongly deciding jurisdictional questions of law or
fact.
 Violation of fundamental provisions of the Act under which the tribunal functions and
fundamental principles of judicial procedure make the tribunal's order void.
 Cases of nullity may also arise during the course or at the conclusion of the inquiry.
 O'Reilly v Mackman
o In the following cases the want of jurisdiction is necessary:
 (a) when the tribunal has wrongly determined a jurisdictional question of
fact or law
 (b) when it has failed to follow the fundamental principles of judicial
procedure, For example: has passed the order without giving an opportunity
of hearing to the party affected;
 (c) when it has violated the fundamental provisions of the Act,
 For example: when it fails to take into account matters which it is
required to take into account or when it takes into account extraneous
and irrelevant matters.
 (d) when it has acted in bad faith

Rule of Conclusive Evidence

 The Legislature may make certain matters non-justiciable by enacting rules of conclusive
evidence or conclusive proof.
 Section 4 of the Evidence Act, 1872 “ When one fact is declared by this Act to be
conclusive proof of another, the court shall, on proof of the one fact, regard the other as
proved, and shall now allow evidence to be given for the purpose of disproving it.
 Rajasthan State Road Transport Corp v Poonam Pahwa,
o In a proper case the High Court or this court in the exercise of its special jurisdiction
has the power to determine how far provisions of the statute have or have not been
complied with. But special powers of this court or of the High Court cannot extend
to reopening a finding by the State Government under section 5 of the Act that the
tenant has not actually resided in the premises for a continuous period of six months
immediately preceding the date of the order or under section 6 that the premises had
become vacant at about the time indicated in the order impugned.
o The Legislature in its wisdom has made those declarations conclusive and it is not
for this court to question that wisdom.

EXCLUSION OF JURISDICTION OF SUPERIOR COURTS

Kihoto Hollohan and the concept of exclusion of superior courts’ jurisdiction

The case dealt with the question that if the decision of the speaker of the House pertaining
to defection, under Para 7 of the 10th Schedule of the Constitution could be subjected to judicial
review. The most relevant observations made by the court subsequent to the arguments of both the
parties can be summarised as under:
 By negating the remedy of judicial review by implementation of Para 7, there would be a
complete exclusion of the power of the SC under Art136 and the HC under Art 226-227.
But it was contended that Para 7 is to be observed as a remedy and not a right and due to
this the jurisdiction of the SC and HC would remain ‘unimpaired’.
 The contention that the provisions of the Tenth Schedule, along with the exclusion of
Paragraph 7, would violate the basic structure of the Constitution by affecting the
democratic rights of elected members and, therefore, of the principles of Parliamentary
democracy was observed as unsound and was rejected.
 A provision in the Constitution conferring finality to the decision of an authority is not
construed as completely excluding judicial review under Articles 136, 226 and 227 of the
Constitution
 As the powers of the Supreme Court under Articles 32 and 136 and that of the High Courts
under Articles 226 and 227 of the Constitution are parts of its basic structure, it is
impossible even by Constitution Amendment to deny the power of judicial review.
 “One of the constructions suggested at the hearing was that this expression covers only the
intermediate stage of the proceedings relating to disqualification under para 6 and not the
end stage when the final order is made under para 6 on the question of disqualification. It
was suggested that this construction would be in line with the construction made by this
Court in its several decisions relating to exclusion of Courts' jurisdiction in election disputes
at the intermediate state under Article 329 of the Constitution. This construction suggested
of para 7 does not commend to us since it is contrary to the clear and unambiguous
language of the provision. The expression `in respect of any matter connected with the
disqualification of a member of a House under this Schedule' is wide enough to include not
merely the intermediate stage of the proceedings relating to disqualification but also the
final order on the question of disqualification made under para 6 which is undoubtedly such
a matter. There is thus express exclusion of all courts' jurisdiction even in respect of the
final order.” (have taken this as it deals with the construction of the statute).
o In the face of this clear language, there is no rule of construction which permits the
reading of para 7 in any different manner since there is no ambiguity in the language
which is capable of only one construction, namely, total exclusion of the Jurisdiction
of all courts including that of the Supreme Court and the High Courts under Articles
136, 226 and 227 of the Constitution in respect of every matter connected with the
disqualification of a member of a House under the Tenth Schedule including the
final decision rendered by the Speaker/Chairman, as the case may be. Para 7 must,
therefore, be read in this manner alone.
JUDICIAL REVIEW, CONSTITUTIONAL REMEDY and APPEAL & REVISION.
 The question of curtailing the jurisdiction of the Supreme Court or High Courts as conferred
by the Constitution does not arise in India
 The jurisdiction conferred by the Constitution can be taken away only by amending the
Constitution and not by statutory enactments
 Exception-Article 262(2) of the Constitution- enables Parliament to provide by law that
"neither the Supreme Court nor any other court shall exercise jurisdiction" in respect of any
dispute relating to waters of inter-state rivers or river valleys
 The law enacted by Parliament in this context is Inter-State Water Disputes Act, 1956
which provides for constitution of Water Disputes Tribunal for adjudication of such
disputes and section 11 of which bars the jurisdiction of all courts including the Supreme
Court in terms of Article 262(2).
 Interpreting this provision it has been held that the bar under section 11 will come into play
when a Tribunal is constituted and till then the Supreme Court can issue interim order
preserving the status quo.
 If the Legislature states that the decision or order of a court or tribunal shall be final and
conclusive, the remedies available under the Constitution remain unfettered.
 The High Court of India apart from exercising supervisory powers under the constitution,
exercise a similar power under the section 115 of the CPC, 1908, over all subordinate
courts. This power of revisions under section 115, which can be excluded by legislative
enactments is construed as readily excluded except by express provision to the effect.
 Statutes should not be construed so as to take away the jurisdiction of the superior courts,
tribunals, government agencies, must be strictly construed and the procedure prescribed
must be followed exactly
 Deep Chand v. Land Acquisition Officer,
o it was held that, the appellate and revisional jurisdiction is not excluded simply
because the subordinate court exercises special jurisdiction.
o This is because when a special act confers a jurisdiction on an established court,
without any words of limitation, then the ordinary procedure of that court including
general right of appeal or revision against its decision is attracted i.e the right is not
taken away.
UNIT 6: GENERAL AND SPECIAL STATUTES,
CLASSIFICATION
( CHECK THE OTHER DOC)

REPEAL OF STATUTES Home

Introduction

 Repeal means to revoke, abrogate or cancel, particularly a statute.


 Repeal maybe expressed or implied by enacting a matter contrary to and inconsistent with
the prior legislation.
 General Clauses Act – Section 6, Repeal is defined as; “abrogation/obliteration of one
statute by another.”
 The end effect of Repeal is as though the first statute had never been passed.
 Modification is NOT Repeal.
 Repeal may be Expressed or Implied.
 Kolhapur Cane Sugar Works Ltd V. UOI
o It was held by the court that proceedings shall be discontinued upon Repeal of a
statute unless:
 Covered under Sec 6 of General Clauses Act
 Savings provision

Kinds of Statutes

 Perpetual
o No time period is fixed for its duration.
o Remains in force until Repealed.
o Called perpetual as it is not abrogated by time or by non use of it.
 Temporary
o They come to an end on the expiry of a specified time.
o Cannot be amended post expiry.
o Can be revived only through re-enactment.
o Effect of repeal
 Section 6, general clauses act:
 Revive anything not in force or existing at the time at which the
Repeal takes effect.
 Affect the previous operation of any enactment so Repealed or
anything duly done or suffered thereunder.
 Affect any right, privilege, obligation or liability acquired, accrued or
incurred under any enactment so Repealed.
 Affect any penalty, forfeiture or punishment incurred in respect of
any offence committed against any enactment so repealed.
 Affect any investigation, legal proceedings or remedy in respect of
any such right, privilege, obligation, liability, penalty, forfeiture or
punishment as aforesaid; and any such investigation, legal
proceeding or remedy maybe instituted, or continued or enforced and
any such penalty, forfeiture or punishment may be imposed as if the
Repealing act or regulation had not been passed.
 Repeal by a Temporary Statute
o State of Orissa V. Bhupendra Kumar - Whether the previous statute which has
been Repealed will revive is subject to the construction of the temporary statute.
 Repealed provisions are treated as if they never existed – They apply only to past
transaction.

Effect on Offences

 After expiration no person can be prosecuted under the act.


 If prosecution has not ended before expiry, the proceedings end.
 If offenses are punishable under ordinary laws, but were being tried by special courts –
where such courts were such statutes – trial will continue in ordinary courts.

Types of Repeal Home


 Express:
o By express provision
o No special form of words are necessary
 Implied:
o By necessary implication
o Not preferred by courts
o May be inferred by
 Where there is a direct conflict between two provisions
 When the legislature intended to lay down an exhaustive code in respect of
the subject matter replacing the earlier law
 When the two laws occupy the same field, but in absence of conflict between
general law and special law Repeal cannot be implied
 Doctrine of Implied Repeal
o It is presumed that the legislature knows the existing state of law and that it did not
intend to create any confusion by retaining any conflicting provisions
o The courts in applying this doctrine are merely supposed to give effect to legislative
intent by examining the object and scope of the two enactments
o Ratanlal Adukia v UoI
 in the above case, it was held that where subsequent special legislation on
the same subject is a self contained compete code in itself, legislative intent
is deemed to be, to exclude the earlier general laws on the subject.

Implied Repeal Cases Laws:

 Dr. Tariq Mehmood Mian V. Govt of Punjab


o Courts do not favour Implied Repeal
o The presumption is always against the intention of the legislature to Repeal by
Implication
o The presumption arises due to the presence, generally, of an express statement to
repeal
o An express statement exists, which shows not only legislative intent, but the
knowledge of the legislature on laws that already exist pertaining to the same
subject matter
 Abdul Samad v Ahmed Khan Lodhi
o Only where two Acts are inconsistent with each other, the latter Act will be said to
impliedly Repeal the former
o Court must be satisfied that both Acts cannot be harmoniously constructed
o Implied Repeal is possible only if the statute’s provisions are plainly repugnant to a
subsequent statute
 Jamshed Ali v State
o In this case, the court laid down that where inconsistency exists, the latter special
Law Repeals, by implication, the former general law

Presumption against Repeal

 Courts can lean against implied Repeal.


 If by any fair interpretation both statutes can stand together, there shall be no implied
Repeal if possible, implied Repeal shall be avoided.
 The scope for application of this Doctrine arises only when the latter Statute provides for a
different punishment or procedure for an offense in comparison to the previous Statute.

Expiry and Repeal of Statutes

 A law is repealed when it is revoked, abrogated, or repealed. Any law may repeal
any Act, in whole or in part, by enacting matter that is contrary to and inconsistent
with the prior legislation, either explicitly or implicitly. As a result, it is common for
a statute to state that certain previous statutory provisions are repealed as a result.
Only if the earlier and later legislative provisions are explicitly incompatible can the
courts treat the matter as repealed by implication. When a repealing clause is
abolished, it would not bring back any provisions that were previously repealed by
it, unless the repealing provision was itself repealed.
 While there is a clear intention to revive, it is possible that common law rules will
once again apply. Section 6 of the General Clauses Act of 1897 "The term "repeal"
refers to the full abrogation or obliteration of one statute by another from the statute
book, as if it had never been enacted. "When an Act is repealed, it must be treated as
though it never existed (except in the case of past and closed transactions)."
 No law will guarantee that it will not be repealed. There is nothing that prevents a
Parliament from enacting a law that cannot be changed or repealed in certain
circumstances. Any Parliament has the right to repeal any Act passed by its
predecessors, but it does not have the power to prohibit the repeal of its own Acts or
to bind its successors.
 A legislation may be permanent or temporary. It is permanent because no time limit
is set for its lifetime, and such a law remains in effect until it is repealed, whether
explicitly or implicitly. A permanent law is not in the sense that it cannot be
repealed; rather, it is perpetual in the sense that it is not abrogated by the passage of
time or non-use. A legislation is temporary if its term is limited to a set period of
time, and it expires at the end of that period unless it is repealed earlier. Simply
because the intent of a statute is temporary, as stated in its preamble, the statute
cannot be considered temporary if no defined date for its length is indicated. A
temporary statute's term may be prolonged by enacting a new statute or exercising a
power granted by the original statute. When it comes to
 A temporary Act's existence is only extended; it cannot be claimed that a new statute
has been enacted; but, if the extension is not followed by any significant changes, it
will not be a success.
 It's a case of simple extension. It seems that a temporary law cannot be renewed
until it has expired. By simply changing the same, you can make it more powerful.
The only logical way to resurrect the long-since-expired law is to by re-enacting a
statute in similar terms or enacting a statute specifically stating that the act, which
had previously expired, is now resurrected.
 General Clauses Act of 1897, Section 6 Repealing Effect - Any right, privilege,
duty, or liability gained or accrued under any repealed enactment is protected under
this provision. This provision saves any tax, forfeiture, or sentence imposed in
connection with any crime committed against any enactment so repealed, pending
prosecution and legal proceedings.
 Section 6A deals with Repeal of an Act that modifies the text of an Act or a
Regulation If any Central Act or Regulation enacted after the commencement of this
Act repeals any legislation by which the text of any Central Act or Regulation was
modified, the repeal shall not impact the life of the repealed enactment until a
different purpose occurs.
 Section 7: revival of repealed enactments. - If any act or regulation is required after
the commencement of this Act for the purpose of reviving any enactment, that
purpose must be stated explicitly.
 The Repealing and Amending Act, 2016, is an Act of the Indian Parliament that
repealed 295 Acts and made minor changes to the Sexual Harassment of Women at
Workplace (Prevention, Prohibition, and Redressal) Act, 2013, and the Governors
(Emoluments, Allowances, and Privileges) Amendment Act, 2014. The Act's aim
was to repeal outdated legislation. Any jurisdiction, office, tradition, responsibility,
right, title, privilege, or restriction shall not be revived or restored by this Act.

UNIT 7

Principles of interpretation used for delegated legislation or subordinate


legislation

 "Delegated legislation" refers to legislation enacted by the executive branch, a legislative


entity, a state or other body acting under the authority of the competent legislature. It allows
the bodies that report to parliament to enact their own laws. It is law that is enacted by
anyone or someone other than Parliament. Parliament may grant authority to another person
or entity to render laws by an Act of Parliament. An Act of Parliament establishes the basis
for a specific law and usually only contains a summary of the Act's intent. By delegating
legislative power to other people or bodies, Parliament allows other people or bodies to add
more information to an Act of Parliament.
 Delegated legislation may also be used to make minor modifications to the law, such as
changing the sanctions under a particular statute. A Local Authority, for example, has the
authority granted to them by some regulations to make delegated legislation and to enact
laws that are specific to their jurisdiction. Since there is more delegated legislation passed
each year than there are Acts of Parliament, delegated legislation plays a critical position in
the making of law. Furthermore, delegated law has the same legal status as the Act of
Parliament that established it.
 Delegated legislation may also be used to make minor modifications to the law, such as
changing the sanctions under a specific statute. A Local Authority, for example, has the
authority granted to them by some regulations to enact delegated legislation and make laws
that are appropriate for their jurisdiction. Since there is more delegated legislation passed
each year than there are Acts of Parliament, it plays a critical role in the making of law.
Delegated law therefore has the same legal status as the Act of Parliament from which it
was derived.
 Second, delegated legislation requires those with specific expert expertise to make laws. A
local government, for example, should make laws that are tailored to the needs of their
community rather than making a blanket rule that may or may not apply to them. A single
Local Authority, rather than Parliament, will make a law to meet local needs, and the Local
Authority would have a better understanding of what is best for the community.
 Finally, delegated legislation can be used to address a condition that Parliament did not
expect before the legislation was passed, making it versatile and useful in lawmaking. As a
result, delegated legislation will adapt to evolving societal needs as well as circumstances
that Parliament did not expect when enacting the Act of Parliament.
 Since the Constitution of India is supreme, any act passed by the legislature must comply
with the constitutional requirements, and if it does not, the court may find it
unconstitutional and void.
 Articles 13, 245 and 246 of the constitution define the constitution's express limits. Article
13(1) states that all laws in effect in India immediately prior to the commencement of the
constitution that are incompatible with the provisions of Part III (fundamental rights) are
invalid to the degree of the inconsistency. According to article 13(2), the state shall not
make any law that abridges or takes away the rights granted by part III (i.e. the
Fundamental Rights), and any law that does so shall be invalid to the degree of the
abridgement.
 As a result, the legislature is prohibited from violating the human rights clauses of Section
III of the constitution. If the parent or enabling Act violates part III of the constitution's
Fundamental Rights, the court will declare it unconstitutional and void, and any subordinate
or delegated law enacted as a result of the act will therefore be found unconstitutional and
void.
 The legislative powers of the parliament and state legislatures are all subject to the
provisions of the constitution, according to Article 245. Parliament may enact laws that
apply to all or part of India's territory, and state legislatures may enact laws that apply to all
or part of a state's territory.
 The Karnataka Cauvery Basin Irrigation Protection Ordinance, 1991 was ruled
unconstitutional by the Cauvery Water Disputes Tribunal on many grounds, including that
it had extraterritorial operation in that it interfered with the equal rights of Tamil Nadu and
Pondicherry to the Cauvery River's waters.
 In other words, no legislation passed by Parliament may be declared unconstitutional
because it has extraterritorial effects. The state legislature's rule, on the other hand, may be
questioned on the grounds of extraterritorial operation. If the parent act is ruled
unconstitutional, the delegated acts enacted under it will be declared unconstitutional as
well.
 If the Enabling or Parent Act violates the Constitution's implied cap, it will be
unconstitutional and void, and any delegated legislation enacted under the Act will be
similarly unconstitutional and void. The implicit restriction of the Constitution is that it
cannot delegate basic legislative functions assigned to the legislature by the Constitution.
The determination of legislative policy and its formulation as a code of conduct is the most
important legislative feature. Delegating legislative power requires the legislature to
establish legislative policy and guidelines for the exercise of the delegated power by the
delegate.
 Where the Enabling or Parent Act is in conflict with delegated legislation: When delegated
law is found to be in direct or indirect conflict with the Enabling Act or Parent Act's
provisions, it is deemed ultra vires the Enabling Act or Parent Act. A rule was found invalid
in Delhi Transport Undertaking v. B.R.I. Hajelay because it conflicted with the provisions
of the Enabling or Parent Act. All persons earning less than 350 rupees per month must be
appointed only by the general manager of the Delhi Transport Undertaking, according to
Section 92 of the Delhi Corporation Act, 1957.

UNIT 8: INTERPRETATION OF FISCAL


STATUTES
Home

INTRODUCTION

 Revenue law is the sole creation of statute and cannot be imposed out of common law. This
means that it is a statute alone which can impose an obligation upon citizens to pay a
specified tax.
 Considered to be a special class of statutes, but warrant no special or own rules of
interpretation.
 Attorney General vs. Caltin Ban 1989
o There was no apparent reason for taxing statutes to warrant principles of
construction distinct from those applicable to other statutes. Once a court has
ascertained the subject-matter to which a taxing statute intended to be applied, there
is no scope for the court to go beyond such a conclusion.
 Tax vs Fees
o A tax is imposed for public purpose for raising general revenue of the State.
o A fee in contrast is imposed for rendering services and bears a broad co-relationship
with the services rendered.
 Tax laws are highly complex, complicated and beyond the understanding of a tax-
payer. The words and expressions used are not simple.
 No one can be taxed by implication.
o A charging section has to be construed strictly
 Keshavji Ravji & Co. vs. CIT – [(1990) 183 ITR 1 (SC)
o The need of interpretation arises only when the words used in the statute are on their
own term, ambivalent and do not manifest the intention of the legislature.

Constitutional Provisions

 Article 265 of the Constitution provides: “No tax shall be levied or collected except by
authority of law”
 Article 366(28) of the Constitution which defines Taxation and Tax reads: “Taxation
includes the imposition of any tax or impost whether general or local or special and ‘tax’
shall be construed accordingly”.
 A scrutiny of Lists I and II would show that there is no overlapping anywhere in the taxing
power and the Constitution gives independent sources of taxation to the Union and the
States.
 A taxing statute is not per se a restriction on the freedom under Article 19 (1) (g) of the
Indian Constitution.
 A taxing statute if divisible in nature and partly falls within and partly outside the
Constitution should not be declared wholly ultra vires. The principle of severability
includes separability in enforcement and this principle should be applied in cases of all
taxing statutes
o Cibatul Ltd v Union of India
 the court held that while the charging section may not be ultra vires , the
procedural section could be held to be ultra vires if it exceeded the
constitutional competence of the legislature which enacted it. It was held that
while section 3, the charging section of the Central Excise and Salt Act 1944
was valid, section 4, the machinery or procedural section, was invalid as it
impinged upon the legislative authority of the state.

CANONS/PRINCIPLES OF INTERPRETATION

Home
Strict Construction
 Statutes which impose taxes or monetary burdens must be construed or interpreted as per
the principle of strict construction.
 Logic: Imposition of taxes is also a kind of imposition of penalty, which can only be
imposed if the language of the state unequivocally states so.
 No scope for intendment, presumption, inference or analogy as to tax.
 Rowlatt J. in Cape Brandy Syndicate v. IRC (1921 1 KB 64)
o "In a taxing statute one has to look merely at what is clearly said. There is no room
for any intendment. There is no equity about a tax. There is no presumption as to a
tax. Nothing is to be read in, nothing is to be implied. One can look fairly at the
language used.”
o Also held in the case of CIT v. Ajax Products Ltd. [1965] 55 ITR, 741
 No tax can be imposed by inference or by analogy or by trying to probe into the intention of
the legislature and by considering what was the substance of the matter.
 CIT v. Elphinstone Spg & Wvg Mills Co Ltd.
 CIT v Motors & General Stores Ltd.
o Subject cannot be taxed unless he comes within the letter of the law. “spirit” of the
law argument is invalid.
 Tarulata Shyam v. CIT [1971] 108 ITR 345 (SC)
o There is no scope for importing into the statute words which are not there.
o The rule of literal construction is widely accepted rule for interpreting the taxing
statutes.
o If the language of the statute is clear and unambiguous, we have to accept the plain
meaning even if it leads to some harshness or injustice to the assessee.
o As long as there is no ambiguity in the statutory language, the rule of literal
interpretation has to be applied.
o A dealer or assessee cannot be subjected to tax without clear and unambiguous
words for the purpose of levying the tax which is authorised by law, enacted by the
Parliament or by the State Legislature.
 Mathuram Agrawal v.State of Madhya Pradesh AIR 2000 SC 109
o In the taxing statute a person or a transaction cannot be subjected to tax on the
ground of spirit of the law or by inference or by analogy.
 CIT vs. Calcutta Knitwears (2014) 362 ITR 673 (SC)
o A taxing statute should be strictly construed even if the literal interpretation results
in hardship or inconvenience, common sense approach equity, logic and morality
have no role to play.
 Vidarbha Irrigation Devs. Corpn. vs. ACIT [(2005) 278 ITR 521 (Bom)
o While interpreting tax statute, the function of the court of law is not to give words in
the statute a strained and unnatural meaning to cover and extent its applicability to
the areas not intended to be covered under the said statute.
 CIT vs. Vadilal Lallubhai [(1972) 86 ITR 2 (SC)
o It is not permissible to construe any provision of a statute, much less a taxing
provision, by reading into it more words than its contains
Strict Construction of Charging Sections

 Charging sections – strict construction


 Benevolent And Procedural Sections – liberal construction
o Bajaj Tempo Ltd. 196 ITR 188 (SC)
 A provision in a taxing statute granting incentives for promoting growth and
development should be construed liberally, and since as provision for
promoting economic growth has to be interpreted liberally, the restriction on
it too has to be construed so as to advance the objective of the provision and
not to frustrate it.
 The rule of construction of a charging section is that before taxing any person, it must be
shown that he falls within the ambit of the charging section by clear words used in the
section. No one can be taxed by implication.
 Upheld in WT Commissioner, Ahmedabad v. Ellis Bridge Gymkhana. AIR 1998 SC 120.
 Practical rule of interpretation and generally resorted to while interpreting the sections
pertaining to incentives, exemptions and deductions etc. A provision for appeal should also
be liberally construed.
 Gursahai Saigal v. CIT 1963 AIR 1062.
o Held: Those sections which impose the charge or levy should be strictly construed;
but those which deal merely with the machinery of assessment and collection should
be construed in a way that makes the machinery workable.
 Commissioner Trade Tax vs. DSM Group of Industries
o The object of provisions of taxing statute being to promote the setting of the new
units and to increase the production of goods such provision has to be interpreted
liberally so that the object can be achieved.
 Vodafone International Holdings V. UOI
o Issue- taxability in India of offshore transfer of shares of a Cayman Islands company
by the Hutchison Group to the Vodafone Group.
o Sec 9- charging section for capital gains
o Direct and indirect transfer
o Literal rule and purposive rule of construction

Beneficial Construction
 When construing a fiscal statue, courts must lean their interpretation toward, or in favour of,
the subject rather in favour of the state.
 Thus, where two interpretations are possible, the one which is beneficial to the assessee
would be preferred.
 IRC v. Duke of Wesminister (1936 AC 1)
o Laid down above principle.
o Held: An assessee may arrange his affairs within the bounds of the law so as to
minimize the incidence of tax.
o In cases where there are two interpretations possible, the one which is
beneficial to the assessee would be preferred.
 A statute which is intended for the benefit of the tax-payer must be construed liberally in
favour of the tax-payer with a view to ensuring that benefit to him and not in a narrow and
pedantic manner with an eye to deprive him of the benefit.
o There is thus no equitable construction permitted in a taxation statute, and the
benefit of the doubt in such statutes invariably goes to the subject.

Construction of Penal Provisions

 Strict interpretation
 Prospective in operation and not retrospective
o Reliance Jute & Industries Ltd v. CIT, West Bengal,1980 AIR 251.
 The cardinal principle of tax laws is that the law to be applied to the assessee
is the law in force in the assessment year unless otherwise provided
expressly or by necessary implication. No retrospective effect to fiscal
statute is possible unless the language of the statute is very clear and plain
and allows for the same.
 Presumption of mens rea
o Concealment of income may be presumed by the department (without mens rea) and
the onus of proof lies on the assessee to show that there is no concealment.

Meaning in Common Parlance


 Meaning in common usage, parlance special in commercial and trade circles must be
considered.
 Annapurna Biscuit Manufacturing Co. v. Commissioner of Sales Tax, UP. 1981
o Held: Taxing statutes should be construed in the way in which they are understood
in ordinary language in the area in which the law is in force.
 Dunlop India Pvt Ltd v. Union of India. 1977 AIR 597.
o Supreme Court held “latex” to fall within the meaning of “rubber” for the purpose of
tax.
Fiscal Statute to be Read as a Whole
 The entire content of a fiscal or taxing statute has to be read as a whole, and not in
piecemeal, to find out the intent of the legislature.
 Grasian Industries Limited v. State of Madhya Pradesh. 1964 AIR 1179.
o Held: Any exemption notification in a fiscal statute must be read in its entirety and
not in parts, to find out the meaning

No Presumption as to Tax
 Presumption of tax does not exist with regard to imposition of taxes.
 Mohammed Ali Khan v. Commissioner of Wealth Tax
o Held that no tax can be imposed by inference, analogy or probing into the intention
of the legislature.

Intention of the Legislature

 Doypack Systems Pvt. Ltd. vs. UOI [1998 (2) SCC 299
o It has to be reiterated that the object of interpretation of a statute is to discover the
intention of Parliament as expressed in the Act.
 GEM Granites vs. CIT (2004) 271 ITR 322 (SC)
o the Hon’ble court observed that what one may believe or think to be the intention of
Parliament cannot prevail if the language of the statute does not support that view,
thus object of the statute has to be gathered from language and not on what one
believes or thinks.

Harmonious Interpretation

 Wherever it is possible to do so, the provision must be harmoniously constructed by


avoiding a conflict. A construction which reduces the statute to a futility has to be avoided.
 CIT vs. R. M. Amin (1971) 82 ITR 194 (Guj)
o Every clause of a statute should be construed with reference to the context and other
clauses of the statute so as, as far as possible, to make a consistent enactment of the
whole statute
 CIT vs. West Bengal Industrial Development Corporation Ltd. – [(1993) 203 ITR 422,
430 (Cal)
o Parliament is normally presumed to legislate in the knowledge of, and having regard
to, relevant judicial decisions.
o If, therefore, Parliament has a subsequent opportunity to alter the effect of a decision
on the legal meaning of an enactment, but refrains from doing so, the implication is
that Parliament approves of that decision and adopts it.
o That was amply demonstrated by the amendment of Sec. 36 (1) (viii) made in 1985.

Mischief Rule
 This rule is also one of the cardinal rules of interpretation when the words of a taxing
statute are ambiguous and incapable of a literal interpretation.
 CIT vs. Shahzada Nand & Sons. (1966 ) 60 ITR 392 (SC)

Purposive Interpretation (Golden Rule)

 However the application of this rule in the interpretation of taxing statutes is rather limited
since the literal rule is more often applicable and it is oft remarked that equity and taxation
are strangers
 Vikrant Tyres Ltd vs. ITO (2001) 247 ITR 821, 826 (SC)

EXTERNAL AIDS TO INTERPRETATION Home

Finance Minister’s speech

 There is no bar in resorting to or referring to speech of FM.


 Interpretation of a statute being an exercise in the ascertainment of meaning, everything
which is logically relevant should be admissible – Chunnilal Onkarmal (P.) Ltd. v. UOI
[1996] 221 ITR 459 (MP)
 In Builders Association vs. Union of India (1994) 209 ITR 877 (SC) the court held that the
opinion of law minister and reply of minister in Parliament regarding a taxing provision
cannot be treated as binding on the Court.

Circulars and Interpretation by Tax Authorities

 Navnitlal Zaveri vs. K.K. Sen (1965) 56 ITR 198 (SC)


o The circulars issued by the CBDT would be binding on officers and persons
employed in the execution of the Income-tax Act and the effect of the circular was
taken into account in deciding the constitutionality of a provision contained in the
Act.
 CIT vs. K. Srinivasan and K. Gopalan (1953) 23 ITR 87 (SC)
o The interpretation placed by the department in the Income-tax Manual has been held
not to be a proper guide when the construction of a statute is involved.

Generalia Specialibus Non Derogant

 In the case of overlapping provisions- freedom of the assessee to chose the one imposing a
lighter burden
 CIT v. Shahzada Nand and Sons 60 ITR 392 (SC)
o The Courts have held the expression to mean that when there is a conflict between a
general and special provision, the latter shall prevail.

EXEMPTION FROM TAXATION

 They have to be interpreted strictly and in its entirety and not in parts.
 CIT vs. Dungarmal Tainwala (1991) 191 ITR 445 (Patna)
o An exemption clause in a taxing statute must be, as far as possible, liberally
construed and in favour of the assessee, provided no violence is done to the
language used.
 Where an exemption is conferred by a statute by an exemption clause, that clause has to be
interpreted liberally and in favour of the assessee but must always be without any violence
to the language used.
 The rule must be construed together with the exemption provision, which must be regarded
as paramount.
 If the tax-payer is within the plain terms of the exemption it cannot be denied its benefit by
calling in aid any supposed intention of the exempting authority.
 In Grasim Industries Limited v State of Madhya Pradesh,
o the Supreme Court held that an exemption notification in connection with a fiscal
statute has to be read in its entirely and not in parts.
 In Tata Oil Mills Company Collector of Central Excise,
o there was a notification which exempted imposition of excise duty on ‘such soap as
is made from indigenous rice bran oil’. This oil can be used in making soap only
after it get converted into fatty acid.
o The Supreme Court held that the exemption applied to both rice bran oil and rice
bran fatty acid.
DOCTRINE OF FAIRNESS

CIT v. Vatika Township Pvt. Ltd. (2014)


The SC in this case dealt with the aspect of retrospective application of Tax Amendments.
The notable points are as under:
 The principle of “lex prospicit non respicit” which translates as “the law looks forwards and
not backwards” was duly considered and upheld by the Supreme Court.
 Any retrospective legislation would be contrary to the general principle of law and would
go against the principle under Art.20
 The court relied on the principle of fairness which stands against the surmise of
retrospective application of laws.
 Any law which modifies any accrued right or would impose disabilities should be treated as
prospective laws unless they were covering up for any previous omission.
 The doctrine of fairness is a relevant factor when construing a statute that confers a benefit
without the infliction of any corresponding or allied detriment, and then the it could be
given a retroactive effect.
 The Supreme Court stated that until there is evidence to the contrary, law is believed not to
be meant to have a retroactive effect. The Supreme Court has pointed out that confidence in
the essence of the rule is based on the bedrock that any human being is entitled to organise
his affairs in accordance with current law and should not be surprised when his
arrangements are disrupted retroactively
 In this case, the Court referred to the case of Govt. of India & others v. Indian tobacco
Association (2005) 7 SCC 396 wherein the Doctrine of fairness was held a relevant factor
to construe a statute conferring a benefit, in the context of it to be given a retrospective
operation.
UNIT 9: INTERPRETATION OF PENAL STATUTES
Home
INTRODUCTION

 Remedial Statute - if a Statute is of such a nature that it doesn't make the offender liable to
any penalty in favour of the State, then it is Remedial Statute. It is just implemented to
ensure that mischiefs are cured and then there is a type of social reform.
 Penal Statute - provides for penalties for disobedience of the law and are directed against
the offender in relation to the Statue by making him liable to imprisonment, fine, forfeiture
or any other penalty
 Remedial Statute is constructed and interpreted liberally and Penal Statute is to be
constructed and interpreted strictly
 A criminal statute may not be enlarged by implication or intent beyond the fair meaning of
the language used .
 In a penal law if there appears to be a reasonable dubiety or ambiguity, it shall be decided in
favour of the person who would be liable to the penalisation.
 If a penal provision fairly be so construed as to avoid the punishment, it must be so
interpreted. If there can be two reasonable interpretations of a penal provision, the more
lenient should be made applicable.
 An accused can be punished only if his act falls clearly into the four corners of the law
without resorting to any special meaning or interpretation of the law.
 If a penal provision is capable of two reasonably possible constructions, then the one that
exempts the accused from penalty must be used rather than the one that does not.
 If a penal provision allows accused to go scot-free because of ambiguity of the law, then it
is the duty of the legislature and not of the courts to fix the law. Unless the words of a
statute clearly make an act criminal, it cannot be construed as criminal.
 Penal liability will not be implied by the courts in the absence of clear and unambiguous
words.

Application rule of construction for penal statutes in the modern


context.

 Maxwell identifies four aspects of the rule that penal statutes must be strictly construed:
o the requirement of express language for the creation of an offence;
o strict interpretation of the words setting out an offence;
o fulfilment to the letter of statutory conditions precedent to the infliction of
punishment;
o strict observance of technical provisions concerning criminal procedure and
jurisdiction.
 If two possible and reasonable constructions can be put upon a penal provision, the court
must lean towards that construction which exempts the subject from penalty rather than the
one which proposes penalty.
 The rule of construction stated by Maxwell is that if there are two reasonable construction,
the court must give the more lenient one which will avoid the penalty in any particular case.
 According to Lord Esher, the settled rule of construction of penal section is that ‘if there is
reasonable interpretation which will avoid the penalty in any particular case we must adopt
that construction.
 Rule of construction in penal statutes does not prevent the court from interpreting a statute
according to its current meaning and applying the language to cover developments in
science and technology not known at the time of passing the statute.
 R v Ireland (1987), Psychiatric injury caused by silent telephone calls was held to amount
to assault and bodily harm under the person Act, 1861 in the light of the current scientific
appreciation of the link between the body and psychiatric injury.
 Penal provision cannot be extended by implication to a particular case or circumstances.

STRICT CONSTRUCTION OF PENAL STATUTES

 If the scope of prohibitory words cover only some class of persons or some well defined
activity, their scope cannot be extended to cover more on consideration of policy or object
if the statute.
 Prohibitory words can be widely construed only if indicated in the statute. On the other
hand if after full consideration no indication is found the benefit of construction will be
given to the subject.
 If the prohibitory words in their own signification bear wider meaning which also fits in
with the object or policy of the statute.
 When in a statue dealing with a criminal offence impinging upon the liberty of citizens, a
loophole is found, it is not for the Judges to cure it, for it is dangerous to derogate from the
principle that a citizen has a right to claim that howsoever mich his conduct may seem to
deserve punishment, he should not be convicted unless that conduct falls within the
definition of the crime of which he is charged.
 In M.V.Joshi v M.V Shimpi
o it was held that “it is now well settled that in the absence of clear compelling
language, the provision should not be given a wider interpretation”.
 In R v Hunt 1987
o it was held that A penal statute must be construed according to its plain, natural and
grammatical meaning.
 N.K. Jain v. C.K. Shah, 1991.
o Unless the words of a statute clearly made an act criminal, it shall not be construed
as criminal. If there is any ambiguity in the words which set out the elements of an
act or omission declared to be an offence, so that it is doubtful whether the act or
omission falls within the statutory words, the ambiguity will be resolved in favour of
the person charged
 Feroze N. Dotivalaz v. P.M Wadhwani and Co., (2003)
o Generally, ordinary meaning is to be assigned to any word or phrase used or defined
in a statute.
o Therefore, unless there is any vagueness or ambiguity, no occasion will arise to
interpret the term in a manner which may add something to the meaning of the word
which ordinarily does not so mean by the definition itself, more particularly, where
it is a restrictive definition.
o Unless there are compelling reasons to do so, meaning of a restrictive and
exhaustive definition would not be expanded or made extensive to embrace things
which are strictly not within the meaning of the word as defined.
 Anup Bhushan Vohra v. Registrar General, High Court of Judicature at Calcutta on (16
September, 2011
o contempt proceedings being quasi-criminal in nature, burden and standard of proof
is the same as required in criminal cases.
o The charges have to be framed as per the statutory rules framed for the purpose and
proved beyond reasonable doubt keeping in mind that the alleged contemnor is
entitled to the benefit of doubt.
o Law does not permit imposing any punishment in contempt proceedings on mere
probabilities; equally, the court cannot punish the alleged contemnor without any
foundation merely on conjectures and surmises.
o As observed above, the contempt proceeding being quasi-criminal in nature require
strict adherence to the procedure prescribed under the rules applicable in such
proceedings.
PURPOSIVE INTERPRETATION APPROACH Home

 It is not necessary that courts must always favour the interpretation which is favourable to
the accused and not the prosecution but it may also chose to go for the interpretation which
is consistent with the object provided in the law.
 In State of Maharashtra v. Tapas D. Neogy. (1999) 7 SCC 685.
o the expression ‘any property’ in section 102 of Cr.P.C. was interpreted to be
inclusive of a ‘bank account’ and hence a police officer who was investigating the
matter was justified in seizing the same
 It is a cardinal principle of construction of statute that when language of the statute is plain
and unambiguous, then the court must give effect to the words used in the statute and it
would not be open to the courts to adopt a hypothetical construction on the ground that such
construction is more consistent with the alleged object and policy of the Act.
 In Kirkness v. John Hudson & Co. Ltd. 1955 (2) ALL ERa 345., Lord Reid pointed out as
to what is the meaning of ‘ambiguous’ and held that – “a provision is not ambiguous merely
because it contains a word which in different context is capable of different meanings and it
would be hard to find anywhere a sentence of any length which does not contain such a
word.

SUPPRESSION OF THE MISCHIEF

 The language of the penal statute can also be interpreted in a manner which suppresses the
lacuna therein and to sabotage the mischief in consonance with the Heydon’s Case.
 Ganga Hire Purchase Pvt. Ltd. Vs. State of Punjab AIR 2000 SC 499,
o while interpreting the section 60(3) of Narcotic Drugs and Psychotropic Substances
Act, 1985, the word ‘owner’ was given a wider meaning for the purpose of
confiscation of the vehicle used in furtherance of the offence mentioned therein i.e.
inclusive of the registered owner where the vehicle was purchased under a hire
purchase agreement when all the instalments were not paid by him.

Retrospective Effect

 It is well established that Penal Statutes do not have Retrospective Effect. (Nag PO Nature
v/s Emperor, J.C.R. 7 Rangoon 355).
 In other words, it could be said, that an accused can neither be convicted nor be penalised
by applying any law retrospectively.
 State of Bombay v/s Vishnu Ramchandra, A.I.R. 1961 S.C 307).
o But, Supreme Court did say in a case that if the retrospective effect of an act is in
favour of the accused, then it can be done so.
 Kedarnath v/s State of West Bengal (A.I.R. 1953 S C. 404)
o included a similar question. In this case, the accused was charged with such an
offence for which he can be sentenced or penalised. Later, the amount of fine was
increased by an amendment, Supreme Court held that the amended amount of fine
can be applied with retrospective effect because it shall be violative of Article 20 (l)
of the Constitution.

Mens Rea in Statutory Offences


 Wrongful Act/Actus Reus
 Guilty Intention/ Mens Rea
 Generally IPC- mens rea is indicated- use of words- intentionally, voluntarily,
dishonestly, knowingly etc
 Legislature –may- create an offence of strict liability- where mens rea is not
necessary
 Sarjoo Prasad v. The State of Uttar Pradesh
 The SC held that any person, whether employer or employee contravening
the provisions of section 7 of the Prevention of Food Adulteration Act,
1954, is liable to punishment under section 16, and it is not necessary for
the prosecution to establish that the person concerned had guilty knowledge
or intention or that he knew that the article was adulterated.
 It was pointed out that the Legislature enacted the Act in the larger interest
of maintenance of public health, and the language was wide enough to
cover every person selling adulterated food whether he had the guilty
knowledge or not
 Some statutory offences- Prevention of food adulteration Act, Weights and
Measures Act- liable even without proof of mens rea/guilty knowledge
 Western India Plywood Ltd vs Shri. P. Ashokan
 Whether the respondent, who is an employee of the appellant, can claim
damages from the appellant -the injury suffered by him during the course of
employment when he was already received the benefit under the provision
of the Employees State Insurance Act 1948.
 SC held that Section 53 of the Employees State Insurance Act, 1948, which
provides that an insured person or his dependents will not be entitled to
"any compensation or damages under the Workmen's Compensation Act,
1923 or any other law for the time being in force or otherwise in respect of
an employment injury", was held to bar even claim for compensation or
damages in Torts although the Act is a beneficial legislation.

Remedial v Penal Statutes

 Legislations enacted with the purpose of bringing into effect- social reform- improving
conditions of certain class of persons-who might have not been fairly treated in the past-
remedial statutes
 These statues prohibit certain acts – provide redress/ compensation to the person aggrieved
by such acts
 Do not make the offender liable for any penalty, but merely provides compensation to the
injured party-remedial statute
 Also known as welfare, beneficial /social justice oriented legislations
 Penal statutes- penalties for disobedience of the law•
 Penalties are directed against the offender-in relation to the State- by making him liable to
imprisonment, fine, forfeiture/other penalty

UNIT 10: MIMANSA AND MAXWELL RULES OF


INTERPRETATION
 Mimansa rules of Interpretation Home
 MAXWELL RULES

MIMANSA RULES

 The word Mimansa means enquiry or investigation.


 Mimansa - (from the Sanskrit word for `reflection' or `interpretation')
 An orthodox Hindu philosophy concerned with the interpretation of Vedic texts and
literature and comprising one part dealing with the earlier writings concerned with right
practice and another part dealing with the later writings concerned with right thought, called
Purva Mimamsa, Uttara Mimamsa.
 The Mimansa principles of interpretation were created for resolving the practical difficulties
in performing the yagyas.
 Although the Mimansa principles were created for religious purpose, they were so rational
and logical that they subsequently began to be used in law, grammar, logic, philosophy, etc.
i.e. they became of universal application.

Rules

 The first rule is about the 'preamble' and the 'epilogue' of the work.
o Any work or article begins with a particular purpose, which is noted in the
'preamble' and ends in epilogue when that purpose is achieved.
o Therefore, Mimansakars are of the view that while entering into the discussion of
interpretation one must see the preamble and the epilogue.
 other rule is of that of 'Abhyas'
o that is the repetitive process, meaning thereby that what has been repeatedly said for
achieving the goal, it is because of the author's continuous effort to support his aim
by various reasoning.
o Therefore Mimansakars say that novelty of the work and aim achieved at must also
be looked at while interpreting. The result arrived at also leads to a correct
interpretation
 Then the last rules spoken of by the Mimansakars are of going through the eulogy.
o Almost in every work something is always said in praise of the aim and object.
o They must be ignored while discussing the interpretation.
 The last but most important rule to be followed is to see how the author criticises the
hurdles in his way and supports his cause.

KINDS OF MIMANSA RULES:

1. Vidhi (Obligatory)

 The main obligatory rule is called a Vidhi (or a Nishedha, if it is in negative form).
 Four types
o UtpattiVidhi, or a substantive injunction (agnihotra)
o ViniyogaVidhi, or applicatory rules
o PrayogVidhi, or rules of procedure, mechanical in nature
o AdhikaraVidhis or rules regarding rights and personal competence
2. Non-obligatory
o Athravada (not mandatory) -It is used only in the course of helping the
interpretation and clarification of law. Does not lay down substantice injunctions, do
not affect the society at large. It is a statement that promotes the aim or objective
Also called as preparatory statement .

The axioms Home

RESOLUTION OF CONFLICT (INTERPRETATION TOOLS) UNDER


MIMANSA RULES

 Sarthakya
o It states that every word that is stated in the scriptures consists of meaning and there
is no world which is used there without meaning.
 Laghava
o This rule indicates that in the case a single rule is generated from a particular text or
word, other interpretations giving different interpretations should not be resolved.
 Arthaikatva
o This rule signifies that the single word or the sentence used must be given a single
meaning and should not be given different meanings.
 Gunapradha
o It states that whenever a word denoting the secondary thought stands contrary to the
primary thought, the word should either be corrected as per the primary thought or
should be left. Guna-subordinate, pradhana-primary.
 Samanjasya
o It states that the possible coalition between the word and sentence of the particular
text should be used as per the spirit of the text itself. The opposite coalition of the
words and sentences should not be used.
 Vikalpa
o It states that whenever controversy arises between two texts, any one text should be
adopted as an alternative.Two principles with equal value, real and irreconcilable
contradiction, then the one that s in accordance with equity, justice and good
conscience and is fit to be used, will be used.
Other axioms
 Shruti - It refers that the words should be taken in their simplest meaning.
 Wachan - Wachan indicates that if there arises any conflict regarding the resolving
meaning of the word in terms of tense, the interpretation should be made looking at the
tense used in remaining texts or words.
o In this regard, Maxwell has also given similar kind of rule of interpretation that the
interpretation of particular text should be done in accordance to other texts as well.
o No interpretation of the tenses contrary to the Vedic Wachan is permitted under
Mimansa.
 Linga - This rule states that in the condition where a word cannot stand on its ordinary
meaning, then the technical meaning of that word should be resolved.
o For example, the word denoting the masculine gender also refers to the feminine
gender.
 Wakya - This rule states that whenever a word or a sentence in the text does not give clear
meaning, then the composition of such sentence or word should be taken into concern and
the possible relevant meaning should be resolved.
 Prakaran - When the words or the sentence of text fails to give concrete meaning, the
meaning of those words of sentences should be resolved with the context of the text.

General Rules regarding the application of texts

 The principle of conceptualizing the texts of compulsive, quasi-compulsive and non-


compulsive nature.

Adhyara Principle –

(i) Anushunga

That which is not said, is deemed to be shared. It is read along with adhyara principle which
operates as casus omissus.
 Mahabir Prasad Dwivedi v. State of U.P.
o The “Anusangha” principle was used which states that an expression occurring in
one clause is often also meant for neighboring clause, and it is only for economy
that it is not mentioned in the latter
Two types of Anushunga principle: Tadapakarsha (transfer forward); Tadutkarsha (transfer
backward)

(ii) Anukarsha Principle

 The following five groups are divided under this rule:


o Kanoon/ Vidhi - It defines law as a command which is positive in nature, possesses
meaning and has meaningful objective.
o Nisedh - It refers to such a law which is mandatory in nature. It consists such
provisions which denote “not to be done”. Thus all the texts of Smiritis which
signify the works which should not be done, falls under this category.
o Mantra - Mantra is considered as the formula. It is sometimes mandatory in nature
and sometimes not-mandatory in depending upon the situation.
o Adhikar Vidhi - Adhikar Vidhi states to whom the law applies to. It directs towards
the ownership of result of any action of someone.
o Uha Bichar - The general meaning of Uha Bichar becomes ‘Arguments.’
o Vadh - Vadh is such a principle of interpretation which talks about the exclusion of
some things which are contrary. This principle helps to coordinate conflict of texts.
It says that if two unusual procedures are such that one is former while the other is
latter, the latter would prevail.

Principles having special relation with the interpretation of texts and customs

 According to this rule, Smirities are superior to Shrutis but the customs are superior to
Smirities.
 Such customs can also be denied under undesirable conditions. In the situation the two
customs contradict each other, the customs which is supported by the Sasthras are
followed.
CASES DECIDED BY JUSTICE KATJU

 Sardar Mohd. Ansar Khan v. State of U.P. 1995 Lab IC 1217


o Markandey Katju J. used the “atidesh” principle of Mimansa and held that the same
principle which applies for teachers should also be applied to clerks and hence the
senior in age would be senior.
o Atidesh means “going from the known to the unknown”
 Udai Shankar Singh v. Branch Manager, 1998(2) All CJ 1364
o Markandey Katju J. allowed the petition holding that the literal rule has not to be
followed and instead the “linga or lakshana” principle has to be followed.
o After all, paralysis of the hand was as bad as amputation of the hand, because both
lead to loss of use of the hand.
o The word ‘amputation’ has hence to be regarded as only illustrative and not
exhaustive. The intention had to be seen and that was to pay compensation for loss
of use of hand.
o The text “kaake bhayo dadhi rakshitam” follows.
 Tribhuwannath Misra v. D.I.O.S. Writ Petition no. 17554 of 1990
o “Samanjasya” principle (principle of harmonious construction) was used.
o Using the Samanjasya principle Katju .J. held that ordinarily the senior most teacher
can officiate, but in exceptional circumstances the next in seniority can officiate, but
in such cases reasons and opportunity of hearing should be given to the senior most
teacher.
 B. Premanand & Others v Mohan Koikal & Others:
o “It may be mentioned that it is not stated anywhere in the Constitution of India that
only Maxwell’s Principles of Interpretation can be utilised. We can utilise any
system of interpretation which can help to resolve a difficulty. Principles of
interpretation are not principles of law but are only a methodology for explaining
the meaning of words used in a text. There is no reason why we should not use
Mimansa Principles of Interpretation in appropriate occasions.”
 Uttar Pradesh State Agro Industrial Corporation Ltd v Kisan Upbhokta Parishad and Ors
o The question before the Supreme Court in the present appeal – whether Animal
Driven Vehicles could be understood as “equipment” or “implement”?
o Applying the Mimansa principles to the present case, it was held that tools or
equipment in the context in which it was used was considered to mean things used
by humans with their hands or legs – hence Animal Driven Vehicles or carts would
not come within the purview of the term.
MAXWELL AND MIMANSA RULES

 The Mimansa and Maxwell systems stressed that the spirit of a statute must be seen in order
to correctly interpret it; however, this does not mean that the Court must always follow the
literal rule of interpretation. It all depends on the situation, the subject, and the goal.
 The required arrangements were made, and so on. The terms of a law, according to
Maxwell's scheme, should be interpreted in the way that best fits the subject of the
enactment and the goal that the Legislature is pursuing.
 A thing that is within the letter of a statute is not within the statute unless it is also within
the Legislature's real meaning, and the terms, if reasonably versatile, must be construed in
the sense that is more in line with that intention, even if it is less right grammatically.
 The literal rule of interpretation in the Mimansa system is known as the Shruti (or Abhida)
principle, and it is this principle that is usually applied when reading a text. However, there
are times when we must deviate from the literal law, and we must rely on other values.
Home
Home

You might also like